Craniofacial 01-22 Flashcards

1
Q

A 3-month-old infant presents with scaphocephaly and sagittal suture ridging. Spring-assisted cranioplasty with an endoscopic approach is planned. Which of the following is an advantage of this procedure over open cranial vault remodeling procedures?

A) Decreased need for additional surgeries
B) Greater control of bony movements
C) Improved aesthetic outcomes
D) Less blood loss
E) Lower risk for developing neurocognitive impairment

A

The correct response is Option D.

Spring-assisted cranioplasty or spring-assisted synostosis surgery is a technique to address sagittal craniosynostosis. This procedure is similar in approach to suturectomy/helmet procedures, which involve removal of the fused suture via small incisions, often with the assistance of an endoscope. In spring-assisted cranioplasty, however, no helmet is applied to the skull to reshape it. Rather, two to three stainless steel springs are placed between the now separated parietal bones that are widened over time as the springs expand. These springs need to be removed around 4 months later in a short second operation. Compared with more traditional open cranial vault reconstruction procedures, spring-assisted cranioplasty is associated with decreased blood loss, shorter operative time (even including the second surgery) and hospital stay, and comparable aesthetic and neurocognitive outcomes. The surgeon is, however, compromising control of the postoperative outcome in favor of the smaller surgical footprint and the guaranteed second surgery required to remove the springs.

How well did you know this?
1
Not at all
2
3
4
5
Perfectly
2
Q

The ear anomaly depicted in the photograph shown is most commonly found in patients with which of the following syndromes?

A) Apert
B) Klippel-Trenaunay
C) Stickler
D) Treacher Collins
E) Van der Woude

A

The correct response is Option D.

The photograph depicts a patient with lobular type microtia and represents failure of formation of the pinna of the ear. Patients commonly have stenosis or absence of the external auditory canal, and in cases of lobular type microtia, exhibit a microtic remnant composed of malformed cartilage and fibrofatty tissue that is reminiscent of the ear lobule. This condition may be seen in isolation but is commonly seen in cases of hemifacial microsomia and Treacher Collins syndrome. Treacher Collins syndrome, or mandibulofacial dysostosis, exhibits a broad phenotypic spectrum and is commonly seen with hypoplasia or absence of the zygoma, coloboma of the lower eyelid, lateral canthal dystopia, absence of the medial lower eyelid lashes, mandibular hypoplasia with a steep occlusal plane, and abnormalities of the external ear, including microtia.

Apert syndrome results from FGFR2 mutations Ser252Trp or Pro253Arg and results in multisuture craniosynostosis, mid-face hypoplasia, complex syndactyly, cystic acne, and other less common findings. It is transmitted as an autosomal dominant condition.

Stickler syndrome is inherited in autosomal dominant and recessive patterns. It results from mutations in collagen genes COL2A1, COL11A1, and COL11A2. This may manifest with ocular abnormalities including early-onset cataracts, vitreous anomalies, retinal detachments, and severe myopia. Patients may have sensorineural hearing loss. Mandibular hypoplasia and Pierre Robin sequence is common in these patients.

Van der Woude syndrome results from a mutation of the IRF6 gene and is a common autosomal dominant condition associated with cleft lip and palate. Patients characteristically exhibit lower lip pits.

Finally, Klippel-Trenaunay syndrome is a congenital capillary-lymphatic-venous malformation of the lower extremity with associated soft tissue and bony overgrowth. This condition is caused by a somatic mutation in the PIK3CA gene and is therefore on the phenotypic spectrum with CLOVES syndrome and other PIK3CA-related overgrowth syndromes (PROS).

How well did you know this?
1
Not at all
2
3
4
5
Perfectly
3
Q

A 3-month-old male infant presents with left occipital flattening and associated external ear and facial asymmetry. Significant head tilt and limited range of motion with neck rotation are noted on physical examination. A photograph is shown. Which of the following muscles is most likely contributing to the patient’s plagiocephaly?

A) Digastric
B) Platysma
C) Splenius cervicis
D) Sternocleidomastoid
E) Trapezius

A

The correct response is Option D.

Congenital muscular torticollis occurs as a result of sternocleidomastoid shortening, presenting as head tilt and limited range of motion with neck rotation. The inability to rotate the neck increases the time spent in the preferred position and contributes to deformational plagiocephaly. The digastric muscle acts to elevate the hyoid, while the platysma is involved in facial expression. The splenius cervicis extends the spinal column, and the trapezius functions to maintain posture and stabilize the scapula.

How well did you know this?
1
Not at all
2
3
4
5
Perfectly
4
Q

A 15-year-old girl with a history of facial asymmetry reports worsening of her deformity in the past 3 years. It has slowly progressed with upper lip and cheek atrophy, mid-face hyperpigmentation, nasal deviation, and worsening occlusal cant. Photographs are shown. The patient reports migraine but denies any other medical problems. Which of the following is the most likely diagnosis?

A) Fibrous dysplasia
B) Goldenhar syndrome
C) Neurofibromatosis
D) Parry-Romberg syndrome
E) Systemic scleroderma

A

The correct response is Option D.

Parry-Romberg syndrome is a rare craniofacial disease characterized by slow progressive hemifacial atrophy of the skin, subcutaneous tissue, and, in severe cases, bony structures, following ipsilateral branches of the trigeminal (V) nerve. It usually occurs in the first two decades of life, and it is more prevalent in females. It is frequently associated with neurologic and ophthalmologic involvement. The severity of deformity varies depending on the age of onset of disease, being more severe the earlier it starts.

How well did you know this?
1
Not at all
2
3
4
5
Perfectly
5
Q

Which of the following best describes the growth pattern of the calvarium relative to a fused suture?

A) Oblique
B) Parallel
C) Perpendicular
D) No relation

A

The correct response is Option B.

Virchow’s law states that bone surrounding a prematurely fused suture only grows parallel to the suture, while growth perpendicular to the suture is restricted. This contributes to the characteristic patterns of cranial shape in the various types of synostoses. Sagittal fusion yields a skull that is long in the anteroposterior direction and narrow bitemporally. Metopic fusion yields a triangular-shaped skull due to constriction caused by the anterior, medial metopic suture. Unilateral coronal synostosis yields an asymmetric skull, notably for a trapezoidal appearance from the overhead inspection. While there may be some ridging above and below the affected suture, this does not represent the growth of normal calvarial bone.

How well did you know this?
1
Not at all
2
3
4
5
Perfectly
6
Q

A 7-year-old girl presents to the clinic with poor school performance, speech difficulties, and hypernasality. The patient also has a history of immunodeficiency and hypocalcemia. Clinically, there is no evidence of overt or submucosal cleft palate. Which of the following is the most appropriate next step for her care?

A) Formal speech evaluation
B) MRI
C) Palatoplasty
D) Observation

A

The correct response is Option A.

Velopharyngeal dysfunction can be divided into insufficiency, incompetence, and mislearning. The term insufficiency is related to a structural defect that results in poor closure of the velopharyngeal port. Incompetence is usually associated with neurological or muscular causes to inefficient port closure. Mislearning is less frequent and related to normal anatomy and muscle function, but inadequate production of sounds.

The patient described has many of the medical abnormalities observed in 22q11.2 deletion syndrome. These include, but are not limited to, congenital heart defects, hypocalcemia, immune deficiencies, speech delay/disorders, palatal abnormalities, intellectual disabilities, and psychiatric disorders. Late speech disorder diagnosis is not uncommon and warrants a thorough evaluation in a cleft center; this patient would also benefit from a genetics consultation. Depending on the formal speech evaluation results, further measures of velopharyngeal closure, such as nasometry and pressure flow, as well as imaging with videofluoroscopy and nasopharyngoscopy, might be recommended.

How well did you know this?
1
Not at all
2
3
4
5
Perfectly
7
Q

A 3-month-old infant presents with flattening of the left forehead, and asymmetry of the orbits and face. He has early evidence of right-sided astigmatism and ocular torticollis. A CT scan is shown. The parents are considering two options: endoscopic suturectomy and helmet therapy at age 3 months or fronto-orbital advancement (FOA) at age 9 months. Compared with endoscopic suturectomy and helmet therapy, FOA is more likely to result in which of the following surgical outcomes?

A) Greater need for revision
B) Greater postoperative facial symmetry
C) Less operative morbidity
D) Lower overall cost
E) More severe strabismus

A

The correct response is Option E.

Surgical intervention is recommended for most patients with craniosynostosis due to the variable risk for increased intracranial pressure (ICP), localized cerebral compression, and developmental delay. Traditional treatment techniques such as fronto-orbital advancement (FOA) and total cranial vault remodeling are still widely used, but minimally invasive options such as endoscopic suturectomy and postoperative helmet therapy (ES + HT), spring-mediated distraction, and conventional distraction have become more commonplace. While each technique has its ardent supporters, there is a growing number of studies that compare the clinical outcomes of each technique. ES + HT is most effective when performed under 4 months of age because the correction depends solely on cranial expansion, while spring-mediated distraction is typically done in older infants (4 to 8 months of age). Cranial remodeling has no strict age limit, but many surgeons defer treatment until 7 to 10 months to decrease the risk associated with anesthesia and recurrence of the malformation. Despite these concerns, there are some data that support early cranial expansion (<6 months of age) to optimize neurocognitive outcomes. Compared with FOA, endoscopic and spring-mediated distraction techniques are less costly, less morbid, and have similar revision.

The patient has left unilateral coronal craniosynostosis (UCS), which manifests as ipsilateral forehead and brow retrusion and shortening of the ipsilateral hemi-cranium, with orbital and facial asymmetry. Open cranial vault procedures do not directly correct the facial symmetry and at least two investigations show better facial asymmetry is achieved with early, minimally invasive approaches. The orbital asymmetry results in strabismus (contralateral head tilting termed ocular torticollis) and contralateral astigmatism in the majority of patients. Several studies confirm significantly better ophthalmologic outcomes following early endoscopic suturectomy for UCS versus FOA performed at an older age.

How well did you know this?
1
Not at all
2
3
4
5
Perfectly
8
Q

A 5-year-old boy has an interrupted aortic arch, low-set ears, broad nasal bridge, velopharyngeal insufficiency, and hypoparathyroidism. A photograph is shown. Which of the following additional findings are most likely in this patient?

A) Facial asymmetry, microtia
B) High myopia, retinal detachment
C) Lower eyelid coloboma, downward-slanting palpebral fissures
D) Lower lip pits, salivary mucous drainage
E) T-cell immunodeficiency, chronic otitis media

A

The correct response is Option E.

22q11.2 Deletion syndrome (also known as DiGeorge, velocardiofacial, and conotruncal anomaly face syndromes) is often identified with help of the “CATCH 22” mnemonic, which refers to its characteristics of cleft palate, abnormal facies, thymic hypoplasia, cardiac malformations, and hypoparathyroidism. T-cell immunodeficiency and recurrent otitis media infections occur secondary to thymic hypoplasia and cleft-related ear anomalies.

Van der Woude syndrome is characterized by cleft palate and/or cleft lip, and lower lip pits/fistulae. Abnormal salivary gland morphology and hypodontia may also occur.

Treacher Collins syndrome (mandibulofacial dysostosis) is characterized by malar and mandibular hypoplasia, down-slanting palpebral fissures, lower eyelid coloboma, and ear anomalies.

Stickler syndrome type 1 is characterized by cleft palate, facies such as mid-face hypoplasia/retrusion, myopia with increased risk for retinal detachment, sensorineural hearing loss, and joint abnormalities including hypermobility and early-onset osteoarthritis.

Hemifacial microsomia is characterized by small and/or flattened maxillary, temporal, and zygomatic bones. Facial asymmetry is further apparent upon animation when soft tissue and nerve hypoplasia occur. Oral clefts and ear anomalies are commonly involved, as well as underdevelopment of the orbits.

Van der Woude syndrome, Treacher Collins syndrome, Stickler syndrome, and hemifacial microsomia are all unrelated to hypoparathyroidism.

How well did you know this?
1
Not at all
2
3
4
5
Perfectly
9
Q

A newborn female presents to the neonatal intensive care unit with failure to thrive and dysmorphic features. Initial evaluation demonstrates cloverleaf skull deformity, midface retrusion, severe exorbitism, respiratory compromise, and broad thumbs. Which of the following procedures in the neonatal period will best decrease this patient’s risks for irreversible sequelae due to her disease?

A) Monobloc osteotomy with external fronto-facial distraction
B) Posterior vault distraction osteogenesis
C) Temporary tarsorrhaphies and mandibular distraction osteogenesis
D) Tongue-lip adhesion and fronto-orbital advancement
E) Tracheostomy, decompressive craniectomy, and temporary tarsorrhaphies

A

The correct response is Option E.

Cloverleaf skull deformity (Kleeblattschädel) associated with Pfeiffer syndrome poses significant challenges to the craniofacial surgeon. Fusion of the coronal, lambdoid, and sagittal sutures significantly increases the risk for intracranial hypertension, necessitating urgent surgery to release the fused sutures to allow the brain to grow. Additionally, patients with severe exorbitism may require a temporary tarsorrhaphy early in life to prevent exposure keratopathy and globe prolapse. Tracheostomy should also be performed at this time to allow for adequate ventilation. Tongue-lip adhesion is not adequate to treat this patient’s respiratory compromise. Fronto-orbital advancement and monobloc distraction should not be performed in the neonatal period. Posterior vault distraction would not address this patient’s orbital exposure or critical airway.

How well did you know this?
1
Not at all
2
3
4
5
Perfectly
10
Q

A 48-year-old man is evaluated after a motor vehicle crash, and CT scanning of the head shows a moderately displaced anterior/posterior frontal sinus fracture. The images are shown. The patient also has associated rhinorrhea; the fluid is sent for analysis. Which of the following is the most sensitive diagnostic finding that would indicate treatment of the fracture with cranialization and surgical repair of the dural tear?

A) High bacterial contamination
B) High glucose content
C) Low hemoglobin percentage
D) Positive beta-2 transferrin level
E) Salty postnasal drainage

A

The correct response is Option D.

A frontal sinus fracture involving the anterior and posterior tables with displacement greater than one table width and associated cerebrospinal fluid (CSF) leak mandates treatment with cranialization and repair of the dural tear. While minimally displaced posterior table fractures with CSF leak may be observed for spontaneous resolution of the leak, any significant displacement will likely not allow the dural tear to adequately heal and significantly increases the risk for bacterial contamination and meningitis.

Accurately diagnosing the presence of a CSF leak in conjunction with significantly displaced posterior table frontal sinus fractures is important in deciding the ultimate management of these complex injuries. The beta-2 transferrin level is the most sensitive test for diagnosis of a true CSF leak. While CSF fluid does tend to have a high glucose content, this could also be seen in bloody rhinorrhea as well and is not as sensitive as a beta-2 transferrin test. Likewise, although CSF rhinorrhea often presents as a salty tasting postnasal drainage, this finding is more subjective and has poor sensitivity. Low hemoglobin concentrations are unlikely after acute fractures but can also be present without CSF leak. High bacterial contamination of the rhinorrhea does not correlate with CSF leak, but the risk for bacterial contamination of the meninges leading to meningitis is the reason for aggressive treatment of displaced posterior table frontal sinus fractures with associated CSF leak.

How well did you know this?
1
Not at all
2
3
4
5
Perfectly
11
Q

A 12-year-old girl with Parry-Romberg syndrome presents to the office for treatment. She has mild symptoms of facial soft-tissue atrophy, but is otherwise well, and has not undergone any formal workup. Which of the following next steps is likely to provide the most helpful and useful information for long-term management of her disease?

A) Blood draw to evaluate complete blood count
B) Lumbar puncture to evaluate cytology
C) Nerve conduction studies to evaluate trigeminal (V) nerve divisions
D) Referral to ophthalmology for complete eye examination
E) Referral to orthopedic surgery for bone age studies

A

The correct response is Option D.

Workup of Parry-Romberg syndrome requires multidisciplinary evaluation in order to provide the patient with the best care. Evaluation by an ophthalmologist is required to determine if the patient has any ocular involvement, as these patients can suffer visual impairment if they have unrecognized disease. Because many patients can have ocular involvement and still be asymptomatic, it is important that they undergo a full ocular examination to fully understand the extent of disease.

While a complete blood count is used liberally for the workup of new health problems, it is not specific or helpful in the diagnosis or management of Parry-Romberg syndrome.

Lumbar puncture may be helpful in patients who have Parry-Romberg syndrome, but only if they have seizures.

Consultation with orthopedic surgery would not be helpful in this patient, and is not a common part of management of patients with Parry-Romberg syndrome. Because this disease is believed to have an autoimmune etiology, referral to a rheumatologist would be far more appropriate.

Nerve conduction studies are not a standard part of a workup for Parry-Romberg syndrome because they would not furnish clinically useful information.

How well did you know this?
1
Not at all
2
3
4
5
Perfectly
12
Q

An 18-year-old woman presents to the office with complaints of worsening facial asymmetry, which began when she was 12 years of age. Physical examination shows significant atrophy of soft tissues of the left side of her face. The plastic surgeon elects to perform reconstruction of the defect with a parascapular flap. Which of the following is the most likely short-term complication for this patient?

A) Cellulitis
B) Facial skin slough
C) Flap loss
D) Hematoma

A

The correct response is Option D.

This patient has hemifacial atrophy, or Parry-Romberg syndrome. Parry-Romberg syndrome is a progressive hemifacial atrophy that can be associated with ocular and neurologic symptoms. The treatment of Parry-Romberg syndrome varies depending on the degree of bony and soft-tissue defect. Fat grafting, alloplastic implants, and free tissue transfer have all been described for correction of the deformity. For larger soft-tissue defects, free tissue transfer may be required. In a study of 177 free flaps, the most common short-term complication associated with free tissue transfer for Parry-Romberg syndrome is hematoma, which occurred in 7% of patients. Flap loss (partial or complete), cellulitis, and facial skin slough were noted in less than 1% of the patients. Overcorrection and the need for debulking/refinement is an expected outcome for this procedure.

How well did you know this?
1
Not at all
2
3
4
5
Perfectly
13
Q

A 6-year-old boy presents with a right-sided craniofacial cleft involving the medial third of the upper eyelid with canthal involvement and orbital malposition. Which of the following best represents the Tessier cleft classification for this patient?

A) No. 3
B) No. 5
C) No. 11
D) No. 13

A

The correct response is Option C.

The number 11 cleft is the cranial extension of the number 3 facial cleft. The cleft originates in the medial orbit and often involves the upper lid as a coloboma or a blepharon. The cleft may also extend into the eyebrow and up to the frontal hairline with a projection of hair extending into the medial third of the forehead.

The number 13 cleft is the paramedial extension of the number 1 facial cleft. The eyelid, eyebrow, and canthal structures are intact in the number 13 cleft, but the medial brow may be displaced inferiorly. There may be V-shaped paramedial projection of the frontal hairline as well as a frontal encephalocele. The skeletal abnormality of the cleft occurs between the nasal bone and the frontal process of the maxilla, causing hypertelorism and widening of the ethmoids, as well as frontal soft tissue and bony abnormalities.

The number 5 cleft begins medial to the lateral oral commissure, proceeding cephalad toward the middle third of the lower eyelid. There is frequently a soft tissue deficiency between the mouth and the lower lid, increasing the risk for corneal exposure. The skeletal defect of the cleft originates in the alveolus, lateral to the canine, and extends through the maxillary sinus to the inferolateral orbit, lateral to the infraorbital foramen.

The number 3 Tessier cleft is often referred to as the oro-naso-ocular cleft. When isolated to the lip, the number 3 cleft can mimic a common cleft lip, beginning at Cupid’s bow peak and extending to the lateral nasal sill. The cleft can extend through the lateral ala to the lower eyelid between the medial canthus and the inferior punctum. A significant soft tissue deficit of the midface may be present with inferior displacement of the lower lid and globe, as well as microphthalmia of the globe. The skeletal defect of the number 3 cleft originates in the alveolus, between the lateral incisor and canine, and extends cephalad to the lateral piriform aperture and medial orbit, terminating in the lacrimal groove of the inferior medial orbit.

How well did you know this?
1
Not at all
2
3
4
5
Perfectly
14
Q

During orbital dissection in preparation for orbital box osteotomy, which of the following anatomical landmarks serves as the starting point for lateral wall osteotomy?

A) Anterior ethmoid foramen
B) Dacryon
C) Inferior orbital fissure
D) Optic foramen
E) Superior orbital fissure

A

The correct response is Option C.

The inferior orbital fissure serves as the starting and ending point of the orbital box osteotomy since only temporal fat is within this fissure. No critical nerves or blood vessels are in this fissure more peripherally. Major structures, such as the infraorbital nerve, travel for a short distance across the apex of the infraorbital fissure as it exits the foramen rotundum and enters the infraorbital canal. Dacryon marks the posterior border of the lacrimal sac. The optic nerve is within the optic foramen, and cranial nerves III, IV, and VI travel in the superior orbital fissure.

How well did you know this?
1
Not at all
2
3
4
5
Perfectly
15
Q

A 2-day-old female newborn is evaluated in the neonatal intensive care unit. Physical examination shows epicanthal folds, a flat nasal bridge, and a wide U-shaped cleft palate. She has micrognathia with 12 mm of overjet, substernal and costal retractions and desaturations while supine, and is not able to feed orally. Ophthalmologic evaluation shows bilateral cataracts. Which of the following metabolic abnormalities is most likely responsible for this patient’s condition?

A) Abnormal cellular response to fibroblast growth factor
B) Abnormal formation of collagen II
C) Abnormal regulation of craniofacial bone and suture embryogenesis
D) Dysregulation of embryogenesis caused by a multi-gene deletion on chromosome 22
E) Inadequate production of surfactant by the lungs

A

The correct response is Option B.

The neonatal patient described in the vignette has clear signs of respiratory distress in the setting of micrognathia and glossoptosis, also known as Pierre Robin sequence (PRS). PRS can be isolated or associated with a broad metabolic disturbance, the most common of which is Stickler syndrome. Stickler syndrome is a disruption of the formation of collagen, which can lead to multiple congenital anomalies including a flat nasal bridge, hearing loss, cleft palate, limb anomalies, micrognathia, and ophthalmologic issues including cataracts.

Mutations in the fibroblast growth factor receptor can lead to syndromic forms of craniosynostosis such as Apert, Crouzon, or Pfeiffer syndromes. Disruption of suture embryogenesis is caused by a mutation in the TWIST1 gene and gives rise to Saethre-Chotzen syndrome. Deletion of the small arm of chromosome 22 leads to 22q11.2 deletion syndrome, previously known as DiGeorge syndrome or Velocardiofacial syndrome. Lack of lung surfactant is most often caused by prematurity and is not associated with PRS.

How well did you know this?
1
Not at all
2
3
4
5
Perfectly
16
Q

A 5-year-old girl has craniosynostosis, a low-set hairline, ptosis, and 4th/5th-digit syndactyly of both hands. Genetic testing is most likely to show a mutation in which of the following genes?

A) EFNB1
B) FGFR2
C) FGFR3
D) MPDZ1
E) TWIST1

A

The correct response is Option E.

The clinical picture is consistent with Saethre-Chotzen syndrome. It is an autosomal dominant condition defined by a genetic mutation or deletion affecting the TWIST1 gene or chromosome 7p21. FGFR2 mutations are predominantly associated with Apert, Crouzon, and Pfeiffer syndromes. FGFR3 mutations are associated with Muenke syndrome and Crouzon syndrome with acanthosis nigricans. EFNB1 is associated with craniofrontonasal syndrome. MPDZ1 is associated with hydrocephalus.

How well did you know this?
1
Not at all
2
3
4
5
Perfectly
17
Q

A 2-year-old boy presents with swelling over the bridge of the nose that has been present since birth. The swelling has been slowly increasing in size and he has hypertelorism. The swelling is soft, compressible, and it transilluminates. There are visible and palpable pulsations, and the mass enlarges when the patient cries. Which of the following is the most likely diagnosis?

A) Encephalocele
B) Glioma
C) Hemangioma
D) Nasal dermoid cyst
E) Nasopharyngeal angiofibroma

A

The correct response is Option A.

Encephaloceles are neural tube defects that result in sac-like protrusions of the meninges (meningocele) or brain and meninges (meningoencephalocele) in various locations along the cranium, such as between the forehead and nose (including naso-orbital, frontonasal, and nasoethmoidal locations) or on the back of the skull. They tend to be soft, compressible masses that transilluminate that may be sessile or pedunculated. Biopsy may result in a cerebrospinal fluid leak.

Glioma is a mass of ectopic neural tissue that does not transilluminate.

Hemangiomas are benign vascular lesions that are present at birth and characterized by a rapid growth phase around the age of 1 to 6 months, followed by gradual involution over 1 to 12 years. They have no intracranial connection and no cerebral pulsations. Nasopharyngeal angiofibromas, also known as juvenile nasopharyngeal angiofibromas, are benign but locally invasive vascular tumors that occur almost exclusively in adolescent males. They present with unilateral or bilateral nasal obstruction, frequent epistaxis or blood-tinged nasal discharge. Nasal dermoid cyst is a benign cystic lesion that does not pulsate and does not transilluminate.

How well did you know this?
1
Not at all
2
3
4
5
Perfectly
18
Q

A 23-year-old woman with severe progressive hemifacial atrophy that has been stable for 3 years now desires a long-term stable reconstruction. Which of the following is the most appropriate recommendation for reconstruction of this patient’s facial asymmetry?

A) Alloplastic bony augmentation
B) Contralateral suction lipectomy
C) Delay reconstruction until it has been stable for 10 years
D) Free tissue transfer
E) Hyaluronic acid injections

A

The correct response is Option D.

Progressive hemifacial atrophy is also known as Parry-Romberg syndrome. The progression is ultimately self-limiting. Reconstruction 2 years or more after burn out is commonly accepted. For very mild asymmetry, hyaluronic acid fillers can improve symmetry, but require recurrent treatments. For mild to moderate asymmetry, microfat grafting can restore symmetry. Multiple sessions may be required to achieve long-term correction. For severe asymmetry, free muscle flap with parascapular flap or anterolateral thigh flap can provide enough soft tissue bulk for long-term correction.

Alloplastic bony augmentation would correct any potential bony deficiencies, but would not address any soft tissue deficiencies.

How well did you know this?
1
Not at all
2
3
4
5
Perfectly
19
Q

A 15-year-old girl has a 12-month history of pain and fullness in the right supraorbital rim. She has café-au-lait spots, a history of precocious puberty, and a recent pathologic rib fracture. Which of the following is the most likely pathology of the lesion?

A) Dermoid cyst
B) Fibrous dysplasia
C) Neurofibroma
D) Osteoblastoma
E) Rhabdomyosarcoma

A

The correct response is Option B.

This patient has a classic presentation of McCune-Albright syndrome. Patients with McCune-Albright present with a triad of polyostotic fibrous dysplasia, precocious puberty, and skin pigmentation (eg, café au lait spots). Additionally, they may have hyperfunctioning endocrinopathies such as growth hormone excess. If these patients present with intramuscular myxomas, it is known as Mazabraud syndrome. Malignant degeneration of fibrous dysplasia has been reported in up to 4% of patients with McCune-Albright syndrome. Management depends on the clinical presentation and functional impact of the lesions, and is primarily surgical.

Patients with neurofibroma may have café au lait spots, but not precocious puberty or pathologic fractures. Dermoid cysts generally do not present with pain and are often noted at a much younger age than the patient described.

How well did you know this?
1
Not at all
2
3
4
5
Perfectly
20
Q

A 14-year-old girl with Crouzon syndrome presents with a severe Angle Class III malocclusion, mid face retrusion, and severe sleep apnea. She is scheduled to undergo Le Fort III advancement using distraction osteogenesis. The risk for complications with this procedure is closest to which of the following?

A) 5%
B) 20%
C) 40%
D) 60%
E) 80%

A

The correct response is Option B.

There are several important advantages of distraction osteogenesis for Le Fort III advancement versus conventional single-stage advancement with bone grafting and these include: less regression, greater advancement distance, and no need for bone grafting. Le Fort III distraction is not without its issues. Major and minor complications have been reported in nearly 20% of patients undergoing this procedure; these complications include bone loss, pin migration, loss of fixation, meningitis, seizures, and cerebrospinal fluid leaks. Several recent reports show that these complications occur in approximately 20% of cases.

How well did you know this?
1
Not at all
2
3
4
5
Perfectly
21
Q

Which of the following genetic mutations is most likely to be found in a patient with orofacial clefting and popliteal pterygium?

A) Gain-of-function mutation in FGFR2
B) Gain-of-function mutation in NF1
C) Gain-of-function mutation in PIK3CA
D) Loss-of-function mutation in IRF6
E) Loss-of-function mutation in TCOF1

A

The correct response is Option D.

IRF6 mutations that result in loss-of-function have been reported in both syndromic Van der Woude syndrome as well as nonsyndromic orofacial clefting; Van der Woude syndrome can include popliteal pterygium.

PIK3CA gain-of-function mutations have been reported with venous malformations and lymphatic malformations. Mutations in TCOF1 are associated with Treacher Collins syndrome; these patients may have cleft palate, but they do not have lip pits. Gain-of-function mutations in FGFR2 have been implicated in syndromic craniosynostosis (Apert syndrome, Crouzon syndrome, and others). Mutations in NF1 result in increased RAS/MAPK signaling and neurofibromatosis type 1.

How well did you know this?
1
Not at all
2
3
4
5
Perfectly
22
Q

A 2-month-old infant is referred for evaluation because he has an abnormal head shape. Physical examination shows low-set ears; short, webbed fingers; and duplicate great toes. A CT scan shows sagittal and lambdoid synostosis. A mutation in which of the following genes is most likely responsible for these findings?

A) FGFR1
B) FGFR2
C) FGFR3
D) RAB23
E) TWIST1

A

The correct response is Option D.

This child has Carpenter syndrome. This syndrome is caused by a mutation in the RAB23 gene, which is located on chromosome 6. Carpenter syndrome is inherited in an autosomal recessive manner, but it can also be caused by de novo mutation in RAB23. In addition to synostosis, symbrachydactyly and preaxial polydactyly are found in patients with Carpenter syndrome.

Mutations in the other genes listed are all associated with syndromic craniosynostoses. Fibroblast growth factor receptor (FGFR) mutations have been associated with several differing syndromes: FGFR1 mutations cause Pfeiffer syndrome, FGFR2 mutations cause Apert and Crouzon syndromes, and FGFR3 mutations cause Muenke syndrome. A mutation of the TWIST1 gene causes Saethre-Chotzen syndrome.

How well did you know this?
1
Not at all
2
3
4
5
Perfectly
23
Q

A 6-month-old female infant presents with a wide, tall forehead, low-set ears, and supraorbital rim retrusion. CT scan demonstrates bicoronal synostosis. Genetic testing is positive for TWIST mutation. Which of the following additional findings is/are characteristic of this patient’s disorder?

A) Cervical spine anomalies
B) Complete cartilaginous tracheal rings
C) Eyelid ptosis
D) Gastroschisis
E) Thumb duplication

A

The correct response is Option C.

The patient described has Saethre-Chotzen syndrome as confirmed by bilateral coronal synostosis, low-set ears, and mutations of the TWIST gene. In addition to these findings, patients with Saethre-Chotzen syndrome often have eyelid ptosis, which is a distinguishing feature from other forms of syndromic craniosynostosis. It is important to recognize ptosis in infants and young children in order to maintain adequate visual pathway development. Thumb duplication is not found in patients with Saethre-Chotzen syndrome. Tracheal anomalies are associated with Pfeiffer syndrome. Cervical spine anomalies can be found in both Apert and Pfeiffer syndromes. Gastroschisis is not associated with syndromic craniosynostosis.

How well did you know this?
1
Not at all
2
3
4
5
Perfectly
24
Q

A 4-year-old boy is referred to the clinic for evaluation. Medical history includes a cardiac defect requiring surgery, submucous cleft palate, hypernasality, and developmental delay. His parents report that he undergoes cardiology follow-up annually. Further testing is most likely to detect an abnormality that will require consultation with which of the following specialties?

A) Endocrinology
B) Gastroenterology
C) Immunology
D) Nephrology
E) Psychiatry

A

The correct response is Option C.

The patient described has 22q11.2 deletion syndrome (formerly known as velocardiofacial syndrome or DiGeorge syndrome). This syndrome is the most common chromosomal deletion error in fetuses, with a prevalence of 1 in 3000 to 6000 live births. 22q11.2 Deletion syndrome is a common cause of hypernasality. Children with congenital heart defects and hypernasality should be worked up for 22q11.2 deletion syndrome. Either a FISH probe or microarray can detect the chromosomal deletion that occurs in the LCR22A–LCR22D region of the chromosome.

Children with 22q11.2 deletion syndrome can present with a myriad of clinical manifestations. The most common clinical manifestation is a congenital cardiac defect, particularly of the outflow tracts (e.g., tetralogy of Fallot). Congenital cardiac disease remains the primary cause of mortality in this patient population.

Hypernasality is another common finding within this patient population, occurring in approximately 65% of patients with 22q11.2 deletion syndrome. Classic workup for this involves imaging of the velopharyngeal mechanism (either nasopharyngoscopy or video fluoroscopy) and imaging of the posterior pharyngeal pharynx with MRI and evaluation of aberrant/medialization of the internal carotid arteries.

Immunologic abnormalities are the most common of the group. A referral to immunology should be initiated in all patients with 22q11.2 deletion syndrome since up to 75% of this patient population can have thymic hypoplasia and diminished T cell production. Children with 22q11.2 deletion syndrome can have chronic infections and poor responses to vaccinations.

Gastrointestinal conditions such as poor feeding, gastroesophageal reflux, and vomiting or constipation occur in approximately 30% of patients with 22q11.2 deletion syndrome. More concerning GI conditions such as malrotation or tracheoesophageal fistula have been found in patients with this syndrome.

Hypocalcemia secondary to hypoparathyroidism can present as tetany, seizures, or feeding issues. Hypocalcemia presents in approximately 50 to 65% of patients with 22q11.2 deletion syndrome. Thyroid function can also be abnormal with hypothyroidism possible.

Nephrology consultation should be considered if abdominal ultrasonography shows renal agenesis, duplication of the collecting system, or cystic kidney disease. About 33% of patients with 22q11.2 deletion syndrome have some abnormality related to the genitourinary system.

Patients with 22q11.2 deletion syndrome are at increased risk for psychiatric disorders such as attention-deficit/hyperactivity disorder (ADHD), autism spectrum disorder, and anxiety (particularly when they progress into adulthood). The rate of schizophrenia is increased in this patient population compared with unaffected individuals.

How well did you know this?
1
Not at all
2
3
4
5
Perfectly
25
Q

Patients with which of the following conditions have the highest incidence of accessory auricle?

A) Apert syndrome
B) Goldenhar syndrome
C) Treacher Collins syndrome
D) VACTERL association
E) Van der Woude syndrome

A

The correct response is Option B.

In a recent review of the literature about congenital accessory auricle, patients with Goldenhar syndrome were found to have an accessory auricle reported 100% of the time.

Of the options listed, VACTERL (Vertebral, Anal, Cardiac, Tracheal, Esophageal, Renal, and Limb association) has the lowest association with an accessory auricle at 16%. Patients with VACTERL typically are found to have vertebral defects, anal atresia, cardiac anomalies, tracheoesophageal fistula, renal anomalies, and limb differences.

Patients with Treacher Collins syndrome are found to have an accessory auricle present 30 to 40% of the time. These patients often have mandibular hypoplasia, zygomatic hypoplasia, external ear anomalies, conductive hearing loss, eyelid colobomas, cleft palate, and dental anomalies. It is associated with several different genes, including TCOF1, POLR1C, and POLR1D.

Patients with Apert syndrome and Van Der Woude syndromes do not typically have accessory auricles.

How well did you know this?
1
Not at all
2
3
4
5
Perfectly
26
Q

A 6-year-old girl is referred for velopharyngeal dysfunction and on physical examination is found to have a submucous cleft and bifid uvula. Which of the following additional clinical findings is most likely to support the diagnosis of 22q11.2 deletion syndrome?

A) Hypocalcemia
B) Hypothyroidism
C) Microtia
D) Overjet
E) Retinal detachment

A

The correct response is Option A.

22q11.2 Deletion syndrome (also referred to as DiGeorge syndrome, velocardiofacial syndrome, and CATCH-22) is a congenital disorder caused by the deletion of a segment of chromosome 22. Symptoms of this syndrome often include anomalous carotid arteries and conotruncal cardiac abnormalities such as truncus arteriosus and tetralogy of Fallot. The prevalence of atrial septal defect (ASD) has been reported to be 12% in patients with velocardiofacial syndrome. Other symptoms and findings include absent or hypoplastic thymus, developmental delay, cleft palate, and hypocalcemia related to hypoparathyroidism. Patients with velocardiofacial syndrome also have abnormal facial features including a broad nasal root, low-set ears, retrognathia, elongated face or maxillary excess, and epicanthal folds. Overjet and retinal detachment are not associated with velocardiofacial syndrome. Hypothyroidism is not as common as hypocalcemia in this patient population.

How well did you know this?
1
Not at all
2
3
4
5
Perfectly
27
Q

An 18-month-old child is brought to the office after undergoing fronto-orbital advancement for metopic craniosynostosis. Which of the following is the earliest age the surgeon can order the x-ray studies and expect to be able to see frontal sinus development?

A) 1 year
B) 2 years
C) 4 years
D) 6 years
E) 10 years

A

The correct response is Option D.

Frontal sinus development is associated with specific age-related periods of growth of the skull. The frontal sinus is absent at birth and during the initial phase of growth of the skull. The sinus is visible only in x-ray studies at the end of the first period of skull growth. This is the time when the endocranial table of the skull ceases to grow and conforms to the general shape of the brain. This is not seen on x-ray studies until 6 years of age or 72 months.

How well did you know this?
1
Not at all
2
3
4
5
Perfectly
28
Q

A female infant is born with severe Treacher Collins syndrome and bilateral Pruzansky III mandible (absence of condyle). Tracheostomy is performed for respiratory distress. Which of the following surgeries is most likely to allow decannulation?

A) Alloplastic condylar reconstruction
B) Bilateral sagittal split osteotomy
C) Costochondral rib grafts
D) Mandibular distraction
E) Tongue-lip adhesion

A

The correct response is Option C.

Patients with Treacher Collins syndrome may have a varied presentation. The mandible hypoplasia may be mild or severe. In this case the patient has no temporomandibular fossa or condyles. Tongue-lip adhesion and mandibular distraction are used in severe Pierre Robin sequence. Because of the absence of condyles and temporal mandibular joints, distraction, and sagittal split osteotomy are not the best options. A costochondral graft will provide better airway support and can also be distracted in the future.

How well did you know this?
1
Not at all
2
3
4
5
Perfectly
29
Q

Which of the following statements is correct about Tessier clefts No. 3, No. 4, and No. 5?

A) Tessier No. 3 involves the alveolar ridge, while Tessier No. 5 does not
B) Tessier No. 3 is medial to the infraorbital nerve, while Tessier No. 4 is lateral
C) Tessier No. 3 only affects the oral region, while Tessier No. 4 only affects the orbital region
D) Tessier No. 4 involves the piriform aperture, while Tessier No. 5 does not
E) Tessier No. 4 is medial to the infraorbital nerve, while Tessier No. 5 is lateral

A

The correct response is Option E.

Tessier No. 3 and No. 4 are medial to the infraorbital nerve, but Tessier No. 5 is lateral.

Tessier No. 3 involves clefts of the nose, orbit, and lip (naso-oral-ocular cleft), whereas Tessier No. 4 involves the lip and orbit (oral-ocular cleft), and the nose is uninvolved.

Tessier No. 5 involves oral, cheek (maxillary sinus), and orbital cleft and is the rarest.

How well did you know this?
1
Not at all
2
3
4
5
Perfectly
30
Q

Which of the following is the most common cause of scaphocephaly without craniosynostosis?

A) Fibromatosis colli
B) Hereditary dolichocephaly
C) Ocular torticollis
D) Prematurity
E) Ventriculoperitoneal shunt

A

The correct response is Option D.

Positional plagiocephaly is very common since the American Academy of Pediatrics began the ‘back to sleep’ campaign to decrease sudden infant death syndrome in 1992. Occipital flattening is the most common type seen. Positional plagiocephaly risk factors include prematurity, intrauterine position, congenital muscular torticollis, plural birth, motor delays, and hypotonia. Ventriculoperitoneal (VP) shunts may induce a secondary craniosynostosis if over-shunting occurs. Scaphocephaly head shape without craniosynostosis is noted in the premature babies in the NICU due to positioning and tone. This corrects with growth, increased tone, and repositioning. Presence of a VP shunt does not lead to the head shape of scaphocephaly without craniosynostosis but may be seen in children after intraventricular hemorrhage in the NICU. Scaphocephaly without craniosynostosis does occur in older infants and children but is not known to be a hereditary condition. Both ocular torticollis and congenital muscular torticollis can cause a head tilt and are associated with deformational plagiocephaly. Fibromatosis colli is seen with severe muscular torticollis and is associated with plagiocephaly.

How well did you know this?
1
Not at all
2
3
4
5
Perfectly
31
Q

A 3-year-old boy is brought for evaluation of a right frontotemporal cranial defect. A photograph is shown. Which of the following materials is most appropriate for reconstruction?

A) Autologous bone
B) Bone morphogenetic protein (BMP)
C) Calcium phosphate paste
D) Porous polyethylene
E) Titanium mesh

A

The correct response is Option A.

Autologous bone is the most appropriate material for inlay cranioplasty for a child this age. Each of the other options has been used in the pediatric population, but they carry important limitations. Porous polyethylene can be used to reconstruct large cranial defects, but is expensive to fabricate, has the potential to become unstable in a growing cranium, and has a higher overall infection rate than autologous bone. Bone morphogenetic protein (BMP) has shown promise in animal models for inducing bone formation in large cranial defects, but it is expensive for a defect of this size; it is a powerful mitogen and can result in dural ossification and, potentially, induction of tumor growth. Calcium phosphate pastes also have been advocated but are very expensive and for a defect of this size, the long-term potential osseointegration is dubious. Furthermore, the risk of infection and/or implant extrusion is higher. Titanium mesh may be appropriate for some children with limited life expectancy or functional needs, but these implants can be bent, offer limited impact resistance, and have a higher overall rate of complications than autologous bone.

How well did you know this?
1
Not at all
2
3
4
5
Perfectly
32
Q

A newborn male is evaluated because of asymmetry of the orbit and eyelid. Physical examination shows a dozen 6- to 10-mm tan macules, axillary freckles, pigmented nodules on the iris, and soft, fleshy, raised, soft-tissue lesions on the forehead and scalp. Which of the following is the most likely diagnosis?

A) Goldenhar syndrome
B) Kabuki syndrome
C) Muenke syndrome
D) Neurofibromatosis
E) Nevus sebaceous

A

The correct response is Option D.

The patient described meets the criteria for neurofibromatosis 1 by clinical diagnosis. Genetic testing is also possible. To make a clinical diagnosis, two or more of the following criteria are required:

Six or more café au lait spots 5 mm or larger (prepubertal) or 15 mm or larger (post-pubertal)

Family history

Two or more neurofibromas

Freckling in armpits or groin

Lisch nodules

Bony dysplasia (sphenoid wing in this case)

Tumor on optic nerve

A patient with Goldenhar syndrome has facial asymmetry, but does not have café au lait spots. Colobomas are common in patients with this syndrome, but Lisch nodules are not.

Muenke syndrome is associated with craniosynostosis. Kabuki syndrome has distinctive associated facial features with long palpebral fissures, everted lower lids, broad nasal tip, arched brows, protruding ears, intellectual disability, and often microcephaly.

Nevus sebaceous is a well-circumscribed hamartomatous lesion composed of sebaceous glands. There is a risk for malignant transformation.

How well did you know this?
1
Not at all
2
3
4
5
Perfectly
33
Q

A 5-year-old boy is referred for evaluation of facial asymmetry. His parents report that they have noticed asymmetry since birth and believed it would improve with time, but it has not. The patient has no history of surgery or physical therapy. On physical examination, the physician notes skull asymmetry and facial asymmetry, including the orbits; there is no obvious neck tilt. A CT scan of the head is most likely to show which of the following?

A) Deformational plagiocephaly
B) Lambdoid craniosynostosis
C) Torticollis
D) Unicoronal craniosynostosis
E) Zygomaticotemporal craniosynostosis

A

The correct response is Option A.

Since the early 1990’s when positional deformational cranial deformities were first identified with the “back-to-sleep” campaigns, they have now become the most common etiology of cranial and facial asymmetry. Prior to that time, torticollis was the most common cause. Craniosynostosis is a much less frequent cause and occurs in the following order of decreasing frequency: unicoronal, lambdoid, zygomaticotemporal. Correct identification of most of the causes of this clinical constellation of findings can be made on clinical examination, but most would still get a CT scan to verify the diagnosis and rule out associated anomalies except for torticollis, which is always a clinical diagnosis.

How well did you know this?
1
Not at all
2
3
4
5
Perfectly
34
Q

Which of the following is the protein encoded by the gene mutated in van der Woude syndrome and popliteal pterygium syndrome?

A) Fibroblast growth factor 10 (FGF10)
B) Fibroblast growth factor receptor 2b (FGFR2b)
C) Interferon regulatory factor 6 (IRF6)
D) Membrane-bound signaling molecule jagged 2 (JAG2)
E) Sonic hedgehog (SHH)

A

The correct response is Option C.

The mechanism of palatal shelf formation must be regulated precisely so that vertical palatal shelves are adhesion-incompetent while they are in close contact with other structures, but once they are raised above the tongue, they rapidly acquire adhesion capability if they are not to remain cleft. Control of periderm differentiation by the membrane-bound signalling molecule jagged 2 (JAG2) is important in this process. Another factor central to this process is interferon regulatory factor 6 (IRF6). The gene IRF6, which has a causal association with van der Woude syndrome, is also linked strongly to the isolated form of clefting. This finding has been replicated in many different populations and ethnic groups. Variants of genes linked to syndromic forms of cleft lip with or without cleft palate that have a mendelian mode of inheritance can also produce phenocopies of non-syndromic clefts. This observation suggests that a strategy of choosing variants of genes associated with syndromic forms of cleft lip with or without cleft palate as candidates for investigations into the cause of non-syndromic clefts could be productive. Other examples of mendelian-inherited syndromes and related genes that, if mutated, could result in or modify the expression of cleft lip with or without cleft palate include Kallmann syndrome (FGFR1), ectrodactyly, ectodermal dysplasia, and clefting syndrome (TP63), X-linked clefting and ankyloglossia (TBX22), Gorlin syndrome (PTCH1), and Margarita Island ectodermal dysplasia (PVRL1[heterozygous]). Although discovery of the genetic cause of van der Woude or popliteal pterygium syndromes will have no immediate therapeutic benefit, advantages for diagnosis are instant, and this knowledge will be potentially useful in genetic counseling. If one gene mutation, which can be identified by prenatal diagnosis, causes cleft lip, cleft lip and palate, or cleft palate alone in a proportion of people, identification of individuals at high risk for having children with the same defect will be possible.

How well did you know this?
1
Not at all
2
3
4
5
Perfectly
35
Q

A 25-year-old woman comes to the office because of a 5-year history of stable facial asymmetry. Physical examination shows fullness of the right mandibular angle; no facial tenderness, masses, or lymphadenopathy is noted. Maxillofacial CT scan shows enlargement of the right masseter muscle compared with the left. Which of the following is the most appropriate first-line treatment for this problem?

A) Fabrication of a bite guard to treat bruxism
B) Injection of botulinum toxin type A into the right masseter muscle
C) Open subtotal myectomy of the right masseter muscle
D) Placement of a left mandibular angle prosthesis
E) Referral to oncology to rule out systemic amyloidosis

A

The correct response is Option B.

The patient described has benign or idiopathic masseter hypertrophy. The classic patient comes to the office because of aesthetic complaints but is otherwise asymptomatic. There are numerous treatments, including partial myotomy, mandibular angle reduction, and contralateral augmentation. Of all the options, the best is botulinum toxin type A injections. These injections are minimally invasive and reversible, and they do not require an anesthetic. On the basis of its low–risk profile, it is the best first-line treatment listed. Amyloidosis is a progressive systemic disease that can cause myopathy and painful bilateral masseter hypertrophy. It is unlikely in this stable, unilateral case. Bruxism can cause masseter hypertrophy, which is most often bilateral. A bite guard will prevent tooth damage, but it will not correct hypertrophy.

How well did you know this?
1
Not at all
2
3
4
5
Perfectly
36
Q

A 4-year-old boy is brought to the office for evaluation of the shape of his head. Physical examination shows an asymmetrically shaped head, low frontal hairline, ptosis of the eyelids, and a prominent crus of helix. Which of the following syndromes is most likely affecting this child?

A) Apert
B) Crouzon
C) Muenke
D) Pfeiffer
E) Saethre-Chotzen

A

The correct response is Option E.

Saethre-Chotzen syndrome is an autosomal dominant disorder associated with TWIST1 gene mutation. It causes an asymmetrical brachycephaly, bicoronal or unicoronal synostosis, low frontal hairline, ptosis of the eyelids, and a prominent crus of helix.

Crouzon syndrome is an autosomal dominant disorder associated with an FGFR2 gene mutation. The patient with this syndrome typically comes to the office because of hydrocephalus, elevated intracranial pressure, Chiari I malformation, bicoronal synostosis and brachycephaly, exorbitism, mid-face hypoplasia, anterior open bite, and normal extremities.

Apert syndrome is an autosomal dominant disorder and shows symptoms of elevated intracranial pressure, bicoronal synostoses, turribrachycephaly, enlarged anterior fontanel, bitemporal widening, occipital flattening, anterior open bite, and complex syndactyly of the hands and feet.

Pfeiffer syndrome is also an autosomal dominant disorder associated with FGFR2 mutation and reveals symptoms of hydrocephalus and a high risk of Chiari malformation, turribrachycephaly with bicoronal synostoses, exorbitism, hypertelorism, down-slanting palpebral fissures, an anterior open bite, and broad thumbs and/or broad halluces.

Muenke syndrome is associated with the FGFR3 gene mutation and comes to the office with coronal synostosis, elevated intracranial pressure, sensorineural hearing loss, and abnormal middle phalanges.

How well did you know this?
1
Not at all
2
3
4
5
Perfectly
37
Q

A 9-year-old boy with Crouzon syndrome is brought to the craniofacial clinic because of obstructive sleep apnea. Apnea hypopnea index is 40. He has a negative overjet of 12 mm, moderate exorbitism, and signs of increased intracranial pressure (>25 mm Hg), which was found during an intracranial pressure monitoring trial last month; intercanthal distance is normal (25 mm). Which of the following types of osteotomy is the most appropriate treatment for this patient?

A) Facial bipartition
B) Le Fort I
C) Le Fort III
D) Monobloc
E) Orbital box

A

The correct response is Option D.

Crouzon-related obstructive sleep apnea and increased intracranial pressure are best treated with frontofacial advancement without adjustment of the orbital width, which is best achieved using the monobloc osteotomy. Orbital box osteotomy will not treat increased intracranial pressure or sleep apnea. Le Fort III osteotomy will treat obstructive sleep apnea but will minimally affect increased intracranial pressure. Lastly, facial bipartition osteotomy is not needed in this case because of the normal intercanthal distance and lack of telorbitism. Le Fort I addresses neither the orbits nor the intracranial pressure.

How well did you know this?
1
Not at all
2
3
4
5
Perfectly
38
Q

An otherwise healthy 5-year-old girl has a yellow, cystic mass on the left sclera, a left preauricular branchial cleft remnant, elevation of the left oral commissure, and soft-tissue deficiency of the left face. It is most appropriate to order which of the following tests to assess for significant comorbidities?

A) Echocardiography
B) Fluorescence in situ hybridization of the branchial cleft remnant
C) Funduscopic examination
D) MRI of the brain
E) Plain x-ray studies of the hands

A

The correct response is Option C.

The patient described has Goldenhar syndrome, a severe form of hemifacial microsomia with variable ear anomalies, mandibular hypoplasia leading to occlusal cant and oral commissure asymmetry, and soft-tissue deficiency on the affected side. Ocular findings associated with Goldenhar syndrome are variable and common (50% incidence) and can occur as epibulbar dermoids as described in this patient as well as microphthalmia, eyelid and optic nerve colobomas (interruption of a circular structure of the eye). Colobomas can be asymptomatic (affecting the iris alone), can lead to exposure keratopathy (in the case of eyelid colobomas), or can lead to visual disturbances (optic disc/nerve coloboma), such as visual field deficits or amblyopia. Care must be taken to identify visual disturbances early in order to implement contralateral eye penalization, or patching, to maximize visual development of the affected eye. The other tests target areas that are not affected in patients with Goldenhar syndrome.

How well did you know this?
1
Not at all
2
3
4
5
Perfectly
39
Q

A newborn male is evaluated in the office for cleft of the soft palate. Physical examination shows severe micrognathia with labored breathing, underdevelopment of the mid face, microtia, and coloboma of the lower eyelids. Mutation in which of the following genes most likely caused these findings?

A) EFNB1
B) FGFR2
C) FGFR3
D) TCOF1
E) TWIST1

A

The correct response is Option D.

Treacher Collins syndrome, named after Edward Treacher Collins, the English ophthalmologist who first described this condition in 1900, is characterized by widely varying degrees of micrognathia, underdevelopment and/or clefting of the midfacial bones (e.g., zygoma), microtia or anotia with or without hearing loss or middle ear atresia, eyelid coloboma, and often respiratory compromise. It occurs with an estimated frequency of 1:50,000 live births, and the overwhelming majority of cases are caused by an autosomally inherited mutation in the TCOF1 gene that leads to underproduction of treacle protein, a neural crest precursor.

Mutations in FGFR2, FGFR3, TWIST1, and EFNB genes are commonly associated with syndromic forms of craniosynostosis. Gain-of-function mutations in the fibroblast growth factor receptors (FGFR) can lead to various syndromes that manifest craniosynostosis including: Apert syndrome (FGFR2), Pfeiffer syndrome (FGFR1 and FGFR2), Crouzon syndrome (FGFR2), Jackson–Weiss syndrome (FGFR2), Muenke syndrome (FGFR3), Crouzonodermoskeletal syndrome (FGFR3), and thanatophoric dysplasia (FGFR3). TWIST1 loss-of-function mutations can lead to another craniosynostosis syndrome—Saethre-Chotzen syndrome. EFNB1 mutations are associated with craniofrontonasal syndrome, an X-linked disorder characterized by hypertelorism, craniosynostosis, clefting of the nasal tip, and digital anomalies.

How well did you know this?
1
Not at all
2
3
4
5
Perfectly
40
Q

A 20-year-old woman is evaluated for facial asymmetry due to loss of volume on one side of the face. She has no history of trauma or infection to that side of her face. Physical examination shows normal dental occlusion with normal bony anatomy and no cranial nerve deficits. She has a mild-to-moderate deficiency of soft tissue along the cheek between the zygomatic and mandibular areas of the face. Which of the following is the most appropriate intervention for correction of this patient’s hemifacial atrophy?

A) Adipofascial free flap
B) Autologous fat grafting
C) Distraction osteogenesis
D) Hyaluronic acid filler
E) Silicone injection

A

The correct response is Option B.

Romberg disease, also known as progressive hemifacial atrophy (PHA), is an acquired disease causing severe deformity of the facial contour. The therapeutic goals of surgery for patients with PHA are to minimize psychosocial effects and to correct the appearance and function of involved facial structures. PHA usually presents in the first 20 years of life, after which it is slowly progressive but self-limited. The atrophy affects subcutaneous tissue, fat, muscle, and osteocartilaginous structures, creating a sunken hemiface appearance. Epidermal cutaneous involvement is minimal, but the tongue, gingiva, teeth, and palate may also be involved. For cases with minimal-to-moderate soft-tissue defects, autologous fat grafting can provide correction of the contour deformities. For more severe soft-tissue deficits, additional or other modalities such as adipofascial free flaps can be used. In more severe cases with skeletal and/or dental abnormalities, interventions such as distraction osteogenesis and bone grafting may be necessary. The injection of free silicone is not recommended and has been replaced by the use of autologous fat and other injectable substances. Hyaluronic acid may provide a temporary solution, but it is costly.

How well did you know this?
1
Not at all
2
3
4
5
Perfectly
41
Q

The patient shown in the photograph is most likely to have which of the following comorbidities?

A) Anosmia
B) Craniosynostosis
C) Epiphora
D) Sensorineural hearing loss
E) Velopharyngeal dysfunction

A

The correct response is Option C.

The patient pictured has a rare craniofacial (Tessier No. 4) cleft. Rare craniofacial clefts are characterized by variable soft-tissue and bony involvement. This patient has a cleft lip that begins lateral to the philtral column and courses laterally to the alar margin. Additionally, this patient has soft-tissue deficiency of the left medial malar region, as well as medial lower lid malposition and medial canthal dystopia. Of all of the options given, these findings suggest an increased risk for nasolacrimal duct abnormalities or inferior canalicular system malposition. Velopharyngeal dysfunction is common in patients with cleft lip and palate. Anosmia can be present in midline craniofacial anomalies, such as craniofrontonasal dysplasia. Rare craniofacial (Tessier No. 4) clefts are not associated with sensorineural hearing loss or craniosynostosis.

How well did you know this?
1
Not at all
2
3
4
5
Perfectly
42
Q

While performing a cranioplasty on a 6-year-old girl, the plastic surgeon accidentally drops the bone graft on the floor. Which of the following is the most appropriate next step?

A) Change the wound classification from 1 to 4
B) Decontaminate with triple antibiotic solution lavage and reuse the graft
C) Discard the graft and use an alloplastic material
D) Steam sterilize and reuse the graft
E) Wash with chlorhexidine and reuse the graft

A

The correct response is Option B.

In this circumstance, the surgeon should wash the graft with sterile triple antibiotic solution (0.1% gentamicin, 0.1% clindamycin, 0.05% polymyxin) and consider reusing it. This graft is not easily re-harvestable, and therefore reusing it is a reasonable alternative. Triple antibiotic solution is readily available and nontoxic to the bone graft, unlike iodine and chlorhexidine. Studies have shown that iodine does not have the antimicrobial effectiveness of other antimicrobials. Studies also show that while chlorhexidine is most effective in eradicating the microorganisms from the graft, its detrimental effect on corticocancellous bone prevents its use in this clinical scenario. If the dropped graft were skin, cartilage, or composite tissue, chlorhexidine would be the ideal antimicrobial.

Discarding the graft is not the ideal alternative given the extremely low likelihood of infection if reused after appropriate decontamination. In this clinical scenario, alloplastic materials are contraindicated due to the patient’s age. One should change the wound classification from 1 to 3, not 4. Steam sterilization is not used for such grafts in such an acute setting and this technique would potentially harm the graft.

How well did you know this?
1
Not at all
2
3
4
5
Perfectly
43
Q

A 53-year-old man presents with a calvarial defect following neurosurgical extirpation of a tumor. Polymethylmethacrylate reinforced by wire mesh is used for the cranioplasty. The use of saline irrigation during the curing of the biopolymer serves which of the following purposes?

A) Dilutes toxic byproducts of polymerization
B) Dissipates thermal energy
C) Irrigates subclinical bacterial colonization
D) Provides an aqueous catalyst for polymerization
E) Reduces the dielectric constant of the substrate

A

The correct response is Option B.

Polymethylmethacrylate polymerization is an exergonic or exothermic reaction. The heat generated can cause injury or necrosis of the surrounding soft tissues and bone. Saline irrigation is an effective method of heat dissipation during this process. The toxic byproducts of polymerization include cyanide gas in small amounts that are dissipated by normal operating room ventilation or suction. Water is not a necessary cofactor for the polymerization process. Reduction of bacterial burden is a rationale for irrigation in general but does not play a specific role in this clinical setting. The dielectric constant of the substrate is elevated or unchanged by the irritant and this is unlikely to have an effect on the polymerization process.

How well did you know this?
1
Not at all
2
3
4
5
Perfectly
44
Q

A 16-month-old male infant is brought to the physician because of congenital anomalies of both feet. The dorsal and plantar aspects are depicted in the photographs shown. Which of the following is the most likely diagnosis?

A) Apert syndrome
B) Crouzon syndrome
C) Jackson-Weiss syndrome
D) Pfeiffer syndrome
E) Saethre-Chotzen syndrome

A

The correct response is Option A.

Syndromic craniosynostosis often presents with findings in the hands and feet, sometimes referred to as acrocephalosyndactyly.

Apert syndrome is unique for having bilateral symmetric complex syndactylies involving nearly all digits, of both the hands and feet.

Crouzon syndrome usually has normal hands and feet.

Jackson-Weiss can have foot anomalies, such as short metatarsals.

Pfeiffer syndrome usually has broad thumbs and toes.

Saethre-Chotzen syndrome may have incomplete single syndactylies, but otherwise does not typically have extremity findings.

How well did you know this?
1
Not at all
2
3
4
5
Perfectly
45
Q

A 2-year-old boy is referred for evaluation of an abnormal frontal prominence that his parents report has become more noticeable during the past year. A photograph is shown. He is otherwise healthy and is meeting all developmental milestones. A CT scan obtained at the request of his pediatrician shows closure of the metopic suture with ectocortical thickening; no other abnormalities are noted. Which of the following is the most appropriate next step in management?

A) Endoscopic suturectomy and postoperative helmet therapy
B) Fronto-orbital advancement
C) Spring-mediated frontal distraction
D) Total calvarial remodeling
E) Observation

A

The correct response is Option E.

This patient has a metopic ridge and would not be correctly classified as having the phenotype associated with pathologic metopic closure—trigonocephaly. Consequently, observation is the only correct answer. Thickening of the metopic suture is a normal variant and should not be interpreted as abnormal unless it is accompanied by frontal narrowing and retrusion of the superior-lateral orbital rims. These findings are not present in this patient.

The metopic suture closes normally within the first year of life, so the presence of a fused metopic suture on CT scan in a child this age is not necessarily abnormal. The degree of frontal narrowing required to classify a patient as having pathologic cranial shape is a matter of much debate and is not settled. Some authors point to the importance of additional clinical (hypotelorism, biparietal widening) or radiographic (endocortical thickening or omega sign on CT) findings to secure the diagnosis of metopic craniosynostosis.

The other responses are surgical interventions and should be invoked only in the context of pathologic metopic closure and resultant trigonocephaly. Endoscopic suturectomy and postoperative helmet therapy, spring-mediated frontal distraction, and fronto-orbital advancement are all viable options to correct the frontal narrowing associated with trigonocephaly. Total calvarial remodeling is rarely if ever required to correct trigonocephaly.

How well did you know this?
1
Not at all
2
3
4
5
Perfectly
46
Q

The congenital anomaly shown in the photograph is thought to be caused by which of the following?

A) Failure of fusion of the lateral and maxillary nasal processes
B) Failure of fusion of the maxillary prominence with the medial nasal prominence
C) Failure of fusion of the medial nasal prominence and the lateral nasal prominence
D) Failure of fusion of the medial nasal prominences
E) Failure of the oronasal membrane to rupture

A

The correct response is Option A.

Proboscis lateralis, which is illustrated in the photograph, is thought to be a failure of fusion between the lateral and maxillary nasal processes. A cleft lip is the result of a failure of fusion of the maxillary prominence with the medial nasal prominence. A midline cleft or Tessier Zero cleft is a result of the failure of fusion of the medial nasal prominences. Choanal atresia is a result of a failure of the oronasal membrane to rupture. Finally, a mandibular cleft or Tessier # 30 cleft is a result of the failure of fusion of the mandibular prominences.

How well did you know this?
1
Not at all
2
3
4
5
Perfectly
47
Q

A 12-year-old boy is evaluated for a vertical furrow near the midline of his face from the hairline to the eyebrows. Each of the listed clinical findings can be seen in hemifacial atrophy EXCEPT:

A) Atrophy of the tongue
B) Change in facial sensation
C) Malar hypoplasia
D) Malocclusion
E) Mandible hypoplasia

A

The correct response is Option B.

The relationship between morphea en coup de sabre and Parry-Romberg syndrome is unclear but there is some overlap. Morphea is characterized by the vertical furrow, atrophy of the tongue and upper lip, absent or flattened zygoma, orbital rim, and a hypoplastic maxilla and mandible on the affected side. A lateral open bite may be seen due to the maxillary and mandibular hypoplasia. Sensation, function of muscles of facial expression, and mastication are normal.

How well did you know this?
1
Not at all
2
3
4
5
Perfectly
48
Q

A newborn male with a Tessier No. 3 orofacial cleft is evaluated in the NICU. Which of the following locations is the most common pathway of this cleft through the alveolar ridge?

A) Between the canine and premolar
B) Between the central and lateral incisors
C) Between the central incisors
D) Between the lateral incisor and canine
E) Between the premolar and first molar

A

The correct response is Option D.

Tessier No. 3 orofacial clefts are the most common type. When they affect the alveolus, they typically traverse between the lateral incisor and the canine and extend into the floor of the nose and through the nasolacrimal system and orbital floor, involving the medial canthal region.

The other options are all incorrect, because they are not typically the route seen in the Tessier No. 3 orofacial cleft.

How well did you know this?
1
Not at all
2
3
4
5
Perfectly
49
Q

A 3-month-old male infant is evaluated because of bilateral preauricular pits and small ears. Physical examination is otherwise unremarkable. The patient failed the newborn hearing test; the parents have not repeated the test. They are not very concerned, because the father is also hearing impaired. Which of the following is the most appropriate next step in management?

A) Echocardiography
B) MRI of the brain
C) Renal ultrasound
D) Repeat hearing test
E) Surgical treatment of the preauricular pits

A

The correct response is Option C.

This child most likely has branchio-oto-renal (BOR) syndrome, an autosomal dominant syndrome. Two genetic mutations (EYA1 and SIX1) have been identified. The clinical findings include auricular malformations, preauricular skin pits, hearing loss, branchial fistulae, external auditory canal stenosis, and renal anomalies. Renal ultrasound should be performed to rule out renal anomalies. Family history of hearing loss should also raise suspicion.

Ear anomalies can also be associated with the oculo-auriculo-vertebral spectrum (OAV), within which hemifacial microsomia falls. OAV spectrum may have associated mandibular, facial nerve, and ocular findings (e.g., epibulbar dermoid, coloboma). Other possible associations include congenital heart defects, such as ventricular septal defect or tetralogy of Fallot. This child does not have other craniofacial findings of hemifacial microsomia, thus cardiac echocardiography is not indicated.

Internal cerebral arterial anomalies may be associated with PHACE syndrome (P, posterior fossa; H, hemangioma; A, arterial anomalies; C, cardiac defects; E, eye anomalies). Investigation of cerebral circulation by MRI of the brain would be prompted by a facial hemangioma, which is absent in this child.

While resection of the preauricular pits may be offered, it does not aid in ruling out other findings and there is no urgency to perform surgical resections in this child at 3 months of age.

Repeating a hearing test will not confirm or rule out OAV or BOR syndrome.

How well did you know this?
1
Not at all
2
3
4
5
Perfectly
50
Q

A 12-hour-old male newborn is evaluated in the neonatal intensive care unit. A diagnosis of CHARGE syndrome is suspected. Review of the medical record and physical examination show congenital heart defect, eyelid colobomas, microtia, hypogonadism, and growth retardation. Which of the following additional conditions is/are most likely in this patient?

A) Craniosynostosis
B) Intrahepatic hemangiomas
C) Lower lip pits
D) Orofacial cleft
E) Syndactyly

A

The correct response is Option D.

Cleft lip and/or palate is found in up to 20% of patients with CHARGE (coloboma of the eye, heart defects, atresia of the nasal choanae, retardation of growth and/or development, genital and/or urinary abnormalities, and ear abnormalities and deafness) syndrome. It is the second most common syndrome associated with cleft lip and palate, following van der Woude syndrome. CHARGE syndrome is believed to be caused by a microdeletion or mutation in the CHD7 (chromodomain helicase DNA-binding protein 7) gene. Over the years, there have been other conditions found to be associated with this syndrome, including limb anomalies, renal anomalies, omphalocele, cranial nerve anomalies, and tracheoesophageal fistula.

Syndactyly is not typically associated with CHARGE syndrome.

Intrahepatic hemangiomas are not typically associated with CHARGE syndrome.

Craniosynostosis is not typically associated with CHARGE syndrome.

Lip pits are commonly found in patients with van der Woude syndrome.

How well did you know this?
1
Not at all
2
3
4
5
Perfectly
51
Q

A 6-year-old girl with 22q11.2 deletion syndrome is brought to the office by her parents. The parents report trouble understanding the child when she speaks. An incompetent velopharyngeal port is suspected. Which of the following is the most likely indication of this malfunction?

A) Consonant omission
B) Glottal stop
C) Hyponasality
D) Nasal air emission
E) Sibilant lateralization

A

The correct response is Option D.

Nasal air emission, along with hypernasality and facial grimacing, are all findings that are commonly observed in patients with an incompetent velopharyngeal port or velopharyngeal dysfunction (VPD). Consonant omissions and glottal stops are two common pathologic compensatory speech patterns that such patients exhibit but are not directly caused by VPD. Instead, they are maladaptive compensatory speech patterns often present in patients with VPD. Although some patients with VPD can have hyponasality or sibilant lateralization, these are unrelated to VPD or an attempt to compensate for VPD.

How well did you know this?
1
Not at all
2
3
4
5
Perfectly
52
Q

A 3-month-old female infant is brought to the office for evaluation of head shape. Examination shows brachycephaly and abnormal orbits. Genetic testing is negative for fibroblast growth factor receptor (FGFR) mutation. Which of the following is the most likely diagnosis?

A) Antley-Bixler syndrome
B) Apert syndrome
C) Crouzon syndrome
D) Pfeiffer syndrome
E) Saethre-Chotzen syndrome

A

The correct response is Option E.

Only Saethre-Chotzen syndrome is not associated with a fibroblast growth factor receptor (FGFR) mutation, but it is instead associated with a mutation of the TWIST gene. It is characterized by brachycephaly with maxillary hypoplasia, a prominent ear crus, and syndactyly. Pfeiffer syndrome (characterized by brachycephaly, mild syndactyly, and broad toes and thumbs) is associated with mutations of both FGFR1 and FGFR2. Apert syndrome (characterized by brachycephaly, syndactyly, and cognitive delay) is autosomal dominant and associated with mutations of FGFR2. Crouzon syndrome is also autosomal dominant and characterized by shallow orbits, craniosynostosis, and maxillary hypoplasia. Antley-Bixler syndrome has four phenotypes, some of which are associated with FGFR mutations and characterized by craniosynostosis, choanal atresia, and radiohumeral synostosis.

How well did you know this?
1
Not at all
2
3
4
5
Perfectly
53
Q

A 3-month-old male infant is brought to the office by his parents for evaluation of skull asymmetry that has worsened since birth. Birth history includes prolonged labor that required cesarean delivery. Physical examination shows flattening of the right posterior occiput with ipsilateral forehead bossing. From a superior view, the right ear is 1 cm more anterior than the left ear, and the anterior fontanelle is open without any bulging. The child’s head tilts to the right and has decreased range of motion when looking to the left. The left side of his neck feels tighter and more rigid compared with the right side. Which of the following is the most appropriate initial management of this child?

A) Cerebral palsy evaluation
A) Occupational therapy of the neck
A) Posterior cranial vault expansion
D) Passive molding helmet therapy
E) Sternocleidomastoid muscle release

A

The correct response is Option B.

This child demonstrates the classic presentation of deformation plagiocephaly with his posterior occiput flattening and compensatory ipsilateral forehead bossing with anterior shifting of his ear on the same side. There are many reasons for deformational plagiocephaly, especially with the current practice of “back to sleep.” Treatment of this focuses on removing the pressure on the affected side. His head tilt and decreased motion are consistent with torticollis. Initial treatment is stretching and occupational therapy to restore usage and balance of his neck muscles. Although helmet therapy can help alleviate pressure on this side, it is not addressing the issue. Further studies have demonstrated that deformational plagiocephaly can be treated with positional changes and behavior modification up until 7 to 8 months of age without difference in head asymmetry compared with those children who began helmet therapy at a younger age. There is no difference in children who fail positional changes compared with those who initiate helmet therapy immediately. At this child’s age of 3 months and with obvious torticollis, the most appropriate initial therapy should be focused on resolving his torticollis and giving him a trial of non-helmet therapy.

How well did you know this?
1
Not at all
2
3
4
5
Perfectly
54
Q

A 3-year-old girl with craniofrontonasal dysplasia presents to the craniofacial team because of moderate exorbitism, severe central sleep apnea, and severe hypertelorism (interocular distance of 40 mm). Which of the following is the most appropriate management plan for this patient?

A) Continuous positive airway pressure until 6 or 8 years of age
B) Facial bipartition surgery within the next year
C) Immediate tracheostomy and assisted ventilation
D) Monobloc advancement surgery within the next year
E) Orbital box osteotomy surgery within the next year

A

The correct response is Option C.

Tracheostomy and assisted ventilation is the most appropriate management. The patient described demonstrates a significant number of central and mixed apneas, indicating that the initiation of breathing by central drive is diminished and that tracheostomy with assisted ventilation set at a base ventilation rate, such as synchronized intermittent mandatory ventilation (SIMV), would be optimal to ensure adequate minute ventilation.

Continuous positive airway pressure (CPAP) until 6 to 8 years of age and then a facial advancement is incorrect. CPAP will assist with obstructive apnea, but will not treat central apnea because no baseline ventilation rate is set using CPAP mode.

Hypertelorism surgery within the next year is incorrect because hypertelorism surgery will not treat sleep apnea without facial advancement. Only the wide orbit position would be treated with an orbital box osteotomy or facial bipartition without advancement.

Facial advancement surgery within the next year is incorrect because advancement will correct the obstructive apnea and a portion of the mixed apneas, but will not address the central sleep apnea. Observation with tracheotomy and assisted ventilation will be more prudent until the central apnea rate improves, usually after time with maturation. The patient described will still have persistent central apnea and is at higher risk for relapse of the face due to the age of under 6 years.

How well did you know this?
1
Not at all
2
3
4
5
Perfectly
55
Q

Which of the following arteries is most likely to be injured in a patient undergoing Le Fort I osteotomy?

A) Descending palatine
B) External carotid
C) Greater palatine
D) Internal carotid
E) Internal maxillary

A

The correct response is Option A.

Descending palatine artery is the correct response since it is at greatest risk after pterygoid osteotomy and down fracture. It is easily visible after down fracture and mobilization of the Le Fort I segment. It is typically injured during osteotomy prior to its division into the greater and lesser palatine arteries.

External carotid artery is incorrect because it is within the neck. Its branches ascend in the face and end in the sphenopalatine artery as it enters the pterygoid foramen.

Internal maxillary artery is incorrect because it is the larger artery prior to division into multiple smaller branches including the middle meningeal, sphenopalatine, and descending palatine arteries.

Greater palatine artery is incorrect because the greater and lesser palatine arteries are below the level of the pterygoid osteotomy. The descending palatine is more proximal and is at greater risk for injury.

How well did you know this?
1
Not at all
2
3
4
5
Perfectly
56
Q

A 6-month-old male infant is evaluated for facial asymmetry noted at birth. Physical examination shows symmetry of the upper one-third of the face, but the right cheek is less prominent than the left. The occlusal plane is canted upward to the right, and the chin point is deviated to the right. In addition, the right ear is small and constricted, and an adjacent preauricular soft-tissue appendage is noted. He has a palsy of the right marginal mandibular nerve. He is otherwise healthy. The parents should be informed that the asymmetry was most likely caused by which of the following?

A) Autosomal dominant mutation
B) Disruption of normal developmental sequence
c) Maternal smoking
D) Omega oil ingestion during pregnancy
E) Uterine deformation

A

The correct response is Option B.

This patient has hemifacial microsomia (also referred to as craniofacial microsomia, Goldenhar syndrome, or oculoauriculovertebral spectrum), the second most common congenital facial malformation after cleft lip/palate. It is estimated to occur in 1:5600 to 1:20,000 live births. Unilateral (or less commonly, bilateral) facial hypoplasia is the primary manifestation, and it can involve ocular structures (orbital size, position, globe development, coloboma, epibulbar dermoids), mandibular structures, auricular structures (preauricular tags and sinuses, microtia or anotia, hearing loss), cranial nerve palsies (cranial facial nerve [VII] is most common); and soft-tissue underdevelopment (including facial clefts). In addition, patients can have cardiac, musculoskeletal, central nervous system, and renal manifestations.

The etiology of hemifacial microsomia (HFM) appears to be heterogeneous. Early investigators using animal models of the condition believed that this was the result of an isolated vascular event in the stapedial artery. While this mechanism cannot be completely dismissed and may have a role in some cases, the prevalence of extracranial findings strongly suggests a more sublime pathogenesis: faulty neural crest cell (NCC) migration. Both genetic and environmental factors have been demonstrated to alter or impair NCC activity. Recent studies have implicated genetic factors in the development of this disorder. Nevertheless, the majority of patients with HFM have no family history and less than 2% of cases arise from autosomal dominant inheritance. Certain vasoactive and nonsteroidal anti-inflammatory drug (NSAID) medications can increase the risk of HFM; maternal ingestion of omega oils has not been associated. Maternal smoking can increase the risk of having a child with HFM, but the association is weak and this is not causative in the vast majority of patients. Uterine deformation can lead to facial asymmetry; however, this typically improves with growth and does not lead to the ear and jaw anomalies in the patient described.

How well did you know this?
1
Not at all
2
3
4
5
Perfectly
57
Q

A 30-year-old primigravid woman is referred to the office because ultrasonography showed polyhydramnios at 22 weeks’ gestation. Additionally, the fetus was observed to have macrosomia, omphalocele, macroglossia, and microtia. There is no evidence of nephromegaly or hepatomegaly. A groove between the lobule and antitragus is noted. Chromosomal analysis indicates rearrangements on chromosome 11p15. Which of the following is the most likely diagnosis?

A) Beckwith-Wiedemann syndrome
B) Binder syndrome
C) Diabetic fetopathy
D) Down syndrome
E) van der Woude syndrome

A

The correct response is Option A.

This is a rare overgrowth disorder involving genetic defect of chromosome 11, commonly characterized by the presence of macrosomia, omphalocele, and macroglossia. These symptoms typically present after 22 weeks’ gestation. Additionally, infants with Beckwith-Wiedemann syndrome often display auricular abnormalities, including characteristic ear folds.

While a common symptom of diabetic fetopathy is macrosomia, a diagnosis of diabetic fetopathy would be incorrect. Common symptoms of diabetic fetopathy include congenital anomalies of the internal organs, such as nephromegaly or hepatomegaly; no such symptoms are detected in this fetus.

The absence of cleft lip and/or cleft palate nephromegaly also precludes the diagnosis of van der Woude syndrome. Additionally, the genetic abnormality responsible for van der Woude syndrome involves chromosome 1, not chromosome 11. Likewise, trisomy 21 (Down syndrome) is not the correct diagnosis, as the chromosomal abnormality responsible for this disease involves chromosome 21, not chromosome 11.

Finally, Binder syndrome is a congenital facial malformation most commonly characterized by a flat nose and retrusion of the midface, neither of which are observed in this fetus. Therefore, Beckwith-Wiedemann syndrome is the correct answer.

How well did you know this?
1
Not at all
2
3
4
5
Perfectly
58
Q

A 20-year-old woman comes to the office to discuss reconstruction for hemifacial atrophy. A photograph is shown. The atrophy began when she was 13 years old and stabilized by 16 years of age. Workup suggests mixed connective tissue etiology that falls within the lupus spectrum. Which of the following is the most appropriate intervention to correct this patient’s deformity?

A) Buried anterolateral thigh free flap
B) Custom silicone prosthesis
C) Gracilis free flap
D) Injectable poly-L-lactic acid
E) Pedicled pectoralis major flap

A

The correct response is Option A.

Facial contour deformities can result from a variety of causes, can be unilateral or bilateral, and may range in severity. The most common cause is Romberg disease, progressive hemifacial atrophy. The cause is unknown but may be a localized form of scleroderma. Systemic lupus erythematosus and scleroderma are the most common connective tissue disorders resulting in soft-tissue atrophy with facial involvement.

Systemic lupus erythematosus is an inflammatory connective tissue disorder of unknown cause that predominantly affects women (90%). The clinical course can be variable and involve almost all organ systems. Cutaneous manifestations include soft-tissue atrophy, telangiectasia, and extremity ulcerations. Scleroderma refers to a spectrum of disorders characterized by diffuse fibrosis of the skin, blood vessels, synovia, and vital organs, such as the kidneys. Women are affected four times more frequently than men and the skin is involved in 95% of cases. The pathogenesis is unclear but may include vascular abnormalities (vasculitis), excess collagen deposition, and excess proteoglycan production in the involved tissues.

Surgical options for correction of hemifacial atrophy include alloplastic implants, free dermis/fat grafts, fat injections, and fasciocutaneous free tissue flaps. For mild defects, a custom alloplastic implant composed of silicone or porous polyethylene may be acceptable, but risks such as skin breakdown with implant exposure, scar contracture, and poor long-term patient satisfaction make an alloplastic option less desirable. In the patient described, the large nature of the defect and thin nature of her skin make an alloplastic option a poor choice.

Fat grafting using the techniques described by Coleman is a viable option for smaller defects. The large defect and need for multiple cycles of fat grafting make this a less desirable option in this patient. Various free flaps including omentum, parascapular, groin, and superficial inferior epigastric artery flaps have been described for correction of hemifacial atrophy. In this case, the anterolateral thigh flap is the most appropriate choice because it allows for harvest of a large amount of tissue, including skin, fat, fascia, and muscle.

A gracilis free flap is a great option for facial reanimation, but it would not have enough size and bulk to correct this patient’s deformity. Similarly, a pectoralis major flap would not be acceptable due to lack of bulk and inability to position the flap without tethering because of the pedicle. Additionally, it would be difficult to transpose the muscle high enough for full correction and an unaesthetic bulge would be notable in the neck.

How well did you know this?
1
Not at all
2
3
4
5
Perfectly
59
Q

A 10-month-old male infant has undergone a cranial vault remodeling for sagittal synostosis. The child has no other comorbidities. On the second postoperative day, the patient’s serum sodium concentration decreases from 140 to 129 mEq/L. Laboratory studies show a suppressed plasma aldosterone concentration. The suspected diagnosis is cerebral salt wasting (CSW). An increase in which of the following is most likely to support this diagnosis?

A) Plasma antidiuretic hormone concentration
B) Plasma sodium concentration
C) Plasma volume
D) Serum uric acid concentration
E) Urine output

A

The correct response is Option E.

Cerebral salt wasting (CSW) is a condition that combines renal sodium and fluid loss after intracranial injury with an overall negative sodium balance and volume contraction. It responds to fluid and salt replacement. It contrasts with the syndrome of inappropriate antidiuretic hormone (SIADH), which is part of the differential diagnosis in hyponatremia as follows:

How well did you know this?
1
Not at all
2
3
4
5
Perfectly
60
Q

A 10-year-old boy with a history of fronto-orbital advancement for craniosynostosis as an infant is brought to the office for evaluation. He would like to play football, but his parents are concerned about a persistent calvarial defect. Physical examination shows a 4 × 4-cm area of the right forehead that has no bony coverage; dural pulsations are observed. Which of the following is the most appropriate management?

A) Fabrication of a custom implant
B) Reconstruction with methylmethacrylate
C) Split calvarial bone grafting
D) Split rib grafting
E) Observation and delayed treatment until after completion of growth

A

The correct response is Option C.

This is a growing child with a large calvarial defect in a problematic location. Therefore, the defect needs to be addressed sooner rather than later, due to the potential risk for trauma and the child’s desire to be active. The gold standard for reconstruction of defects of this size and location is split calvarial bone grafting. Rib grafting would work, but would necessitate another donor site and contouring issues. A custom implant or methylmethacrylate would fix the problem in the short term, but would not grow with the child, necessitating additional surgery in the future.

How well did you know this?
1
Not at all
2
3
4
5
Perfectly
61
Q

Which of the following congenital clefts involves the lower eyelid?

A) 0
B) 1
C) 5
D) 8
E) 9

A

The correct response is Option C.

Craniofacial clefts are highly variable and can range from mild forme fruste to severe with involvement of all the layers of the soft tissue and skeletal structures. Tessier described a classification system still in use today based on the bony and soft-tissue landmarks involved in the clefts. Corrective surgery is required in stages, with the timing based on the level of functional problems, including ocular exposure, and airway problems early on. The clefts can be broadly grouped into midline and paramedian clefts (numbers 0 to 14, 1 to 13, 2 to 12), oro-nasal-ocular clefts (numbers 3 to 11, 4 to 10, 5 to 9), and lateral clefts (numbers 6, 7, and 8). In this system, the numbers 0 to 6 refer to clefts below the orbital and numbers 8 to 14 are above the orbit, with 7 being truly lateral.

How well did you know this?
1
Not at all
2
3
4
5
Perfectly
62
Q

A 6-month-old girl is brought to the office for evaluation of an isolated cleft of the soft palate. A photograph is shown. Which of the following genes is most likely defective in this patient?

A) FGFR2

B) IRF6

C) MSX1

D) SHH

E) TGF-beta

A

The correct response is Option B.

The patient has Van der Woude syndrome, an autosomal dominant condition affecting 1:35,000 to 1:100,000 persons. The pathognomonic features include lower lip pits, as seen in the photograph, and clefting of the palate, lip, or both. This syndrome is the most common single-gene cause of cleft lip and palate. IRF6 codes for a transcription factor that is involved in the early development. The mutated copy of the gene decreases the amount of active protein and results in the defects associated with this syndrome and popliteal pterygium syndrome. The remaining genes and their products have been implicated in craniofacial morphogenesis and etiopathogenesis of cleft lip/palate. Nevertheless, none are associated with lip pits or are an autosomal dominant cause of facial clefting.

How well did you know this?
1
Not at all
2
3
4
5
Perfectly
63
Q

A 2-day-old male newborn is evaluated for the skull findings shown in the CT scan. Which of the following best describes the anomaly in this patient?

A) Brachycephaly
B) Kleeblattschädel deformity
C) Posterior plagiocephaly
D) Scaphocephaly
E) Trigonocephaly

A

The correct response is Option B.

This CT scan shows craniosynostosis of multiple sutures including the coronal, lambdoid, and a portion of the sagittal suture, which is characteristic of a Kleeblattschädel deformity. Brachycephaly is characterized by bicoronal craniosynostosis alone and is most commonly seen in syndromic craniosynostosis. Scaphocephaly is isolated involvement of the sagittal suture and is the most common type of craniosynostosis. Metopic synostosis is the cause of trigoncephaly and this suture is open in the CT scan. Lambdoid, which produces posterior plagiocephaly synostosis, is seen in the CT scan but not in isolation and is the least common of the single suture synostoses.

How well did you know this?
1
Not at all
2
3
4
5
Perfectly
64
Q

According to the Tessier cleft classification types, at which of the following areas is the most common facial cleft centered?

A) Frontonasal suture
B) Frontozygomatic suture
C) Nasomaxillary buttress
D) Superior orbit
E) Zygomaticotemporal suture

A

The correct response is Option E.

Patients with rarer facial clefts are classified according to the system proposed by Tessier in 1973. Each cleft is classified by the soft-tissue findings and the bony defects as they relate to the orbit. Nos. 0 through 7 occur in the lower half of the face as delineated by the orbital commissure, and Nos. 9 through 14 occur in the upper half of the face. The most common facial cleft is the No. 7 cleft centered in the region of the zygomaticotemporal suture. It includes variable expression of macrostomia and hypoplasia of the zygoma.

How well did you know this?
1
Not at all
2
3
4
5
Perfectly
65
Q

A 23-year-old man comes to the office because of a progressive 15-year history of severe unilateral volume loss in the face. The patient says he is unhappy with the changes in his facial appearance but has not noted any changes recently. On examination, volume loss is significant and appears to be limited to the soft tissue. No bony deficit is noted. Which of the following is the most appropriate management?

A) Alloplastic bony augmentation of the maxilla
B) Autologous soft-tissue augmentation with a cross-facial nerve graft followed by microvascular gracilis muscle transfer
C) Autologous soft-tissue augmentation with a microvascular parascapular flap
D) Autologous soft-tissue augmentation with a pedicled temporalis muscle flap
E) Hyaluronic acid augmentation of the soft tissues

A

The correct response is Option C.

It is most likely that this patient has progressive, but now stable, hemifacial atrophy, or Parry-Romberg syndrome. The course is characterized by progressive unilateral atrophy of the soft tissues of the face. The underlying skeleton may be affected in severe forms of the disease. Surgery is indicated when the changes stabilize. The recommended treatment is free tissue transfer, preferably a microvascular parascapular flap, followed by a secondary autologous fat grafting for refinement. Augmentation of the maxilla is not indicated in the absence of bony atrophy. Staged cross-facial nerve grafting followed by microvascular gracilis transfer is indicated for absence of facial nerve function. A pedicled temporalis muscle flap would likely not provide sufficient bulk where needed and potentially leave a hollow at the donor site. Alloplastic augmentation of the soft tissues with hyaluronic acid is only approved for volume loss due to human immunodeficiency virus infection.

How well did you know this?
1
Not at all
2
3
4
5
Perfectly
66
Q

A 9-year-old boy is brought to the office because of a long-standing history of pulsatile exophthalmos and proptosis of the right side. CT scan shows absence of the greater sphenoid wing. Which of the following is the most likely diagnosis?

A) Hemifacial microstomia
B) Neurofibromatosis
C) Tessier No. 5 cleft
D) Treacher Collins syndrome
E) Unilateral coronal synostosis

A

The correct response is Option B.

Although each of the disorders or syndromes listed is associated with a periocular soft-tissue or orbital abnormality, only neurofibromatosis is associated with agenesis of the sphenoid wing.

While plastic surgeons primarily manage the soft-tissue manifestations of neurofibromatosis, the disorder is associated with skeletal abnormalities including sphenoid wing aplasia, macrocephaly, scoliosis, and tibial bowing. Sphenoid wing aplasia causes brain herniation through the middle cranial fossa into the orbit with pulsatile exophthalmos, vertical dystopia, and an enlarged orbital volume. Surgical correction involves bone grafts or titanium mesh to reconstruct the sphenoid wing with reduction of the temporal lobe into the middle cranial fossa.

Unilateral coronal synostosis is characterized by foreshortening of the orbital roof on the affected side. Synostosis of the coronal suture causes superior displacement of the lesser wing of the sphenoid with the radiographic appearance of the harlequin deformity. Patients may have strabismus occurring secondary to relative paresis of the superior oblique muscle. The child may tilt the head to compensate. Treacher Collins syndrome is associated with lower eyelid colobomas, zygomatic hypoplasia with hypoplastic or absent zygomas and inferolateral orbital rim clefting, absence of eyelashes, and bilateral mandibular hypoplasia. Hemifacial microstomia is a variant of craniofacial microsomia, distinguished by the presence of concomitant ocular abnormalities, including epibulbar dermoids. The Tessier No. 5 cleft begins behind the canine and extends through the maxillary sinus to the orbital floor. Colobomas of the lateral lower eyelids and clefting of the upper lip medial to the oral commissure are associated.

How well did you know this?
1
Not at all
2
3
4
5
Perfectly
67
Q

A 4-month-old male infant is brought to the office for evaluation of an abnormal head shape that has been present since birth. The patient is developmentally appropriate and has no other abnormalites. A photograph is shown. At which of the following ages does the involved suture normally fuse?

A) 2 weeks
B) 8 months
C) 3 years
D) 10 years
E) 22 years

A

The correct response is Option B.

The patient in the photograph has metopic craniosynostosis. The primary clinical features of trigonocephaly are a palpable metopic ridge, bossing with thickened bone at the glabella, bifrontal narrowing, recession of the superior orbital rims, temporal narrowing, and hypotelorism. Trigonocephaly is also known as a keel-shaped deformity.

The metopic suture is the first cranial suture to fuse and this usually occurs at approximately 8 months of age. The other cranial sutures generally fuse in adulthood.

Treatment depends on the age of presentation and extent of deformity. Options for management are varied and range from no intervention for minimal deformity, burring of the metopic ridge only, endoscopic synostectomy with helmet therapy, and bilateral fronto-orbital advancement.

How well did you know this?
1
Not at all
2
3
4
5
Perfectly
68
Q

A 12-year-old girl is evaluated for left-sided progressive hemifacial atrophy. Onset of symptoms started at 10 years of age. She has a history of migraines and experienced a seizure 1 year ago. Physical examination shows left-sided facial atrophy including cheek depression, enophthalmos, and dry skin with hyperpigmentation and alopecia. Maxillofacial CT scan shows associated skeletal changes. Which of the following clinical features is most likely associated with the early onset of the disease process?

A) Enophthalmos
B) Facial paralysis
C) Hyperpigmentation
D) Seizure disorder
E) Skeletal atrophy

A

The correct response is Option E.

Progressive hemifacial atrophy, also known as Parry-Romberg syndrome, is a rare disorder characterized by a self-limiting, slow, progressive, unilateral facial atrophy affecting the skin, subcutaneous tissue, muscle, and osteocartilaginous adjacent structures. It involves one or more trigeminal dermatomes with symptoms usually appearing in the first or second decade of life. The progression of atrophy usually lasts a variable 2 to 10 years. The etiology and pathogenesis of this disease are uncertain.

The severity of soft-tissue involvement appears to be independent of age of onset, facial distribution, or extent of the disease process; however, age of onset has been shown to correlate with the degree of bony hypoplasia. Onset of the disease at an earlier age during skeletal immaturity has a higher likelihood of skeletal involvement.

Clinical features of hemifacial atrophy include enophthalmos, cheek depression, deviation of the mouth and nose towards the affected side, ipsilateral show of teeth, and tongue atrophy. In addition, extracutaneous involvement has been described, including hemiatrophy of the contralateral or ipsilateral arm, trunk or leg, as well as dental, ocular, and neurologic abnormalities such as enophthalmos, headaches, facial pain, and seizures. An age correlation with these features has not been demonstrated. Facial paresthesia has been described but motor function remains intact.

How well did you know this?
1
Not at all
2
3
4
5
Perfectly
69
Q

A 6-month-old girl is evaluated because of a “flat head.” Physical examination shows right posterior flatness of the cranium with a transcranial difference of 7 mm (minimal to moderate), and mastoid bossing on the ipsilateral side. The right ear and left forehead are anteriorly displaced. The parents report that the child prefers to turn her head to the right. The patient is otherwise healthy and reaching developmental milestones. Which of the following is the most appropriate next step in management?

A) Cranial orthotic molding helmet
B) Crib positioning and physical therapy only
C) CT scan of the head
D) X-ray studies of the skull
E) Reassurance

A

The correct response is Option C.

The clinical findings are suggestive of right unilateral lambdoidal synostosis (synostotic plagiocephaly). If craniosynostosis is suspected, the child may need surgical correction of the defect. Therefore, it is incorrect to pursue reassurance and conservative therapy without confirming the presence of a prematurely fused suture.

Lambdoidal synostosis is the least common premature fusion of a suture (2 to 3% of all synostosis cases). The physical findings have some similarities and distinct differences from the frequently seen positional deformational plagiocephaly. Positional plagiocephaly can have ipsilateral forehead protuberance and anterior displacement of the ipsilateral ear. Lambdoid synostosis, on the other hand, is associated with ipsilateral mastoid bossing and protuberance of the contralateral forehead. The position of the ear, once another landmark to help distinguish between positional and synostotic plagiocephaly, with lambdoid synostosis associated with posterior displacement of the ipsilateral ear, has been questioned in recent years. A literature review showed that the ipsilateral ear has been reported to be nondisplaced, anteriorly displaced, or posteriorly displaced. Therefore, the significance of the position of the ear is unclear at this time.

There is no need for plain film series before head CT scan. Imaging studies are performed to confirm synostosis before surgical intervention.

How well did you know this?
1
Not at all
2
3
4
5
Perfectly
70
Q

An 8-hour-old male newborn is examined in the neonatal intensive care unit for epibulbar dermoids. X-ray studies show fused cervical vertebrae. Which of the following additional features is most consistent with this diagnosis?

A) Cleft palate
B) Lower lip pits
C) Microtia
D) Syndactyly
E) Telecanthus

A

The correct response is Option C.

Oculoauricular dysplasia (Goldenhar syndrome) may have many different possible manifestations, but is typically characterized by ear anomalies, epibulbar dermoids, facial and mandibular hypoplasia, and vertebral anomalies.

Cleft palate, although it may be encountered in patients with Goldenhar syndrome, is not generally regarded as a classic finding and is less common than microtia in affected patients.

Lower lip pits are a finding suggestive of van der Woude syndrome, the most common orofacial clefting syndrome, rather than Goldenhar syndrome.

Syndactyly is not generally regarded as a component of Goldenhar syndrome.

Telecanthus is not a classic finding in patients with Goldenhar syndrome.

How well did you know this?
1
Not at all
2
3
4
5
Perfectly
71
Q

A 4-week-old male newborn is evaluated for complete left unilateral cleft of the lip and palate. Genetics workup shows IRF6gene mutation. Which of the following additional physical findings is most likely in this patient?

A) Agenesis of the corpus callosum
B) Glossoptosis
C) Hypodontia
D) Medialized internal carotid arteries
E) Preauricular accessory tag

A

The correct response is Option C.

The mutation for van der Woude syndrome has been mapped to the interferon regulatory factor 6 (IRF6) gene in chromosome 1. The inheritance is autosomal dominant with variable penetrance. Other associated findings include hypodontia (as high as 86%), high arched palate, lip pits, syngnathia, and ankyloglossia. Agenesis of the corpus callosum and preauricular skin tags are not associated with van der Woude syndrome.

Glossoptosis is associated with Pierre Robin sequence. Medialized internal carotids can be seen in patients with velocardiofacial syndrome.

How well did you know this?
1
Not at all
2
3
4
5
Perfectly
72
Q

The association between craniofacial defects and cardiac malformations in patients with velocardiofacial syndrome results from a disruption in the cellular development of which of the following?

A) Cardiogenic mesoderm
B) Ectodermal placodes
C) Lateral plate mesoderm
D) Neural crest
E) Somites

A

The correct response is Option D.

Neural crest cells derive from the ectoderm layer of the developing embryo, specifically the neuroectoderm of the forebrain, midbrain, and hindbrain. The neural crest contributes significantly to the craniofacial region, and also to the conotruncal endocardial cushions that are responsible for dividing the outflow tract of the heart into separate pulmonary and aortic components. Therefore, defects in neural crest cell development will frequently result in malformations of both the craniofacial area and cardiac septum. Examples of this association are: Treacher Collins syndrome, Pierre Robin sequence, 22q11.2 deletion syndrome, and oculoauriculovertebral syndrome.

The cardiogenic mesoderm derives from splanchnic (visceral) mesoderm, and contributes the precursor cells that differentiate into the endocardium and myocardium. It does not contribute to the head and neck.

Ectodermal placodes are separate from the neuroectoderm and consist of areas of thickened ectoderm that guide neural crest cells in forming the cranial sensory ganglia.

Lateral plate mesoderm is a derivative of the mesoderm layer of the embryo and contributes to the laryngeal cartilages and associated connective tissue of the head and neck.

Paraxial mesoderm produces somites that form the skull, meninges, voluntary craniofacial musculature, and dermis and connective tissue of the dorsal aspect of the head.

How well did you know this?
1
Not at all
2
3
4
5
Perfectly
73
Q

A female newborn is evaluated after an uneventful delivery because of microtia of the left ear. The face appears otherwise symmetrical. Which of the following studies is most appropriate to obtain?

A) CT scan to rule out tethered cord
B) Echocardiography to rule out ventricular septal defect
C) Extremity x-ray studies to rule out limb-length discrepancies
D) Magnetic resonance angiography of the brain to rule out ipsilateral cerebral artery anomalies
E) Ultrasonography of the kidney to screen for structural anomalies

A

The correct response is Option E.

Syndromic ear anomalies are associated with an increased risk of renal anomalies in syndromes such as brachio-oto-renal syndrome, Townes-Brocks syndrome, etc.

Cardiac abnormalities are not associated with isolated microtia, but they are associated with extended spectrum hemifacial microsomia (oculoauriculovertebral dysplasia).

Magnetic resonance angiography of the brain to rule out internal cerebral artery anomalies is indicated in children suspected of PHACE syndrome (P, posterior fossa; H, hemangioma; A, arterial anomalies; C, cardiac defects; E, eye anomalies) when clinical symptoms include a large segmental hemangioma.

Tethered cord is not associated with ear anomalies. It is a concern with myelomeningoceles and spina bifida, and with Chiari malformations.

Limb-length discrepancies are not associated with microtia.

How well did you know this?
1
Not at all
2
3
4
5
Perfectly
74
Q

An 8-month-old female infant is brought to the office by her parents. Physical examination shows a wedge-shaped skull with a keel formation on the forehead, close-set eyes, and hollowness of the temporal fossa on both sides of the head. Premature cranial suture ossification at which of the following sites is the most likely cause of this patient’s condition?

A) Bicoronal
B) Lambdoid
C) Metopic
D) Sagittal
E) Unicoronal

A

The correct response is Option C.

Craniosynostosis refers to the premature fusion of one or more cranial sutures that make up the cranial vault and cranial base. Once this fusion occurs prematurely, the growth of the skull is altered and the development of the head takes on a characteristic morphologic shape that is determined by the fusing suture.

Trigonocephaly is classically characterized by a typically wedge-shaped skull when viewed from above; it originates from a premature stenosis of the metopic suture followed by a bilateral growth restriction of the forehead. This results in bitemporal narrowing and hypotelorism.

Plagiocephaly or unilateral coronal synostosis is characterized by the flattening of the forehead and frontoparietal region ipsilateral to the fused suture. As a result of this fusion, a compensatory bulge occurs in the opposite frontoparietal skull. The temporal fossa on the side of the fusion is convex and the ear becomes anteriorly displaced. The petrous portion of the temporal bone that contains the glenoid fossa is also displaced forward and the articulation with the mandible is displaced forward as a result. The nasal radix is also deviated toward the fused side and the tip of the nose is turned to the opposite side.

Ridging of the sagittal suture forms a narrow biparietal skull. Scaphocephaly shows compensatory growth in the frontal region or frontal bossing and/or occipital coning. There is associated enlargement of the head circumference. Sagittal synostosis remains the most frequent of the nonsyndromic craniosynostosis.

Unilateral lambdoid synostosis has ridging of the lambdoid suture, ipsilateral parieto-occipital flattening, prominence of the mastoid air cells, posterior displacement of the ear on the side of the occipital flattening, and scoliosis of the base of the skull, resulting in curvature of the cervical spine.

How well did you know this?
1
Not at all
2
3
4
5
Perfectly
75
Q

A 12-year-old girl is brought to the office because of an enlarging mass of bone in the maxilla, precocious puberty, and café-au-lait spots. No aesthetic distortion or functional impact from the lesion is noted. She has intermittent bone pain in the upper extremities. Which of the following treatments is most appropriate in this patient?

A) Calcitonin
B) Doxycycline
C) Pamidronate
D) Prednisone
E) Radiation therapy

A

The correct response is Option C.

The patient described has McCune-Albright syndrome. It is a triad of polyostotic fibrous dysplasia, precocious puberty, and café-au-lait spots. Surgical intervention is not indicated in asymptomatic lesions. The general indications for surgery are aesthetic imbalance, facial disfigurement, distortion of functional occlusion, orbital dystopia, ocular proptosis, impingement on neural foramina, impingement on the optic nerve, and intractable pain. It is debatable whether contour reduction or resection and replacement of the afflicted bone is preferable. Recent literature seems to favor the latter. Biopsy is generally not needed for diagnosis, as imaging studies are generally specific for fibrous dysplasia. Equivocal imaging may warrant biopsy to confirm the diagnosis.

In general, medical treatment has had little impact on fibrous dysplasia. Early attempts at treatment included chemotherapy, glucocorticoids, calcitonin, and radiation therapy, which were all unsuccessful.

Radiation therapy should never be used, as it clearly promotes sarcomatous degeneration of fibrous dysplasia.

Pamidronate, a bone resorption-inhibiting bisphosphonate, has been shown in multiple small studies to decrease pain associated with fibrous dysplasia and decrease bone turnover. Nonsteroidal anti-inflammatory drugs can be effective, as can narcotic analgesics in pain control. Referral to a pain specialist may be necessary.

How well did you know this?
1
Not at all
2
3
4
5
Perfectly
76
Q

A 10-month-old male infant with 22q11.2 deletion and preoperative basic metabolic profile within the reference range is scheduled for palatoplasty to treat a soft palate cleft. After surgery, which of the following laboratory studies is most appropriate within the first 6 hours?

A) Fluorescent in situ hybridization
B) Measurement of serum phosphate concentration
C) Measurement of serum calcium concentration
D) Measurement of serum potassium concentration
E) Measurement of serum sodium concentration

A

The correct response is Option C.

Patients with 22q11.2 deletion may have cardiac abnormalities, renal issues, immune deficiencies, speech and feeding delays, mental health issues, developmental delay, cleft palate, and calcium regulation disturbances. Postoperatively, patients with 22q11.2 deletion may be at higher risk than non-deletion patients for hypocalcemia and should have postoperative calcium concentration checked in the first 6 hours postoperatively to identify and correct any abnormalities. Failure to identify postoperative hypocalcemia may lead to increased morbidity and mortality. Fluorescent in situ hybridization is a genetic test that is unnecessary in this patient who already has the diagnosis of 22q11.2 deletion. The sodium, potassium, and phosphate values are not routinely obtained early postoperatively.

77
Q

A 28-year-old man is evaluated because of the facial deformity shown in the photograph. Three years ago, he underwent resection of an infratemporal malignancy and intraoperative alloplastic reconstruction of bony defects. Postoperatively, he underwent extensive radiation therapy. Which of the following is the most appropriate method for restoring facial volume in this patient?

A) Custom-fabricated alloplastic implantation
B) Dermal fat grafting
C) Implantation of layered acellular dermis
D) Parascapular free flap reconstruction
E) Serial fat grafting

A

The correct response is Option D.

The patient described has marked loss of facial soft-tissue volume related to the initial tumor resection and the adverse effects of postoperative radiation treatment. The best method for restoring soft-tissue volume is a scapular free flap. This method of reconstruction has advantages over the others listed. The free scapular flap does not rely on the damaged and scarred soft-tissue envelope for vascular support and, thus, it will retain its volume. In contrast, fat grafting, dermal fat graft, and layered acellular dermis all undergo some resorption, especially in this poorly vascularized recipient site. The scapular flap is of sufficient thickness to correct even a volume defect of this size. Although the other soft-tissue reconstructive options can improve contour, the volume required in this patient cannot be achieved with these modalities alone. The use of an alloplastic reconstruction alone can improve mid-facial volume, but will not address the lower third deficit. In addition, there is a moderate risk of extrusion and/or infection with this technique alone.

78
Q

A 3-year-old boy is brought to the office because of abnormal head shape since birth. Photographs are shown. The patient is developmentally appropriate and has no other medical problems. Which of the following is the most appropriate surgical procedure for correction of this deformity?

A) Bilateral fronto-orbital advancement
B) Endoscopic craniotomy and helmet therapy
C) Monobloc distraction
D) Spring-mediated cranioplasty
E) Total vault reconstruction

A

The correct response is Option E.

The patient in the photograph has the classic features of sagittal suture synostosis. The primary clinical features of scaphocephaly are lengthening of the cranial vault in the anterior-posterior dimension, an anteriorly displaced cranial vertex, bullet-shaped occiput, biparietal and/or temporal narrowing and frontal bossing. All of these features are present in this patient. Because of the patient’s late clinical presentation for treatment, the most appropriate surgical procedure would be a total cranial vault reconstruction of which there are many variations and techniques.

Because of the patient’s age, he is not a candidate for either endoscopic-assisted wide strip craniotomy or spring-mediated cranioplasty. Ideal candidates for either procedure are ideally under age 6 months and 9 months, respectively, for an adequate clinical result. The results of both techniques have been shown to be comparable to traditional open remodeling procedures and are generally less invasive surgical procedures.

Although the monobloc distraction may allow the opportunity to remodel the forehead, there is no clinical indication for midface distraction in the patient described. A monobloc distraction alone will not correct the other abnormal features of scaphocephaly. It is not an indicated procedure for the correction of isolated sagittal suture synostosis.

Bilateral fronto-orbital advancement may allow the opportunity to remodel the forehead, but it will not address the other cranial vault abnormalities. There is no indication for remodeling the supraorbital bar in this case.

79
Q

A newborn in the neonatal intensive care unit is small for gestational age and has choanal atresia, cryptorchidism, a ventricular septal defect, and abnormal external ear framework. Prenatal findings included growth retardation with poor fetal movement. Which of the following additional characteristics is most likely in this newborn?

A) Bicoronal craniosynostosis
B) Coloboma
C) Radial hypoplasia
D) Syndactyly
E) Webbing of the neck

A

The correct response is Option B.

The newborn described has CHARGE (coloboma of the eye, heart defects, atresia of the nasal choanae, retardation of growth and/or development, genital and/or urinary abnormalities, and ear abnormalities and deafness) syndrome. Bicoronal craniosynostosis is associated with Crouzon, Apert, and Pfeiffer syndromes, which do not include all of the other defects. Radial hypoplasia is associated with VACTERL (vertebral defects, anal atresia, cardiac defects, tracheo-esophageal fistula, renal anomalies, and limb anomalies) syndrome. Syndactyly can be seen in many conditions including Down syndrome, Apert syndrome, and Carpenter syndrome. Webbing of the neck can be commonly seen in Noonan syndrome, Klippel-Feil syndrome, and Turner syndrome.

80
Q

A 4-month-old female infant is brought to the office for evaluation. A photograph is shown. Which of the following additional abnormalities are most likely associated with this patient’s condition?

A) Glossoptosis and cleft palate
B) Hydronephrosis of the kidneys and hearing loss
C) Lacrimal duct obstruction and coloboma of the eyelids
D) Posterior fossa abnormalities and stenotic cerebral arteries
E) Supernumerary teeth and duplicate maxilla

A

The correct response is Option E.

The patient described has bilateral macrostomia, also known as Tessier No. 7 cleft, the most common facial cleft in the Tessier classification system. This resulted from the failure of fusion between the maxillary and mandibular processes. Repair of the macrostomia can be undertaken in the first months of life. Duplicated maxilla has been reported in as high as 39% of patients with macrostomia. It is defined as having multiple supernumerary teeth and marked overlap of the maxillary arches. Other craniofacial findings such as mild mandibular/condylar anomalies and alveolar clefting have also been reported. The anatomy can be defined by three-dimensional CT scan and panoramic x-ray study (Panorex). Therefore, it is important to continue to observe these children with dental and orthodontic workups as they grow, even after the repair of the macrostomia.

Glossoptosis and cleft palate are associated with Pierre Robin sequence. Renal anomalies may be associated with congenital anomalies of the ears, such as in branchiootorenal syndrome.

Posterior fossa abnormalities and intracranial arterial anomalies are associated with PHACE syndrome. (P, posterior fossa; H, hemangioma; A, arterial anomalies; C, cardiac defects; E, eye anomalies).

Coloboma of the eyelids and lacrimal gland anomalies are associated with Tessier No. 3 clefts (oro-nasal-ocular clefts).

Macrostomia is most commonly associated with hemifacial microsomia.

81
Q

Which of the following best describes the orbital deformities observed in a patient with Apert syndrome?

A) Enophthalmos
B) Esotropia
C) Exophthalmos
D) Exorbitism
E) Exotropia

A

The correct response is Option D.

Exorbitism describes bulging of the globe due to decreased volume of the bony orbit, whereas exophthalmos describes protrusion of the globe due to increased orbital soft-tissue contents. There is decreased orbital bony volume due to craniosynostosis in Apert syndrome.

Exotropia and esotropia describe strabismus, in which the eyes are deviated laterally (outward) or medially (inward), respectively.

Enophthalmos is a condition where the globe is recessed in the orbit. This can be due to increased bony volume or malposition of the bony orbit after trauma and/or under-correction or repositioning of the bony skeleton.

82
Q

In patients with syndromic craniosynostosis, successful mid facial distraction is most likely to result in which of the following outcomes?

A) Decreased ANB angle
B) Decreased exorbitism
C) Decreased upper airway volume
D) Increased negative overjet
E) Increased SNB angle

A

The correct response is Option B.

Distraction osteogenesis allows anterior movements of the midface that cannot be achieved with conventional Le Fort III and bone grafting. The advancement obtained with single-stage midfacial advancement is limited by soft-tissue tension to 10 to 12 mm, and bone grafting is required to buttress the advanced segments. These procedures have long operative times and major blood loss, and some reports suggest long-term relapse presumably due to bone graft resorption. In contrast, the gradual soft-tissue creep achieved with distraction allows osseous advancement in excess of 30 mm. In addition, distraction osteogenesis obviates the need for bone grafting and relapse is minimal.

The effect of successful midfacial advancement is a decrease in SNB, an increase in ANB, decrease in negative overjet, and an increase in upper airway volume. The latter change often leads to an improvement in obstructive sleep apnea. Exorbitism is reduced by the increase in orbital volume.

83
Q

A 2-year-old girl is brought to the office because of the lower eyelid colobomas and malar deficiency shown in the photograph. Which of the following syndromes is most likely?

A) Apert
B) Crouzon
C) Goldenhar
D) Pfeiffer
E) Treacher Collins

A

The correct response is Option E.

Bilateral lower eyelid colobomas are commonly found in patients with Treacher Collins syndrome (TCS). TCS is also known as mandibulofacial dysostosis, first and second branchial arch syndrome, and Franceschetti-Zwahlen-Klein syndrome. Edward Treacher Collins described the syndrome in 1900. It is autosomal dominant with variable penetrance and has an incidence of 1 in 7,000 live births. It has significant dysmorphology, which includes lower eyelid colobomas, clefted and hypoplastic zygomas, clefted lateral orbit, hypoplastic mandible, lateral canthal vertical dystopia, antimongolian palpebral fissure, ear deformities, long anterior sideburns, anterior open bite, cleft palate, and macrostomia.

Crouzon, Apert, and Pfeiffer syndromes involve craniosynostosis, typically bicoronal. These patients also have underdevelopment of the mid face also. They do not have any eyelid abnormalities. Apert patients also may have a cleft palate and syndactyly of the hands and feet.

Goldenhar syndrome involves epibulbar dermoids of one or both eyes and underdevelopment of one or both sides of the face. It is also known as hemifacial and bifacial microsomia. The soft tissue as well as the bone is hypoplastic.

84
Q

A male newborn is emergently intubated in the prenatal intensive care unit because of severe respiratory distress. Examination shows marked mandibular micrognathia and a cleft palate. Nasopharyngoscopy shows glossoptosis, laryngomalacia, and laryngeal webbing. Which of the following is the most appropriate treatment for the airway compromise?

A) Extubation and prone positioning
B) Insertion of a nasal airway
C) Mandibular distraction
D) Tongue-lip adhesion
E) Tracheostomy

A

The correct response is Option E.

Treatment of airway compromise associated with Robin sequence depends on the severity and etiology of the airway obstruction. Obstruction is often caused by glossoptosis (tongue retropositioning into the oral pharynx), but this may not be the only cause of respiratory distress in these infants. Additional anatomic and physiologic causes of poor respiration (e.g., central apnea, laryngeal anomalies) have been documented in Robin sequence patients and can limit treatment options. Mandibular distraction and tongue-lip adhesion are both effective addressing the glossoptosis and upper airway obstruction, but will have no effect on the subglottic laryngomalacia and webbing. Similarly, prone positioning and nasal airway do not address the lower airway causes of airway obstruction in this child. Only tracheostomy will bypass the laryngeal anomalies and ensure adequate airway patency.

85
Q

A 20-month-old girl is brought to the office for evaluation of mid face hypoplasia, craniosynostosis, and anomalies of the hands and feet. The most likely cause of these findings is a genetic error in which of the following?

A) 22q.11
B) FGFR2
C) IRF6
D) TCOF1

A

The correct response is Option B.

The patient described has Apert syndrome. This autosomal dominant syndrome is characterized by bicoronal craniosynostosis leading to turribrachycephaly, mid face hypoplasia, and complex hand and foot syndactyly. Patients with Crouzon syndrome, an autosomal dominant disorder, typically have craniosynostosis involving the coronal, sagittal, and lambdoid sutures, as well as turribrachycephaly. Other findings include mid face hypoplasia, exorbitism, and proptosis. The extremities are normal. Defects in the Fibroblast Growth Factor Receptor-2 (FGFR2) are found.

22q deletion syndrome, which has several presentations, including DiGeorge syndrome, velocardiofacial syndrome, and Shprintzen syndrome, is caused by the deletion of a small piece of chromosome 22. The deletion occurs near the middle of the chromosome at a location designated q11.2; i.e., on the long arm of one of the pairs of chromosome 22. Characteristic signs include congenital heart disease, cleft palate, learning disabilities, mild elongation of facial features, and mental illness in the teenage years.

Mutations in the IRF6 gene cause van der Woude syndrome. Van der Woude syndrome is an autosomal dominant form of cleft lip and palate. Affected individuals usually have lip pits.

Mutations in the TCOF1 gene cause Treacher Collins syndrome. The official name of this gene is “Treacher Collins-Franceschetti syndrome 1.” Patients with Treacher Collins syndrome, or mandibular dysostosis, have hypoplasia of the zygoma, maxilla, and mandible, downward slanting of the palpebral fissures, colobomas of the lower eyelids, absence of eyelashes, and auricular defects.

86
Q

A 45-year-old man comes to the office because of slowly progressive unilateral loss of facial volume on the right side that began 30 years ago. Physical examination shows facial asymmetry and a line of cutaneous sclerosis on the forehead. No abnormalities in ocular mobility and visual acuity are noted, and no malocclusion is present. Which of the following is the most appropriate management?

A) Change of antiretroviral medications
B) Immunosuppression with oral administration of corticosteroids and methotrexate
C) Restoration of skeletal support with bone grafting
D) Soft-tissue augmentation with fat grafting
E) Observation only

A

The correct response is Option D.

Parry first described the syndrome of progressive facial atrophy in 1825, followed by Romberg in 1846. Parry-Romberg syndrome (also known as progressive hemifacial atrophy) is a rare neurocutaneous syndrome characterized by progressive shrinkage and degeneration of the tissues beneath the skin, usually on only one side of the face (hemifacial atrophy) but occasionally extending to other parts of the body.

The syndrome often begins with a circumscribed patch of scleroderma in the frontal region of the scalp which is associated with a loss of hair and the appearance of a depressed linear scar extending down through the mid face on the affected side. This scar is referred to as a “coup de sabre,” and is indistinguishable from the scar observed in frontal linear scleroderma.

Symptoms and physical findings usually become apparent during the first or early during the second decade of life. The average age of onset is 9 years, and the majority of individuals experience symptoms before age 20 years. The disease may progress for several years before eventually going into remission.

Most patients do not have severe enough disease to warrant immunosuppression. However, for those with more severe and progressive disease, treatments used include methotrexate (for which there is limited evidence in linear scleroderma), corticosteroids, cyclophosphamide, and azathioprine, but benefits are unclear.

The timing of surgical intervention is generally agreed to be the best following exhaustion of the disease course and completion of facial growth. When Parry-Romberg syndrome is severe, the skeleton may be affected, and bony restoration may be required. Soft-tissue reconstruction in patients with severe disease may require free tissue transfer. It is much more common to begin reconstruction with autologous fat transfers. Off-the-shelf cutaneous fillers have also been employed.

The use of highly active antiretroviral therapy with protease inhibitors can result in a syndrome of peripheral wasting, facial fat atrophy, and central adiposity in as many as 64% of patients who are HIV-positive who are treated with this regimen for 1 year. This is usually a bilateral process, but also responds to similar treatment options. Switching antiretroviral drugs may be effective but is only employed in those patients with complete viral suppression. Injections of poly-L-lactic acid (Sculptra) are now licensed for cosmetic management of facial lipoatrophy.

87
Q

A 2-week-old male newborn is brought to the office for evaluation of median craniofacial dysraphia. His parents say that aside from the deformity, the child is thriving. Physical examination shows Tessier No. 0 and 14 clefts. A clinical photograph and a three-dimensional CT scan are shown. Which of the following additional findings is most likely in this patient?

A) Choanal atresia
B) Contact between the dura and ectoderm through the anterior fontanelle
C) Failure of closure of the foramen cecum
D) Heterotopic glial tissue
E) Rathke pouch cyst

A

The correct response is Option C.

OThe classification scheme by Tessier is perhaps the most accepted and basic approach to describe these rare craniofacial clefts. As it only provides an anatomical description, others have further developed description schemes. Some have an embryonic emphasis to further characterize these disorders. These severe clefts can be associated with encephalocele, which if not repaired can be life-threatening. Nasal dermoids arise from contact between dura and ectoderm through the foramen cecum. These can be dangerous if there is intracranial extension. Heterotopic glial tissue is equivalent to the term glioma because these are generally not connected to the intracranial space and are treated as benign lesions. The Rathke pouch occurs during development when the stomodeum ectoderm invaginates toward the hypophysis and remnants are usually located in the nasopharynx as a cyst. An encephalocele is similar to a glioma but contains meninges and/or brain (encephalomeningocele) and can communicate with the ventricle (encephalomeningocystocele). Fifteen percent of these are intranasal. Resection of these without knowing about their intracranial communication can be disastrous. Left untreated, these lesions can significantly increase the risk of meningitis and be life-threatening.

88
Q

Patients with hemifacial microsomia have an increased incidence of which of the following?

A) Craniosynostosis
B) Glossoptosis
C) Hemihypertrophy
D) Microstomia
E) Velopharyngeal insufficiency

A

The correct response is Option E.

Hemifacial microsomia (HFM) is the second most common congenital anomaly, with a reported incidence of approximately one in 5,600 live births. It is thought to be a result of vascular injury to the first and second branchial arches during the 30th to 45th day of pregnancy. The presentation of HFM is highly variable and ranges from mild facial asymmetry and microtia to a severe asymmetry of the orbit and mid and lower faces. Vertebral, cardiac, and renal malformations can also occur with HFM.

A number of studies document velopharyngeal insufficiency (VPI) in patients with HFM. This VPI results from a unilateral hypodynamic palate. With this lack of movement, the nasopharyngeal port cannot be closed fully, and the speech is hypernasal. Some patients are able to compensate for the asymmetry of the palatal movement and have normal speech. Approximately 15% of patients with HFM have evidence of VPI based on speech evaluation and nasoendoscopy.

89
Q

A 21-year-old man has an intercanthal distance of 37 mm (N = 30 mm), and an interorbital distance of 35 mm (N = 28 mm). Which of the following is the most likely diagnosis?

A ) Esotropia
B ) Exophthalmos
C ) Exotropia
D ) Hypertelorism
E ) Pseudohypertelorism

A

The correct response is Option D.

Orbital hypertelorism refers to a condition in which the interorbital distance, measured from dacryon (the junction of the frontal, lacrimal, and maxillary bones) to dacryon is increased. Orbital hypertelorism occurs in the setting of a number of conditions, including frontonasal malformations, craniofrontonasal dysplasia, craniofacial clefts, encephaloceles, and other mostly syndromic disorders. Some authors distinguish orbital hypertelorism from interorbital hypertelorism. In the latter, the distance between the medial orbital walls is increased just as in orbital hypertelorism but the lateral orbital walls are in a normal position. The mean interorbital distance is 28 mm in men and 25 mm in women.

Exotropia refers to a form of strabismus where the eyes are deviated outward (laterally), in contrast to esotropia, where the eyes are deviated inward (medially). Either exotropia or esotropia may exist in hypertelorism and data are not provided in the question to make this diagnosis. Pseudohypertelorism, or telecanthus, occurs when the intercanthal distance, defined as the distance between the medial canthal tendon insertions, is increased but the interorbital distance is normal. Telecanthus may give the appearance of hypertelorism, but can be differentiated on plain film x-rays or computed tomography. Exophthalmos refers to protrusion of the globe due to an increase in the size of the orbital contents in the presence of a normal bony orbit. Graves disease is one of the most common causes of exophthalmos. This is in contrast to exorbitism, which is protrusion of the globe due to a decrease in the capacity of the bony orbit, which may be seen in hypertelorism.

90
Q

The CT scan shown is obtained from a 21-year-old woman. Which of the following is the risk of transmission of this patient’s disease to her offspring?

A ) 25%
B ) 50%
C ) 75%
D ) 100%
E ) Cannot be determined

A

The correct response is Option B.

The patient described has the characteristic craniofacial findings of Treacher

Collins syndrome, notably hypoplasia of the zygomatic bones and mandible, external ear abnormalities, coloboma (notching) of the lower eyelid, absence of the lower eyelid cilia, and preauricular hair displacement. Mutation of the gene TCOF1 is the only gene known to be associated with Treacher Collins syndrome, and it is inherited in an autosomal dominant fashion.

91
Q

A 1-year-old boy is brought to the office because his mother is concerned about the appearance of his mouth (shown). This patient’s condition is most likely the result of incomplete merging of which of the following structures?

A ) Anterior and posterior hillocks
B ) Frontonasal and maxillary prominences
C ) Mandibular and maxillary prominences
D ) Maxillary prominence and medial nasal process
E ) Second and third pharyngeal arches

A

The correct response is Option C.

The patient described has a transverse facial cleft, or macrostomia, resulting from incomplete merging of the mandibular and maxillary prominences of the first pharyngeal arch. This orofacial cleft, also known as a Tessier No. 7 cleft, can vary in size from a minor lateral displacement of the oral commissure to a more extensive defect involving the skin and muscle of the lateral mouth and cheek; the zygomatic arch; and, rarely, the temporal region. Macrostomia can occur in isolation but is associated more commonly with hemifacial microsomia, occurring in up to 23% of this population. The anterior and posterior auricular hillocks, derived from the first and second pharyngeal arches, respectively, form on either side of the first pharyngeal groove at 6 weeks’ gestation and coalesce to form the external ear. The hillocks begin as swellings of mesenchyme at the upper aspect of the embryonic neck and ascend to the side of the head during development. The confluence of the second and third arches takes place in the upper neck, not the face. Cleft lip results from failed merging of the maxillary prominence and the medial nasal process. The frontonasal prominence forms the central forehead, the nasal and perinasal structures, the central portion of the upper lip, and the premaxilla. Failed merging with the maxillary prominence can result in a Tessier No. 2, No. 3, or No. 4 facial cleft.

92
Q

An otherwise healthy 13-year-old girl comes to the office because of a 3-year history of facial asymmetry that has worsened progressively. Examination shows a coup de sabre deformity on the right with the depression extending to the neck. There is thinning of the skin and fat of the right cheek. Which of the following additional findings is most likely in this patient?

A ) Bilateral disease
B ) Inflammatory changes in the skin
C ) Involvement of the tongue in the disease process
D ) Mandibular and maxillary hypoplasia
E ) Vascular malformations

A

The correct response is Option D.

The patient described has Romberg disease, also known as progressive hemifacial atrophy. Hypoplasia is seen in both maxillary and mandibular dental arches. The disorder may be associated with disturbance in normal dental growth, with foreshortening of the dental roots.

The disease is of unknown origin, and characteristically it begins in the first or second decade of life as a localized and progressive atrophy of skin and subcutaneous tissue within the dermatome of one of the branches of the trigeminal nerve. The classic earliest sign is the ‘coup de sabre,’ which affects only about half of the patients. In progressive facial hemiatrophy or Parry-Romberg syndrome, cutaneous inflammation, induration, and adhesion are absent or minimal, and atrophy usually involves only one entire side of the face. The disease is rarely bilateral. Despite the fact that the frontal region is the most commonly affected, the frontal bone is infrequently involved. Vascular malformations are not associated with Romberg disease.

93
Q

A 9-year-old girl is brought for evaluation by her mother because of a 1-year history of headaches and precocious puberty. Physical examination shows café-au-lait spots over her abdomen and a bony prominence of her right parieto-occipital area. CT scan of the calvarium is shown. Which of the following is the most likely diagnosis?

A ) Klippel-Feil syndrome
B ) McCune-Albright syndrome
C ) Paget disease
D ) Proteus syndrome
E ) Renal osteodystrophy

A

The correct response is Option B.

The entity represented in the CT scan shown is fibrous dysplasia, which, in combination with precocious puberty and café-au-lait spots, is known as McCune-Albright syndrome. These patients typically have polyostotic fibrous dysplasia, most commonly affecting the skull, long bones, and ribs. Affected bone may show minor changes that are detectable only on imaging studies such as CT scans or bone scans. Such scans may also show severe change or overgrowth resulting in impingement of the optic nerves, mass effect on the brain, or disfigurement.

The precocious puberty in the scenario described is the result of gonadotropin-independent autonomous ovarian or testicular function and is more common in girls than in boys. Klippel-Feil syndrome is characterized by congenital fusion of any two of the seven cervical vertebrae. Traits of this condition include a short neck, low occipital hairline, and restricted mobility of the upper spine. Paget disease of the bone is also known as osteitis deformans and is a chronic disorder resulting in enlarged, deformed bones. It is typically diagnosed in the third or fourth decade of life by a blood test for alkaline phosphatase. The excessive breakdown and formation of bone can result in weakening of the bone.

Proteus syndrome causes atypical bone development and skin overgrowth. It is an extremely rare congenital disorder, and it is variable in its symptoms. Renal osteodystrophy is a bone mineralization deficiency resulting from electrolyte and endocrine abnormalities associated with chronic kidney disease.

94
Q

An otherwise healthy 10-month-old female infant is brought to the office because her parents are concerned about the appearance of her head. Examination shows left frontal and left posterior flattening with deviation of the nasal root to the left side. Which of the following is the most appropriate management?

A ) Molding helmet
B ) MRI of the head
C ) Positioning
D ) Surgery
E ) Observation only

A

The correct response is Option D.

In the scenario described, flattening of the left side of the forehead, deviation of the nasal root to the left side, and flattening of the left occiput are consistent with left coronal synostosis, and surgery is ultimately the treatment of choice. The scenario does not describe deformational plagiocephaly, which may be treated with a molding helmet, positioning, or observation. In an otherwise healthy baby with these findings, there is no indication for an MRI of the head.

95
Q

Which of the following sutures (A-E) in the figures shown is the first to close in the nonpathologic state?

A ) Sagittal
B ) Metopic
C ) Coronal
D ) Lambdoid
E ) Squamosal

A

The correct response is Option B.

Physiologic or normal closure of the metopic suture occurs first. Recent studies report an earlier closure date than originally thought. The metopic suture may start to close as early as 6 months. The rest of the sutures identified close much later, with the sagittal suture (A) closing at 22 years, the coronal suture (C) closing at 24 years, the lambdoid suture (D) closing around 26 years, and the squamosal suture (E) closing at 35 years.

96
Q

Which of the following synostoses is most predictably treated with endoscopic suturectomy and postoperative orthotic molding?

A) Coronal
B) Lambdoidal
C) Metopic
D) Sagittal
E) Squamosal

A

The correct response is Option D.

Patients with sagittal synostosis may undergo an endoscopic correction that entails a synostectomy of the fused suture and additional bone, microfracturing of the parietal bones, and postoperative orthotic use to achieve an ideal calvarial form with normal cephalic index. There is no type of fixation utilized in this surgery. It must be done at a younger age than the open approach. The ideal time frame is between 2 to 4 months of age. Surgeons have tried endoscopic treatment for other types of synostoses with variable and controversial results. The open approach is still most successful in treatment of lambdoidal, coronal, and metopic synostosis. Squamosal synostosis generally does not require surgical intervention.

97
Q

A 3-day-old female newborn is in the neonatal intensive care unit because of airway obstruction, micrognathia, glossoptosis, and cleft palate. Placement of the patient in the prone position has failed to stabilize the airway. Which of the following is the most appropriate immediate next step in management?

A) Endotracheal intubation
B) Mandibular distraction
C) Tongue-lip adhesion
D) Tracheostomy

A

The correct response is Option A.

The Pierre Robin sequence consists of micrognathia or retrognathia, glossoptosis, and airway obstruction (with or without cleft palate). Cleft palate is a frequently associated feature, but not cleft lip. There is little evidence of genetic transmissions. The retrognathia is believed to contribute to the glossoptosis, which in turn produces the airway obstruction.

Initial management is conservative. Prone positioning is the mainstay of initial airway management. Upright feedings, the use of nasogastric tubes, and endotracheal intubation may assist with the early management of the child.

Before undertaking any operative intervention designed to address the glossoptosis, such as mandibular distraction or tongue-lip adhesion, a nasoendoscopy is recommended. This is performed to rule out other anatomical sites or causes of airway obstruction. For airway obstruction arising only from glossoptosis, a tongue-lip adhesion is a reasonable first treatment option. This procedure is most effective in infants with good prospects for mandibular growth early in infancy (ie, Stickler syndrome, velocardiofacial syndrome, nonsyndromic patients). In some syndromes with poor mandibular growth potential, such as Treacher Collins syndrome or facial microsomia, or when the degree of mandibular hypoplasia or retrusion is particularly severe, mandibular distraction may provide a more effective management option.

98
Q

A 6-year-old boy is brought to the office because of persistent hypernasal speech. He has a history of cardiac anomalies and learning difficulties. Physical examination shows a broad nose, malar flattening, epicanthal folds, retrognathia, and vertical maxillary excess. Intraoral examination shows a bifid uvula and a palpable notch of the posterior nasal spine. Which of the following imaging studies is most appropriate prior to surgical intervention?

A) Carotid angiography
B) MRA of the head and neck
C) PET scan of the brain
D) Renal ultrasonography

A

The correct response is Option B.

The patient described appears to have velocardiofacial syndrome, an autosomal dominant condition caused by a deletion of the long arm of chromosome 22. Manifestations of velocardiofacial syndrome include cleft palate, velopharyngeal insufficiency, and cardiac abnormalities. Abnormal facial features associated with this syndrome include a broad, prominent nose, malar flattening, epicanthal folds, retrognathia, and vertical maxillary excess. MRA is the diagnostic study of choice for detecting abnormalities of the carotid vasculature, notably medialization, which may complicate palatal or pharyngeal surgery. While carotid angiography would yield similar information, it is too invasive. Neither PET scan of the brain nor renal ultrasonography addresses this issue.

99
Q

A 6-week-old female infant is brought to the office for evaluation of a skull deformity shown in the photograph. Physical examination shows the absence of calvarial bone in multiple areas of the cranium. CT scan (also shown) confirms a kleeblattschädel skull deformity. Which of the following is the most likely indication for surgical intervention at this time?

A) Airway compromise
B) Hydrocephalus
C) Increased intracranial pressure
D) Loss of vision
E) Orbital exposure

A

The correct response is Option C.

The kleeblattschädel skull deformity occurs secondary to multiple suture synostoses. This results in a significant increase in intracranial pressure that causes the skull deformity shown in the CT scan. The increase in intracranial pressure produces a ?moth-eaten? appearance and is the reason for early surgical intervention. This skull deformity is not typically associated with hydrocephalus. Visual compromise sometimes occurs late in life secondary to increased intracranial pressure. Orbital exposure can be a reason for early surgical intervention in some types of syndromic craniosynostosis, such as Crouzon syndrome and Apert syndrome, but is not seen in the scenario described. Airway compromise would be an indication for urgent surgical intervention, but it is not typical for kleeblattschädel skull deformity.

100
Q

A 3-year-old boy is brought to the office because of a congenital soft tissue notch of the lower lateral eyelid. Which of the following is the most appropriate Tessier classification for the underlying craniofacial cleft?

A) Tessier No. 3
B) Tessier No. 4
C) Tessier No. 6
D) Tessier No. 10

A

The correct response is Option C.

The Tessier No. 3 cleft defect symptoms include a defect between the lateral incisors and canine. This cleft typically involves the alar base and medial canthal region. In severe cases, the cleft may enter the orbit medial to the punctum. Tessier No. 4 facial clefts extend from the upper lip, around the alar base, along the nasomaxillary junction, and across the tear duct and medial orbital tissues. Tessier No. 6 facial clefts may disrupt bone and soft tissues along the lower lateral orbit. These defects are frequently associated with colobomas of the lower eyelid. Extending into the frontal bone, Tessier No. 10 clefts involve the superior orbital rim and medial third of the orbit. Proptosis with fronto-orbital encephalocele may be present with significant clefts at this location. No. 7 clefts may produce macrostomia and extend through the lateral zygomatic arch. An image is shown.

101
Q

A 16-year-old girl is brought to the office for consultation regarding reconstruction to correct hemifacial atrophy. The parents first noticed the condition when the patient was 6 years old; it has been stable for 18 months. Physical examination shows an asymmetric face with atrophy of the right side. There is significant unilateral atrophy of skin, subcutaneous tissue, and bone. Facial reconstruction is planned. Which of the following is the most appropriate method of reconstruction?

A) Latissimus dorsi free flap
B) Omental free flap
C) Osteocutaneous fibula flap
D) Parascapular free flap
E) Silicone injection

A

The correct response is Option D.

Of the choices given, the most appropriate method of reconstruction in the patient described, who has Romberg disease, is a parascapular free flap.

Patients with Romberg disease may have deformities composed of skin, subcutaneous tissue, muscle, and bone. Options for reconstruction include synthetic material and implants, biologic fillers, bone grafts, fat grafts, and free tissue transfer.

Facial contouring with silicone injection has been unsatisfactory with severe scarring, contracture formation, and skin breakdown. Removal of liquid silicone can be a very challenging task and can make future reconstructions more difficult. Muscle and myocutaneous flaps are not ideal, as they can be too bulky, and the eventual muscle atrophy leads to unpredictable long-term results. Omental flaps for facial recontouring have been described, but they have several drawbacks. These include the need for an intra-abdominal harvest and the difficulty in long-term flap fixation, with eventual descent given the absence of dermal or fascial components to be used in fixation. The parascapular flap provides a versatile source of composite tissue that remains relatively stable as it matures, and it provides tissue components for appropriate fixation.

102
Q

A 40-year-old Caucasian woman comes to the office for consultation because she is dissatisfied with the appearance of her smile. Physical examination shows bilabial incompetence, malocclusion, a retrusive chin, and a gummy smile. Cephalometric analysis shows decreased SNA and SNB angles. Which of the following is the most likely diagnosis?

A) Mandibular deficiency
B) Mandibular excess
C) Maxillary retrusion
D) Vertical maxillary deficiency
E) Vertical maxillary excess

A

The correct response is Option E.

Long-face deformity (vertical maxillary excess) is characterized by an increase in the lower third of the face. A gummy smile is associated with this deformity. The mid facial appearance is flattened. Although all types of Angle occlusion patterns are possible, type II is most common.

Mandibular retrusion is associated with a retrusive chin and Angle class II malocclusion, but the teeth would not be visible in repose. Mandibular excess would not be associated with a retrusive chin but would be associated with Angle class III malocclusion.

Maxillary retrusion is associated with maxillary hypoplasia and often a shortening of the lower vertical facial height. Angle class III malocclusion is seen with this deformity, and cephalometric analysis shows a decreased SNA angle but an increased SNB angle.

Vertical maxillary deficiency, although often associated with Angle class II malocclusion, results in shortening of the lower third of the face, and the teeth are not seen in repose. SNA and SNB angles are frequently increased in this deformity.

103
Q

A 38-year-old man comes to the office because of facial asymmetry and pain in the left mid face. He says that he first started noticing the asymmetry at 14 years of age. The pain began 3 years ago. Physical examination shows atrophy of the temporal and mid facial soft tissue and bone. A photograph is shown. Which of the following is the most likely diagnosis?

A) Bell palsy
B) Hemifacial microsomia
C) Parry-Romberg syndrome
D) Torticollis
E) Treacher Collins syndrome

A

The correct response is Option C.

Acquired facial asymmetry, as demonstrated in the photograph, is known as Parry-Romberg syndrome, or hemifacial atrophy. It involves both skeletal and soft tissue, and it is characterized by slow atrophy of the tissues of half of the face. It typically appears between the ages of 5 and 15 years, and it can be associated with neurologic symptoms such as seizures and trigeminal neuralgia. Facial atrophy usually begins in the temporal and/or nasolabial fold region and progresses to involve the mouth, the area around the eye, and the brow. Many patients go on to develop atrophy of half of the upper lip and tongue and exposure or damage to the roots of teeth on the affected side. The skin overlying the affected areas may become hyperpigmented. The atrophy may progress slowly and plateau, or it may progress indefinitely. The etiology of Parry-Romberg syndrome is unknown and occurs sporadically.

Bell palsy is a mononeuropathy involving cranial nerve VII and the facial nerve; it results in unilateral facial nerve paralysis. It is not usually self-limiting but may result in residual weakness in rare cases.

Hemifacial microsomia is a congenital anomaly involving an underdevelopment of either one or both sides of the face. It more commonly affects the lower half of the face, resulting in mandibular hypoplasia, microtia, and vestigial remnants. Hemifacial microsomia is the second most common facial congenital difference, after cleft lip and palate.

Torticollis involves tightness of the sternocleidomastoid muscle of the neck resulting in a head tilt toward the affected muscle. Patients may have a mild underdevelopment of the lower half of the face on the affected side secondary to deformational pressure. Torticollis is generally congenital, thought to be secondary to intrauterine positioning, and is treated with physical therapy.

104
Q

Which of the following syndromes is most commonly associated with Pierre Robin sequence?

A) Beckwith-Wiedemann
B) Facial microsomia
C) Stickler
D) Treacher Collins
E) Velocardiofacial

A

The correct response is Option C.

Pierre Robin sequence denotes a group of physical findings that include micrognathia or retrognathia, glossoptosis, and respiratory distress with or without cleft palate. Once thought to be a syndrome, it is now understood to be a developmental sequence; ie, ?a pattern of multiple anomalies derived from a single known or presumed prior anomaly or mechanical factor.? The underdeveloped (micrognathia) or retropositioned (retrognathia) mandible limits space for the growing tongue and forces it to assume a posterior/elevated position in the oropharynx (glossoptosis). This leads to delayed elevation/fusion of the palatal shelves (cleft palate) and respiratory distress.

The mandibular anomaly in Pierre Robin sequence is causally heterogeneous and can result from either a malformation (eg, Stickler syndrome), a disruption (eg, amniotic band syndrome), or deformation (eg, oligohydramnios). Nearly half of all cases have an identifiable syndrome, and over 40 associated syndromes have been described. In syndromic Pierre Robin sequence, over 40% are Stickler, which is caused by one of three collagen mutations (type II, COL2A1 is the most common) Stickler patients have wide phenotypic variability that often results in delayed or missed diagnosis in minor cases. Clinical findings associated with Stickler syndrome are ocular problems (retinal detachment, myopia, and blindness), facial abnormalities (flat nose, small mandible, or cleft palate), hearing loss, and degenerative joint disease and pain.

Although patients with Beckwith-Wiedemann syndrome, facial microsomia, Treacher Collins syndrome, and velocardiofacial syndrome may have symptoms of Pierre Robin sequence, these associations are significantly less common than Stickler syndrome.

105
Q

A 2-month-old male infant is brought to the office because of mid face hypoplasia, craniosynostosis, and bilateral hand and foot anomalies. A photograph of the left foot is shown. This patient most likely has which of the following syndromes?

A) Apert
B) Crouzon
C) Goldenhar
D) Nager
E) Treacher Collins

A

The correct response is Option A.

The patient described has Apert syndrome. This autosomal dominant syndrome is characterized by bicoronal craniosynostosis that leads to turribrachycephaly, mid face hypoplasia, and complex hand and feet syndactyly. Patients with Crouzon syndrome, an autosomal dominant disorder, typically have craniosynostosis involving the coronal, sagittal, and lambdoid sutures, as well as turribrachycephaly. Other findings include mid face hypoplasia, exorbitism, and proptosis. The extremities are normal.

Goldenhar syndrome, or oculoauriculovertebral dysplasia, involves asymmetry of the hard and soft tissues of the face. This condition is most commonly unilateral but may be seen bilaterally in some patients. Manifestations of this syndrome include hypoplasia involving the mandible and underlying soft tissues of the face, epibulbar dermoids, and varied degrees of microtia on the affected side. Most patients have associated vertebral abnormalities. Nager syndrome, or acrofacial dysostosis, is an autosomal recessive disorder characterized by craniofacial and upper extremity abnormalities. Patients with Nager syndrome have hypoplasia of the orbits, zygoma, maxilla, mandible, and soft palate. Auricular defects may also be present. Hypoplasia or agenesis occurs in the radius, thumbs, and metacarpals. Some patients may have radioulnar synostosis and elbow joint deformities. Patients with Treacher Collins syndrome, or mandibular dysostosis, have hypoplasia of the zygoma, maxilla, and mandible, downward slanting of the palpebral fissures, colobomas of the lower eyelids, absence of eyelashes, and auricular defects.

106
Q

An 11-year-old boy is brought to the office because of a 1-year history of progressive left-sided hemifacial atrophy. He has the classic coup de sabre appearance. Examination shows facial bony structures that are intact. Which of the following imaging studies is most appropriate to include in a diagnostic workup of this patient?

A ) MRI of the abdomen

B ) MRI of the brain

C ) Plain x-ray study of the chest

D ) Plain x-ray studies of the hands

A

The correct response is Option B.

The patient described has progressive hemifacial atrophy, or Parry-Romberg syndrome (PRS). This rare disorder is characterized by progressive wasting of the skin and subcutaneous tissues, and, in some cases, the muscle and bone of the face. It is typically unilateral. Age of onset is the first or second decade of life in most cases. Progression of the wasting may continue for several years. The etiology is unknown, but autoimmune, infectious, neurologic, and traumatic causes have all been implicated in various cases and investigations, and it is considered by many to be part of the spectrum of linear scleroderma. Lacking any adequate medical treatment, PRS is typically allowed to run its course, with reconstruction of the resulting defects with fat injection, soft-tissue augmentation with free tissue transfer, bone grafting, and orthognathic surgery as indicated.

Neurologic symptoms are commonly associated with PRS, including seizures, migraine, Horner syndrome, and hemiplegia. Various ophthalmologic conditions are also common. Abnormalities are frequently seen on MRI of the brain, even in the absence of neurologic symptoms, and therefore MRI of the brain is indicated such that these abnormalities might be investigated further as needed. Defects in the skin may also occur elsewhere in the body, but bony abnormalities of the extremities or abnormalities of the chest, abdomen, or neck are not typically found. Therefore, the other imaging studies listed would not be expected to yield useful information in most cases of PRS. Other appropriate tests might include serology for viral or autoimmune etiologies of PRS.

107
Q

A 4-year-old girl is brought to the office for evaluation of hemifacial microsomia. Physical examination shows maxillary hypoplasia, orbital dystopia, and complete absence of the mandibular condyle. The presence of which of the following additional findings is most suggestive of Goldenhar syndrome?

A ) Epibulbar dermoids

B ) Facial nerve VII impairment

C ) Midfacial port-wine stai

n D ) Multiple pits of the lower lip

E ) Upper eyelid colobomas

A

The correct response is Option A.

The presence of epibulbar dermoids is a key clinical finding that distinguishes Goldenhar syndrome from hemifacial microsomia. Although Goldenhar syndrome is also frequently associated with defects of cranial nerve VII, this finding is also commonly described in other craniofacial anomalies, including hemifacial microsomia.

A port-wine stain present within the V1 or V2 distribution is suggestive of potential Sturge-Weber syndrome.

Van der Woude syndrome is commonly associated with lower lip pits as well as cleft lip/palate.

Upper lid colobomas are often described in relation to hemifacial microsomia, of which Goldenhar syndrome is a variant.

108
Q

An 8-year-old boy is brought to the office because he has been unable to smile, close his mouth, or completely close his eyes since birth. Physical examination shows facial immobility, strabismus, and syndactyly of the ring and little fingers. Which of the following is the most likely diagnosis?

A ) Klippel-Trénaunay syndrome

B ) Möbius syndrome

C ) Pierre Robin sequence

D ) Poland syndrome

E ) Treacher Collins syndrome

A

The correct response is Option B.

Although von Graefe described a case of congenital facial diplegia in 1880, the syndrome was reviewed and defined further by Paul Julius Möbius, a German neurologist, in 1888 and 1892. Because of these contributions, Möbius is now the eponym used to describe the syndrome. In most studies, Möbius syndrome is defined as congenital facial weakness combined with abnormal ocular abduction €“ weakness of cranial nerves VII and VI. The typical phenotypic appearance of an affected individual includes an immobile facial appearance and ocular palsy. A mask-like facial appearance is pathoneumonic. Additional musculoskeletal abnormalities occur in one third of patients with Möbius syndrome.

The typical physical features of Treacher Collins syndrome include downward slanting eyes, lower eyelid colobomas, micrognathia, conductive hearing loss, underdeveloped zygoma, and malformed or absent ears. This is also considered the combination of Tessier No. 6, 7, and 8 clefts.

Poland syndrome is characterized by underdevelopment or absence of the pectoralis muscle on one side. Syndactyly often occurs on the ipsilateral hand. There are no associated facial anomalies.

Pierre Robin is not a syndrome but rather a sequence or a collection of physical findings that appear together. Phenotype includes micrognathia, glossoptosis, and cleft palate. Breathing and eating difficulty often results.

Klippel-Trénaunay syndrome is characterized by a triad of port-wine stain, varicose veins, and bony and soft-tissue hypertrophy involving an extremity.

109
Q

A 2-day-old male newborn is evaluated in the neonatal intensive care unit because of a “jaw deformity” and difficulty breathing. The patient was born at term following an uncomplicated pregnancy and delivery. He responds appropriately to stimulation. Examination shows micrognathia, glossoptosis, and a cleft palate. The infant demonstrates retractions while breathing. Pulse oximetry shows an oxygen saturation of 92%. Which of the following is the most appropriate initial management?

A ) CT scan of the head with three-dimensional reconstruction

B ) Endoscopic evaluation of the airway

C ) Prone positioning of the newborn

D ) Tongue-lip adhesion

E ) Tracheostomy

A

The correct response is Option C.

The patient described has Pierre Robin sequence. These patients have micrognathia and glossoptosis; sometimes they will have a cleft palate in addition to the other findings. Most patients do not require operative intervention to correct their mandibular hypoplasia. The most important first step is airway control. These patients have airway obstruction secondary to the large size of their tongue relative to their diminutive mandible. Placing them in prone position allows for the tongue and jaw to fall forward, frequently alleviating their airway problems. Should this be insufficient, tongue-lip adhesion is a possibility. Many surgeons would consider a more thorough evaluation of the airway endoscopically and by CT scan to decide on subsequent management. Should the airway obstruction be tongue-based as expected, bilateral mandibular distraction could be considered. If lower airway anomalies are also present, then tracheostomy is more appropriate.

110
Q

A 2-month-old female infant is brought to the office because her parents are concerned about the flat appearance of her forehead that they first noticed 2 weeks ago. Physical examination shows flattening of the right side of the forehead and left side of the occiput, and the left ear is positioned farther forward than the right. A photograph is shown. Which of the following is the most appropriate initial management?

A ) Calvarial vault remodeling

B ) CT scan

C ) MRI

D ) Placement of a molding helmet

E ) Repositioning exercises

A

The correct response is Option E.

The physical findings described are consistent with deformational (positional) plagiocephaly. In contradistinction to craniosynostosis, deformational plagiocephaly will demonstrate deviation of the nasal root away from the side of the forehead with flattening, and the supraorbital rim will be depressed or lowered. Physical findings are sufficient in the scenario described to diagnose the child without CT scan. Occupational and physical therapy should be instituted to encourage behaviors such as head-turning to the right and tipping the top of the head to the left shoulder. In cases that are either severe or are diagnosed late, molding helmets may be advisable.

111
Q

A 30-year-old man comes to the office for consultation regarding the facial abnormality shown. He reports that he first noticed a change 15 years ago and that the deformity has worsened since then. Medical history includes atrophy of the soft tissue and bone of the face. The facial atrophy in this syndrome progresses according to the specific distribution of which of the following nerves?

A ) Facial (VII)

B ) Glossopharyngeal (IX)

C ) Hypoglossal (XII)

D ) Trigeminal (V)

E ) Vagus (X)

A

The correct response is Option D.

Parry-Romberg syndrome is a progressive hemifacial atrophy that follows a specific distribution of one or more branches of the trigeminal nerve (cranial nerve V). It involves the skin, soft tissue, cartilage, and bone and was first described in 1825 by Parry and then in 1846 by Romberg. The epidemiology shows a female-to-male ratio of 1.5:1. It can appear in early infancy or adolescence. The etiology is unclear, and theories include a genetic alteration in the embryogenesis of the central nervous system, loss of the cervical sympathetic nerve after neuritis, and viral or bacterial infection.

112
Q

A 1-year-old boy has had the anomaly shown since birth. Which of the following syndromes is the most likely diagnosis?

A ) Apert

B ) Crouzon

C ) Goldenhar

D ) Saethre-Chotzen

E ) Treacher Collins

A

The correct response is Option E.

Bilateral lower eyelid colobomas are commonly found in patients with Treacher Collins syndrome (TCS). TCS is also known as mandibulofacial dysostosis, first and second branchial arch syndrome, and Franceschetti-Zwahlen-Klein syndrome. Edward Treacher Collins described the syndrome in 1900. It is autosomal dominant with variable penetrance and has an incidence of 1 in 7000 live births. TCS has significant dysmorphology, which includes lower eyelid colobomas, cleft and hypoplastic zygomas, cleft lateral orbit, hypoplastic mandible, lateral canthal vertical dystopia, antimongolian palpebral fissure, ear deformities, long anterior sideburns, anterior open bite, cleft palate, and macrostomia.

Crouzon, Apert, and Saethre-Chotzen syndromes involve craniosynostosis, typically bicoronal. Patients with these syndromes also have underdevelopment of the mid face. They do not have eyelid abnormalities. Patients with Apert syndrome also may have a cleft palate and syndactyly of the hands and feet.

Goldenhar syndrome involves epibulbar dermoids of one or both eyes and underdevelopment of one or both sides of the face. It is also known as hemifacial and bifacial microsomia. The soft tissue and the bone are hypoplastic.

113
Q

A 30-year-old woman comes to the office because of a 1-year history of a clicking sensation when she opens her mouth. She was involved in a motor vehicle collision in which her face struck the steering wheel 1 year ago. Physical examination shows midline dental structures without deviation. Which of the following is the most likely cause of this patient’s condition?

A ) Disruption of the lateral pterygoid muscle

B ) Foreign body within the joint space

C ) Malunion of a coronoid fractur

e D ) Nonunion of a condylar fracture

E ) Subluxation of the articular disk

A

The correct response is Option E.

Motion at the temporomandibular joint (TMJ) is best appreciated by placing one €™s fingers either inside the external auditory canal or just anterior to it. The sensation of clicking when the jaw is repeatedly opened and closed is usually caused by subluxation of the articular disk. The disk normally lies centrally between the two joint spaces. Conservative treatment involves adjustment of the patient €™s bite with a splint, anti-inflammatory drugs, and physical therapy. Surgical treatment is reserved for patients who fail conservative therapy. Air within the joint space may occur following open fractures of the mandibular condyle. The presence of a foreign body within the joint space produces pain and decreased range of motion rather than clicking. Similar symptoms are also noted in patients with degenerative disease affecting the TMJ.

114
Q

A 9-month-old boy is brought to the office because of a midline glabellar mass. The parents report that it has enlarged gradually since they first noticed it 6 months ago; it does not change in size when the patient cries. Physical examination shows a nonmobile, firm, and nontender mass. The nasal root is not broadened, and intercanthal distance is within normal limits. Which of the following is the most likely diagnosis?

A ) Dermoid cyst

B ) Encephalocele

C ) Glioma

D ) Hemangioma

E ) Pilomatricoma

A

The correct response is Option A.

The most likely diagnosis in the scenario described is a dermoid cyst. Nasal dermoids are the most common congenital nasal mass. Dermoid cysts most often occur in children in the lateral brow or midline glabellar region. They generally grow slowly, and intracranial communication should be ruled out with either a CT scan or MRI. Intracranial communication is less likely in this scenario because there is no broadening of the nasal root or increased intercanthal distance.

An encephalocele is a midline malformation that is present at birth and addressed shortly thereafter. On physical examination, it would be soft and mobile. CT scan or MRI would confirm this diagnosis. A hemangioma or encephalocele may change size with crying.

A glioma is heterotopic neural tissue left during the regression of neurologic tissue during embryonic development. Like an encephalocele, broadening of the nasal root and widened intercanthal distance are common.

A hemangioma typically presents with sporadic growth during the first 12 months of life, then it reaches a plateau and eventually regresses in most cases. On physical examination, this would be neither fixed nor firm and is typically discolored.

A pilomatricoma is a rare, benign, circumscribed epithelial neoplasm that is derived from hair matrix cells. It is classically not fixed and very superficial.

115
Q

A 12-month-old boy is referred by his pediatrician for possible craniosynostosis. He is healthy and has achieved developmental milestones appropriately. His parents report that he has a ridge on his forehead that they first noticed when he was 3 months of age. Physical examination shows a palpable midline ridge with normal facial contour. CT scans obtained by his pediatrician are shown. Which of the following is the most appropriate management?

A ) Diagnostic plain x-ray studies

B ) Endoscopic strip craniectomy and postoperative helmet therapy

C ) Fronto-orbital advancement

D ) Serologic testing for mutations of fibroblast growth factor receptors 1, 2, and 3

E ) Observation

A

The correct response is Option E.

Craniosynostosis results in characteristic changes to the cranium. The metopic suture is different because unlike other calvarial sutures that ultimately fuse in the third decade of life, it normally

closes before 12 months of age. Accordingly, the finding of a closed suture at this age is not abnormal. The phenotype associated with premature (or pathological) metopic fusion is trigonocephaly ( €œtriangle head €). Patients present with a midline forehead prominence, a variable degree of frontal and lateral orbital narrowing, bilateral parietal widening, and hypotelorism. In contrast, the presence of an isolated metopic ridge is a normal variant. It is the shape of the fronto-orbital region that determines the need for operative intervention.

The CT scans clearly demonstrate metopic fusion; plain x-ray studies are not necessary to confirm the diagnosis. Because the patient described has only an isolated metopic ridge (not trigonocephaly), operative treatment is not warranted. Additionally, endoscopic strip craniectomy is generally not effective after 3 months of age. Mutations in fibroblast growth factor receptors 1, 2, and 3 have been identified in unilateral and bilateral coronal synostosis but not in isolated metopic synostosis.

116
Q

A 15-year-old girl with a history of an optic glioma, multiple café-au-lait spots, and a large plexiform neurofibroma of the cheek comes to the office for evaluation. Which of the following best represents her lifetime risk of developing a malignant peripheral nerve sheath tumor?

A ) Less than 15%

B ) 20% to 35%

C ) 40% to 60%

D ) 65% to 80%

E ) Greater than 85%

A

The correct response is Option A.

The patient described has neurofibromatosis 1 (NF1), an autosomal genetic disorder that leaves affected individuals at risk for developing a variety of benign and malignant tumors. The most common tumors are neurofibromas and optic gliomas. Plexiform neurofibromas are clinically present in approximately 25% of patients. This type of neurofibroma consists of a network of neurofibroma tissue and grows along the length of nerves, often involving multiple nerve fascicles, branches, and plexi. Individuals with NF1 have a 7 to 13% lifetime risk of developing a malignant peripheral nerve sheath tumor (MPNST), which usually arises in a pre-existing plexiform neurofibroma. Diagnosis of a MPNST is problematic within the context of NF1 because the emergence of a lump is not unusual. The clinical symptoms of malignancy are intertwined with the symptoms of benign tumors. Rapid growth and other symptoms, such as pain, are indications for the need for a biopsy.

117
Q
  • Which of the following cranial sutures is the first to undergo physiologic closure?
  • A ) Coronal
  • B ) Lambdoid
  • C ) Metopic
  • D ) Sagittal
  • E ) Squamosal
A

•Physiologic or normal closure of the metopic suture occurs prior to any of the other sutures listed. Recent studies report a potentially even earlier closure of the metopic, at six months, than previously thought. Prior to this most recent study, the metopic was thought to close normally around 2 years of age. All of the other sutures begin closure much later with the closure progression being sagittal (22 years), coronal (24 years), lambdoid (26 years), and the squamosal (35 years).

118
Q
  • A 3-year-old boy with hemifacial microsomia has the anomaly shown in the photograph. Which of the following is the most likely diagnosis?
  • A ) Goldenhar syndrome
  • B ) Stickler syndrome
  • C ) Treacher Collins syndrome
  • D ) Van der Woude syndrome
  • E ) Velocardiofacial syndrome
A
  • Goldenhar syndrome (oculoauriculovertebral spectrum) is a subset of hemifacial microsomia and, in addition to microsomia of the face, includes vertebral anomalies and/or epibulbar dermoids (as shown in the photograph).
  • Stickler syndrome, or hereditary arthro-ophthalmopathy, presents with flat facies, myopia, and spondyloepiphyseal dysplasia and is associated with clefts of the palate.
  • Treacher Collins syndrome is also known as mandibulofacial dysostosis. Patients with Treacher Collins have bilateral Tessier No. 6, 7, and 8 clefts and present with mandibular deformities, lower eyelid colobomas, auricular deformities, and hypoplastic zygomas.
  • Patients with van der Woude syndrome present with lower lip pits, with or without cleft lip, with or without missing second premolars.
  • Velocardiofacial syndrome, or the 22q.11 deletion, is found in 8% to 10% of isolated clefts of the palate and is associated with heart anomalies, velopharyngeal dysfunction, and other conditions.
119
Q
  • A 6-month-old girl is brought to the office for evaluation of left anterior plagiocephaly. Physical examination shows posterior displacement of the ipsilateral supraorbital rim, deviation of the nasal root, and bossing of the right forehead. A three-dimensional CT is shown. Which of the following is the most appropriate management?
  • A ) Fronto-orbital advancement
  • B ) Molding helmet therapy
  • C ) Monobloc advancement
  • D ) Repositioning of the child during sleep
A
  • The patient described has unilateral coronal synostosis. The most common presenting symptom is ipsilateral anterior forehead flattening, or plagiocephaly. This finding can be seen in patients with deformational plagiocephaly. Several anatomic features can be used to differentiate these two different etiologies of plagiocephaly.
  • Synostotic plagiocephaly is caused by early fusion of the coronal suture on the affected side, or unicoronal synostosis. It is far less common than deformational plagiocephaly, which is thought to be caused by intrauterine deformational forces acting in the fetal skull.
  • In synostotic plagiocephaly, there is widening of the ipsilateral palpebral fissure, superior and posterior displacement of the supraorbital rim and eyebrow on the affected side, a higher ipsilateral ear, and deviation of the nasal root toward the affected side. The anterior fontanelle is displaced away from the affected side. The appropriate surgical procedure to correct synostotic plagiocephaly is a fronto-orbital advancement procedure. Resection of the left coronal suture will eliminate the abnormal suture but will not correct the other facial deformities.
  • In contrast, in deformational plagiocephaly, there is an ipsilateral narrowing of the palpebral fissure with a lower eyebrow, an inferiorly positioned ipsilateral ear, and a normal nasal root and anterior fontanelle. The cranial vault looks compressed from the forehead to the opposite occiput. Possible treatment options for deformational plagiocephaly include repositioning of the child during sleep and molding helmet therapy.
  • A monobloc advancement procedure involves repositioning of both the forehead and midface in a single operation and is not indicated to treat plagiocephaly.
120
Q

A 3-year-old girl with mid face hypoplasia, proptosis, and malocclusion is brought to the office for consultation. Which of the following is the most likely occlusal relationship in this patient?

  • A ) Angle class I
  • B ) Angle class II, division 1
  • C ) Angle class II, division 2
  • D ) Angle class III
A

•The patient described with Crouzon disease and a hypoplastic mid face has an Angle class III malocclusion. Angle class I is characterized by the alignment of the mesiobuccal cusp of the maxillary first molar with the mesiobuccal groove of the mandibular first molar. Angle class II is characterized by the alignment of the buccal groove of the lower first molar distal to the mesiobuccal cusp of the upper first molar. Angle class II division 1 is associated with flaring of the maxillary incisors and increased overjet. Angle class II division 2 is associated with lingually displaced maxillary teeth and excessive labial inclination of the maxillary central incisors. Angle class III is characterized by the alignment of the buccal groove of the lower first molar mesial (anterior) to the mesiobuccal cusp of the upper first molar.

121
Q
  • A 3-year-old boy is brought to the office by his parents for consultation for macrostomia caused by a lateral facial cleft. This patient is most likely to have a bony defect that traverses which of the following locations?
  • A ) Mandibular bicuspid
  • B ) Mandibular lateral incisor
  • C ) Maxillary canine
  • D ) Maxillary lateral incisor
  • E ) Maxillary second molar

A

•The patient described has a Tessier No. 7 facial cleft, which involves the soft tissue of the oral commissure, sideburns, and external ear (shown below). The bony involvement generally traverses the zygomatic arch and maxillary second molar.

122
Q
  • Syndromic cleft palate is most commonly found in patients with which of the following syndromes?
  • A ) Apert
  • B ) Saethre-Chotzen
  • C ) Stickler
  • D ) Van der Woude
A

•Stickler syndrome accounts for 25% of syndromic cleft palate cases. As the most common form of syndromic cleft palate, this syndrome is also associated with ocular and auditory defects. Apert syndrome is an autosomal dominant form of syndromic craniosynostosis. Although this syndrome is defined by uni- or bicoronal craniosynostosis and acrosyndactyly, palatal clefting is not recognized as a syndromic component. Saethre-Chotzen syndrome is an autosomal acrocephalosyndactyly characterized by coronal craniosynostosis, low hairline, proptosis, antimongoloid slanting of palpebral fissures, and nasal deviation. Cleft palate defects are not associated with this syndrome. Van der Woude syndrome, associated with lower lip pitting and cleft palate, accounts for 1% of syndromic cleft defects.

123
Q

A 28-year-old man with a skull tumor underwent resection with a resultant bony defect and exposed dura mater. A reconstructive alloplastic cranioplasty was performed, which was complicated by iatrogenic burns to the dura mater, underlying brain, and surrounding soft tissue. Which of the following alloplastic materials was most likely used?

A ) High-density porous polyethylene

B ) Methylmethacrylate

C ) Nonceramic hydroxyapatite cement

D ) Polytetrafluoroethylene

E ) Silicone

A
  • Methylmethacrylate is an alloplastic acrylic resin that has been used in craniofacial reconstruction, forehead augmentation, chest wall reconstruction, and, when impregnated with antibiotics, for treatment of infected long bone fractures. It is fashioned by mixing a powder with a liquid monomer. The resultant paste is formed in an exothermic reaction that can achieve temperatures of 70 °C. Therefore, it is mandatory that the paste be cooled with constant irrigation as it is applied and molded into the defect in order to prevent thermal injury. In animal craniofacial experiments, temperature elevations of 28 °C have been recorded but with subdural temperature elevations of only 0.56 °C if constant irrigation was used.
  • High-density porous polyethylene (Medpor) has also been used for the repair of craniofacial defects as well as in the realm of cosmetic surgery. It is prepackaged in sterile form and is available in sheets, blocks, and prefabricated shapes/implants. It does not generate exothermic reactions when contoured for use.
  • Nonceramic hydroxyapatite cement is based on calcium phosphate and requires mixing with water in order to form a paste before it is applied. This paste, however, is formed without an exothermic reaction. It is also resorbable and is replaced eventually with bone (though not 100% replacement many times, thereby potentially leading to a contour deformity).
  • Polytetrafluoroethylene (PTFE) has been used for subcutaneous augmentation (Gore-Tex), facial aesthetic surgery, vascular conduits, and even for facial implants (Proplast), although Proplast was recalled by the FDA secondary to reports of foreign body reaction and biomechanical failure. It is biologically inert and noncarcinogenic, and it also does not give off an exothermic reaction when used.
124
Q
  • An 8-year-old boy is brought to the office because of new-onset facial asymmetry that has slowly progressed over the past six months. The patient is otherwise healthy and has no other abnormalities. A current photograph is shown. Which of the following is the most likely diagnosis?
  • A ) Binder syndrome
  • B ) Craniofacial fibrous dysplasia
  • C ) Oculoauriculovertebral spectrum
  • D ) Romberg disease
  • E ) Treacher Collins syndrome
A
  • Romberg disease (progressive hemifacial atrophy, linear scleroderma, Parry-Romberg) is a progressive atrophy of one side of the face. Patients with Romberg disease often have a pigmented streak on the forehead, progressing to the”coupe de sabre” or facial depression. There is progressive atrophy of the skin, subcutaneous tissue, and bone. Many believe that it is a subset of localized or linear scleroderma.
  • Patients with Binder syndrome, or nasomaxillary or premaxillary hypoplasia, have low-set and flat nasal tip, a short, retracted columella, Class III occlusion, and a columella and upper lip that is”drawn into” the floor of the nostrils.
  • Fibrous dysplasia is a bony dysplasia, presenting as localized bony deformity (ground glass appearance on CT scan). Fibrous dysplasia may be limited to a localized area (monostotic) or may be generalized, affecting many sites (polyostotic), and is called McCune-Albright syndrome.
  • Goldenhar syndrome (oculoauriculovertebral spectrum) is a subset of hemifacial microsomia and, in addition to microsomia of the face, includes vertebral anomalies and/or epibulbar dermoids.
  • Treacher Collins syndrome is also known as mandibulofacial dysostosis. Patients with Treacher Collins have bilateral Tessier No. 6, 7, and 8 clefts and present with mandibular deformities, lower eyelid colobomas, auricular deformities, and hypoplastic zygomas.

125
Q
  • Patients with which of the following syndromes typically demonstrate hypernasality with speech?
  • A ) Crouzon
  • B ) Hemifacial microsomia
  • C ) Pfeiffer
  • D ) Saethre-Chotzen
  • E ) Velocardiofacial
A
  • Patients with velocardiofacial syndrome (VCF) typically demonstrate velopharyngeal insufficiency, which results in a hypernasal quality of speech. VCF is a result of a 22q11.2 deletion and has a wide spectrum of anomalies. The palate abnormality in VCF can vary from palatal weakness to a submucous or complete cleft of the palate. The characteristic facies are malar flattening, narrow palpebral fissures, small ears, vertical maxillary excess, and relative mandibular retrusion. The cardiovascular anomalies include ventricular septal defect, tetralogy of Fallot, and right-sided aortic arch. In planning surgery for velopharyngeal insufficiency, the most important finding is medial displacement of the carotid artery into the pharynx. Learning disabilities and psychiatric disorders are also seen in varying degrees in patients with VCF.
  • Crouzon, Pfeiffer, and Saethre-Chotzen syndromes all involve craniosynostosis, and patients do not have palatal issues resulting in velopharyngeal insufficiency. Hemifacial microsomia is the second-most common facial deformity after cleft lip and palate. There is a marked variability in presentation from mild to severe affecting one side of the face, including the ear, mandible, cranial nerve VII, and the soft tissue.
126
Q

A 14-year-old girl is brought to the office by her parents for consultation regarding facial asymmetry. On the basis of the photograph shown, which of the following is the most likely diagnosis?

(A) Congenital cranial (VII) nerve palsy

(B) Craniofacial microsomia

(C) Goldenhar syndrome

(D) Romberg disease

(E) Unilateral coronal craniosynostosis

A

The correct response is Option D.

The patient shown has Romberg disease (progressive hemifacial atrophy). This is a unilateral condition that affects girls more commonly than boys, and onset is in childhood with progression from 2 to 10 years of age. It usually involves skin, subcutaneous tissue, muscle, and bone but spares the cranial nerves and their function. Treatment is complex, with reconstruction of facial bone structure and augmentation followed by cutaneous contour correction with free tissue transfer. Parascapular flap is the flap of choice.

Unilateral coronal craniosynostosis can result in incomplete correction of vertical dystopia, secondary to continued growth constriction at the affected side cranial base. Soft-tissue loss and lower face asymmetry are not associated features. Hemifacial microsomia is a congenital, nonprogressive abnormality of the first branchial arch derivatives (ie, mandible and auricle). The mandibular ramus and condyle are variably hypoplastic, and overlying soft tissues (muscle, subcutaneous fat) are often hypoplastic. Ipsilateral macrostomia (transverse facial cleft) can also present in this syndrome. Goldenhar syndrome is a variant of craniofacial microsomia and is distinguished by presence of concomitant ocular abnormalities, including epibulbar dermoid.

127
Q

Which of the following facial anomalies is most common in patients with craniofacial microsomia?

(A) Facial nerve palsy

(B) Mandibular hypoplasia

(C) Maxillary hypoplasia

(D) Microtia

(E) Parotid gland hypoplasia

A

The correct response is Option B.

Craniofacial microsomia represents the second most common congenital anomaly affecting the head and neck behind cleft of the lip and/or palate. Multiple craniofacial and other anomalies are commonly seen as part of the constellation of findings. The most common anomalies are those affecting the mandible (89% to 100%) and ear (66% to 99%). Less common associations that must be sought involve the vertebrae and ribs (16% to 60%), ipsilateral facial nerve (10% to 45%), and genitourinary structures (4% to 15%), among others.

128
Q

In the United States, occurrence of encephaloceles is most common in which of the following anatomic regions?

(A) Frontoethmoidal

(B) Occipital

(C) Parietal

(D) Sphenoidal

A

The correct response is Option B.

In the United States, encephaloceles are most common in the occipital location. In Asia, they are more common in the frontoethmoidal region. The prognosis with these malformations depends upon the presence or absence of herniated brain tissue in the encephalocele.

129
Q

A 4-month-old boy is brought to the emergency department by his parents for evaluation after he had a seizure for the first time. Physical examination shows low nasal bridge and short neck. On laboratory studies, serum calcium level is 6.0 mg/dL (normal = 9.0 €“10.5 mg/dL) and serum phosphate level is 7.0 mg/dL (normal = 2.5 €“4.5 mg/dL). Which of the following is the most likely diagnosis?

(A) Albright syndrome

(B) Binder syndrome

(C) Carpenter syndrome

(D) Down syndrome

(E) Ectodermal dysplasia

A

The correct response is Option A.

Albright syndrome is the only syndrome listed that affects calcium and phosphate metabolism. It is caused by an autosomal-dominant mutation affecting receptor binding to adenylate cyclase. The resultant craniofacial malformations include a low nasal bridge and a short neck. Binder syndrome is manifested by maxillary hypoplasia and midface retrusion. Carpenter syndrome is a rare autosomal-recessive condition that results in brachycephaly due to variable synostoses, in addition to defects of the cardiovascular, musculoskeletal, and genital systems. Down syndrome is caused by trisomy of the 21st chromosome. Craniofacial manifestations include brachycephaly, a low nasal bridge, and inner epicanthal folds. Ectodermal dysplasia is an X €‘linked recessive syndrome that results in hypoplastic skin and sparse dermal appendages.

130
Q

An 8 €‘month-old girl is brought to the office for evaluation of the skull asymmetry shown. Physical examination shows posterior plagiocephaly with flattening of the right occipital area and an ipsilateral mastoid bulge. CT scan is shown. Which of the following is the most appropriate management for correction of this deformity?

(A) No treatment

(B) Modification of sleep position

(C) Molding helmet therapy

(D) Remodeling of the posterior vault

(E) Bilateral fronto €‘orbital advancement and remodeling

A

The correct response is Option D.

This infant has premature obliteration of her right lambdoid suture (lambdoid synostosis), resulting in posterior plagiocephaly. Lambdoid synostosis is treated with posterior vault expansion and remodeling. Without treatment, increased intracranial pressure can occur. Modification of the infant €™s sleep position and use of molding helmet therapy are appropriate treatments for plagiocephaly caused by deformation forces, but not for plagiocephaly caused by lambdoid synostosis. Bilateral fronto-orbital advancement and remodeling is not appropriate for managing lambdoid synostosis because the synostosis and skull asymmetry are of the posterior vault.

Management of posterior plagiocephaly requires differentiation of occipitoparietal flattening caused by lambdoid synostosis from that caused by deformation forces because deformational plagiocephaly does not require surgical correction. However, the two entities are often difficult to distinguish. Lambdoid synostosis causes a more trapezium €‘shaped skull with occipitoparietal flattening and an ipsilateral mastoid bulge. The ipsilateral ear is posteriorly positioned in relation to the other ear. Deformational plagiocephaly causes a parallelogram €‘shaped skull and displaces the ipsilateral ear anteriorly.

131
Q

A 7-year-old girl is brought to the office for consultation regarding the congenital malformation shown. Which of the following anatomic structures is absent in this patient?

(A) Anterior nasal spine

(B) Dentoalveolus

(C) Hard palate

(D) Maxilla

(E) Nasal bones

A

The correct response is Option A.

The anterior nasal spine is absent in patients with Binder syndrome. In 1962, Von Binder described a form of nasomaxillary hypoplasia that included nasomaxillary hypoplasia, a convex lip, a vertically short nose, a flat frontonasal angle, an absent anterior nasal spine, limited nasal mucosa, and hypoplastic frontal sinuses. It is believed to be caused by a disturbance of the prosencephalic induction center at a critical phase in development.

132
Q

A 6-month-old boy is referred by a pediatrician for evaluation of asymmetric shape of the skull. Physical examination shows flattening of the right posterior skull, prominence of the right side of the forehead and zygomatic complex, and forward advancement of the right ear. Which of the following is NOT an option in the management of this patient €™s condition?

(A) Observation

(B) Frequent repositioning exercises

(C) Helmet-molding therapy

(D) Three-dimensional CT scan of the head

(E) Surgical remodeling of the cranial vault

A

The correct response is Option E.

Surgical management of plagiocephaly is not indicated at this young age.

Since the €œback €‘to €‘sleep € campaign began recommending that young children be placed on their backs to sleep in an effort to limit the incidence of sudden infant death syndrome (SIDS), many children have been referred for evaluation of plagiocephaly. Physical examination continues to be the diagnostic method for separating plagiocephaly as a result of positional molding (ie, external pressures) from synostotic plagiocephaly.

Positional plagiocephaly results from external pressure on the skull €”in this case, the right posterior skull. As the brain grows, it pushes the ipsilateral structures forward, including the zygomatic complex, forehead, and ear. From a vertex view, the skull takes on the appearance of a parallelogram.

In contrast, when a growth suture is fused as in craniosynostosis, growth is restricted perpendicular to the suture and accelerated parallel to that suture (Virchow €™s Law). In cases of suspected lambdoid synostosis, the growth of the contralateral forehead is accelerated and the contralateral structures (such as the ear) are advanced. From the vertex, the skull becomes trapezoid in appearance. Similarly from a posterior view, there is ipsilateral mastoid bossing and the tilt.

A CT scan may be obtained for many reasons but is not necessary to establish the diagnosis in this patient. Physical examination should be adequate to diagnose positional plagiocephaly.

Treatment of positional plagiocephaly initially involves behavioral changes and frequent repositioning of the infant. Prone playtime activities are encouraged. Sometimes physical therapy is required. Custom helmet molding therapy is recommended for severe skull asymmetry or for patients with other developmental issues that would complicate behavioral modifications or repositioning techniques. In this child, poor muscle tone and slight developmental delay lead us to recommend helmet therapy. It is not unusual for the posterior fontanelle to be closed at this time.

133
Q
  • Which of the following findings does NOT support a clinical diagnosis of posterior deformational plagiocephaly?
  • A ) Anteriorly displaced ipsilateral ear
  • B ) Horizontal and level skull base
  • C ) Ipsilateral mastoid bulging
  • D ) Ipsilateral occipital flattening
  • E ) Parallelogram-shaped head
A
  • Posterior deformational plagiocephaly, resulting from a persistent head position during sleep, occurs frequently, is different from craniosynostosis, and is treated with either successful repositioning in sleep or cranial molding helmet therapy. Clinically, posterior deformational plagiocephaly presents with ipsilateral occipital head flattening, ipsilateral forehead displacement, ipsilateral ear forward displacement, and an overall parallelogram-shaped head when viewed from the top-down position. The skull base is horizontal, level, and unaffected in posterior deformational plagiocephaly.
  • Unilateral lambdoid craniosynostosis also presents with ipsilateral occipital flattening. Because the lambdoid suture is fused, compensatory growth, parallel to the fused suture, results in contralateral parietal bossing and ipsilateral mastoid bulging resulting in a canted skull base. This produces an overall trapezoid head shape when viewed from the top-down position.
134
Q

If the 13-year-old boy whose CT scan is shown has an epibulbar dermoid, the most appropriate classification of his condition is which of the following syndromes?

(A) Goldenhar

(B) Stickler

(C) Treacher Collins

(D) Van der Woude

(E) Velocardiofacial

A

The correct response is Option A.

The CT scan demonstrates a patient with the type IIb mandibular deformity of hemifacial or craniofacial microsomia. Patients with hemifacial microsomia with associated epibulbar dermoids and spinal or scapular deformities are classified as having Goldenhar syndrome.

Van der Woude syndrome is an autosomal-dominant cleft lip/palate syndrome. The defining feature in addition to cleft lip and/or palate is €œlip pits. € These lip pits are often found on the lower lip and are accessory salivary glands.

Velocardiofacial syndrome (previously known as DiGeorge syndrome, conotruncal anomaly, Sphrintzen syndrome, Catch-22 syndrome, and 22 minus syndrome) is a syndrome diagnosed by the addition or deletion of the 22q.11 gene. Common findings include congenital heart anomalies, clefting and velopharyngeal insufficiency, and facies that include a broad nasal root and narrow alar bases, elongated face with low tone, low-set ears with folded helixes, malar flattening, and retrognathia. It is estimated that up to 8% to 10% of patients with isolated cleft palate carry the 22q.11 deletion/addition.

Treacher Collins syndrome or mandibulofacial dysostosis has three pathognomonic features: lower eyelid colobomas, zygomatic hypoplasia with hypoplastic or absent zygomas and inferolateral orbital rim clefting, and bilateral mandibular hypoplasia. Bilateral ear anomalies ranging from simple deformities to complete microtia are often additional findings.

Stickler syndrome is associated with Pierre Robin sequence (micrognathia, glossoptosis, respiratory distress, and possibly cleft palate) and includes possible skeletal anomalies as well as optical conditions such as myopia.

135
Q

In the Tessier system, which of the following classifications represents the most common facial cleft?

(A) Tessier No. 0

(B) Tessier No. 3

(C) Tessier No. 7

(D) Tessier No. 9

(E) Tessier No. 14

A

The correct response is Option C.

The most common facial cleft is a Tessier No. 7. This is a cleft that begins at the lateral oral commissure and extends laterally. From a soft-tissue standpoint, it creates macrostomia. A Tessier No. 0 cleft involves the midline of the upper lip and nose. The Tessier No. 14 cleft is the cranial extension of this. A Tessier No. 3 cleft involves the lateral nasal ala and the medial canthus of the eye. A Tessier No. 9 cleft is actually the least common cleft. It extends from the superolateral orbit into the temporal region.

136
Q

Which of the following facial anomalies is most common in patients with craniofacial microsomia?

(A) Facial nerve palsy

(B) Mandibular hypoplasia

(C) Maxillary hypoplasia

(D) Microtia

(E) Parotid gland hypoplasia

A

The correct response is Option B.

Craniofacial microsomia represents the second most common congenital anomaly affecting the head and neck behind cleft of the lip and/or palate. Multiple craniofacial and other anomalies are commonly seen as part of the constellation of findings. The most common anomalies are those affecting the mandible (89% to 100%) and ear (66% to 99%). Less common associations that must be sought involve the vertebrae and ribs (16% to 60%), ipsilateral facial nerve (10% to 45%), and genitourinary structures (4% to 15%), among others.

137
Q

In the United States, occurrence of encephaloceles is most common in which of the following anatomic regions?

(A) Frontoethmoidal

(B) Occipital

(C) Parietal

(D) Sphenoidal

A

The correct response is Option B.

In the United States, encephaloceles are most common in the occipital location. In Asia, they are more common in the frontoethmoidal region. The prognosis with these malformations depends upon the presence or absence of herniated brain tissue in the encephalocele.

138
Q

In the Tessier system, which of the following classifications represents the most common facial cleft?

(A) Tessier No. 0

(B) Tessier No. 3

(C) Tessier No. 7

(D) Tessier No. 9

(E) Tessier No. 14

A

The correct response is Option C.

The most common facial cleft is a Tessier No. 7. This is a cleft that begins at the lateral oral commissure and extends laterally. From a soft-tissue standpoint, it creates macrostomia. A Tessier No. 0 cleft involves the midline of the upper lip and nose. The Tessier No. 14 cleft is the cranial extension of this. A Tessier No. 3 cleft involves the lateral nasal ala and the medial canthus of the eye. A Tessier No. 9 cleft is actually the least common cleft. It extends from the superolateral orbit into the temporal region.

139
Q

A 4-month-old boy is brought to the emergency department by his parents for evaluation after he had a seizure for the first time. Physical examination shows low nasal bridge and short neck. On laboratory studies, serum calcium level is 6.0 mg/dL (normal = 9.0 €“10.5 mg/dL) and serum phosphate level is 7.0 mg/dL (normal = 2.5 €“4.5 mg/dL). Which of the following is the most likely diagnosis?

(A) Albright syndrome

(B) Binder syndrome

(C) Carpenter syndrome

(D) Down syndrome

(E) Ectodermal dysplasia

A

The correct response is Option A.

Albright syndrome is the only syndrome listed that affects calcium and phosphate metabolism. It is caused by an autosomal-dominant mutation affecting receptor binding to adenylate cyclase. The resultant craniofacial malformations include a low nasal bridge and a short neck. Binder syndrome is manifested by maxillary hypoplasia and midface retrusion. Carpenter syndrome is a rare autosomal-recessive condition that results in brachycephaly due to variable synostoses, in addition to defects of the cardiovascular, musculoskeletal, and genital systems. Down syndrome is caused by trisomy of the 21st chromosome. Craniofacial manifestations include brachycephaly, a low nasal bridge, and inner epicanthal folds. Ectodermal dysplasia is an X €‘linked recessive syndrome that results in hypoplastic skin and sparse dermal appendages.

140
Q

The 2-year-old boy shown has features typical of which of the following syndromes?

(A) Apert

(B) Binder

(C) Crouzon

(D) Pfeiffer

(E) Treacher Collins

A

The correct response is Option E.

The appearance of the patient shown demonstrates Treacher Collins syndrome, characterized by bilaterally symmetrical abnormalities of structures within the first and second branchial arches. Features include a convex facial profile with a retrusive lower jaw and chin, anti €‘mongoloid slant of the palpebral fissures, lower lid colobomas, partial absence of eyelid cilia, absent or malformed external ears, hypoplasia of the malar bones, and variable cleft palate.

Binder syndrome is characterized by nasomaxillary hypoplasia. Apert, Crouzon, and Pfeiffer syndromes are all craniosynostotic syndromes that result in midfacial hypoplasia and a concave facial profile of varying degrees.

141
Q

A 4-month-old infant is evaluated because of a lesion near the lateral aspect of the left eyebrow. A photograph is shown. The mass is firm, well-circumscribed, and does not move with overlying skin movement. Which of the following preoperative imaging studies is most appropriate?

A) CT scan
B) MRI
C) Ultrasonography
D) X-ray studies
E) Imaging studies are not necessary

A

The correct response is Option E.

Dermoid cysts of the craniofacial region represent about 7% of all dermoid cysts and have an incidence of around 1 in 1000 live births. These masses can arise in multiple anatomic locations on the head, including the lateral eyebrow (sometimes referred to as an angular dermoid), midline nose, glabella, medial canthus, temporal or occipital scalp, frontal/anterior fontanel, and postauricular sulcus. Intracranial extension occurs in midline nasal dermoid cysts (13% below and 0% above the nasofrontal suture), frontal/fontanel (36.8%), temporal (16.2%), and occiput (16.3%), while this almost never occurs in the remaining locations (lateral brow, parietal, medial canthus, auricular). Cysts of the lateral brow represent almost two-thirds of all craniofacial dermoid cysts, and intracranially, extension or contact with the dura does not occur. Thus, the diagnosis is clinical, and further imaging in a child this young is not warranted. Overland et al. advocate imaging (CT or MRI) in older patients (>2 years of age) due to an increased risk for orbital extension with growth.

142
Q

A 6-month-old boy is referred by a pediatrician for evaluation of asymmetric shape of the skull. Physical examination shows flattening of the right posterior skull, prominence of the right side of the forehead and zygomatic complex, and forward advancement of the right ear. Which of the following is NOT an option in the management of this patient €™s condition?

(A) Observation

(B) Frequent repositioning exercises

(C) Helmet-molding therapy

(D) Three-dimensional CT scan of the head

(E) Surgical remodeling of the cranial vault

A

The correct response is Option E.

Surgical management of plagiocephaly is not indicated at this young age.

Since the €œback €‘to €‘sleep € campaign began recommending that young children be placed on their backs to sleep in an effort to limit the incidence of sudden infant death syndrome (SIDS), many children have been referred for evaluation of plagiocephaly. Physical examination continues to be the diagnostic method for separating plagiocephaly as a result of positional molding (ie, external pressures) from synostotic plagiocephaly.

Positional plagiocephaly results from external pressure on the skull €”in this case, the right posterior skull. As the brain grows, it pushes the ipsilateral structures forward, including the zygomatic complex, forehead, and ear. From a vertex view, the skull takes on the appearance of a parallelogram.

In contrast, when a growth suture is fused as in craniosynostosis, growth is restricted perpendicular to the suture and accelerated parallel to that suture (Virchow €™s Law). In cases of suspected lambdoid synostosis, the growth of the contralateral forehead is accelerated and the contralateral structures (such as the ear) are advanced. From the vertex, the skull becomes trapezoid in appearance. Similarly from a posterior view, there is ipsilateral mastoid bossing and the tilt.

A CT scan may be obtained for many reasons but is not necessary to establish the diagnosis in this patient. Physical examination should be adequate to diagnose positional plagiocephaly.

Treatment of positional plagiocephaly initially involves behavioral changes and frequent repositioning of the infant. Prone playtime activities are encouraged. Sometimes physical therapy is required. Custom helmet molding therapy is recommended for severe skull asymmetry or for patients with other developmental issues that would complicate behavioral modifications or repositioning techniques. In this child, poor muscle tone and slight developmental delay lead us to recommend helmet therapy. It is not unusual for the posterior fontanelle to be closed at this time.

143
Q

An 8 €‘month-old girl is brought to the office for evaluation of the skull asymmetry shown. Physical examination shows posterior plagiocephaly with flattening of the right occipital area and an ipsilateral mastoid bulge. CT scan is shown. Which of the following is the most appropriate management for correction of this deformity?

(A) No treatment

(B) Modification of sleep position

(C) Molding helmet therapy

(D) Remodeling of the posterior vault

(E) Bilateral fronto €‘orbital advancement and remodeling

A

The correct response is Option D.

This infant has premature obliteration of her right lambdoid suture (lambdoid synostosis), resulting in posterior plagiocephaly. Lambdoid synostosis is treated with posterior vault expansion and remodeling. Without treatment, increased intracranial pressure can occur. Modification of the infant €™s sleep position and use of molding helmet therapy are appropriate treatments for plagiocephaly caused by deformation forces, but not for plagiocephaly caused by lambdoid synostosis. Bilateral fronto-orbital advancement and remodeling is not appropriate for managing lambdoid synostosis because the synostosis and skull asymmetry are of the posterior vault.

Management of posterior plagiocephaly requires differentiation of occipitoparietal flattening caused by lambdoid synostosis from that caused by deformation forces because deformational plagiocephaly does not require surgical correction. However, the two entities are often difficult to distinguish. Lambdoid synostosis causes a more trapezium €‘shaped skull with occipitoparietal flattening and an ipsilateral mastoid bulge. The ipsilateral ear is posteriorly positioned in relation to the other ear. Deformational plagiocephaly causes a parallelogram €‘shaped skull and displaces the ipsilateral ear anteriorly.

144
Q

A 7-year-old girl is brought to the office for consultation regarding the congenital malformation shown. Which of the following anatomic structures is absent in this patient?

(A) Anterior nasal spine

(B) Dentoalveolus

(C) Hard palate

(D) Maxilla

(E) Nasal bones

A

The correct response is Option A.

The anterior nasal spine is absent in patients with Binder syndrome. In 1962, Von Binder described a form of nasomaxillary hypoplasia that included nasomaxillary hypoplasia, a convex lip, a vertically short nose, a flat frontonasal angle, an absent anterior nasal spine, limited nasal mucosa, and hypoplastic frontal sinuses. It is believed to be caused by a disturbance of the prosencephalic induction center at a critical phase in development.

145
Q

If the 13-year-old boy whose CT scan is shown has an epibulbar dermoid, the most appropriate classification of his condition is which of the following syndromes?

(A) Goldenhar

(B) Stickler

(C) Treacher Collins

(D) Van der Woude

(E) Velocardiofacial

A

The correct response is Option A.

The CT scan demonstrates a patient with the type IIb mandibular deformity of hemifacial or craniofacial microsomia. Patients with hemifacial microsomia with associated epibulbar dermoids and spinal or scapular deformities are classified as having Goldenhar syndrome.

Van der Woude syndrome is an autosomal-dominant cleft lip/palate syndrome. The defining feature in addition to cleft lip and/or palate is €œlip pits. € These lip pits are often found on the lower lip and are accessory salivary glands.

Velocardiofacial syndrome (previously known as DiGeorge syndrome, conotruncal anomaly, Sphrintzen syndrome, Catch-22 syndrome, and 22 minus syndrome) is a syndrome diagnosed by the addition or deletion of the 22q.11 gene. Common findings include congenital heart anomalies, clefting and velopharyngeal insufficiency, and facies that include a broad nasal root and narrow alar bases, elongated face with low tone, low-set ears with folded helixes, malar flattening, and retrognathia. It is estimated that up to 8% to 10% of patients with isolated cleft palate carry the 22q.11 deletion/addition.

Treacher Collins syndrome or mandibulofacial dysostosis has three pathognomonic features: lower eyelid colobomas, zygomatic hypoplasia with hypoplastic or absent zygomas and inferolateral orbital rim clefting, and bilateral mandibular hypoplasia. Bilateral ear anomalies ranging from simple deformities to complete microtia are often additional findings.

Stickler syndrome is associated with Pierre Robin sequence (micrognathia, glossoptosis, respiratory distress, and possibly cleft palate) and includes possible skeletal anomalies as well as optical conditions such as myopia.

146
Q

A 14-year-old girl is brought to the office by her parents for consultation regarding facial asymmetry. On the basis of the photograph shown, which of the following is the most likely diagnosis?

(A) Congenital cranial (VII) nerve palsy

(B) Craniofacial microsomia

(C) Goldenhar syndrome

(D) Romberg disease

(E) Unilateral coronal craniosynostosis

A

The correct response is Option D.

The patient shown has Romberg disease (progressive hemifacial atrophy). This is a unilateral condition that affects girls more commonly than boys, and onset is in childhood with progression from 2 to 10 years of age. It usually involves skin, subcutaneous tissue, muscle, and bone but spares the cranial nerves and their function. Treatment is complex, with reconstruction of facial bone structure and augmentation followed by cutaneous contour correction with free tissue transfer. Parascapular flap is the flap of choice.

Unilateral coronal craniosynostosis can result in incomplete correction of vertical dystopia, secondary to continued growth constriction at the affected side cranial base. Soft-tissue loss and lower face asymmetry are not associated features. Hemifacial microsomia is a congenital, nonprogressive abnormality of the first branchial arch derivatives (ie, mandible and auricle). The mandibular ramus and condyle are variably hypoplastic, and overlying soft tissues (muscle, subcutaneous fat) are often hypoplastic. Ipsilateral macrostomia (transverse facial cleft) can also present in this syndrome. Goldenhar syndrome is a variant of craniofacial microsomia and is distinguished by presence of concomitant ocular abnormalities, including epibulbar dermoid.

147
Q

An 8-year-old girl has bilateral coloboma and retraction of the lower eyelids, ptosis of the upper eyelids, inferior displacement of the lateral canthi, hypoplasia of the temporalis muscles, microtia, and abnormal hairline. Which of the following additional findings in this patient represents the main underlying characteristic of the full expression of Treacher Collins syndrome?
(A) Absence of the malar bone and zygomatic arch
(B) Bilateral Tessier cleft number 4
(C) Deformed mandibular condyle
(D) Premature fusion of the coronal sutures
(E) Tessier cleft numbers 0 and 14

A

The correct response is Option A.

According to Tessier, the main characteristic of the complete form of Treacher Collins syndrome is the absence of the malar bone and zygomatic arch. This absence is caused by combined Tessier clefts numbers 6, 7, and 8. These clefts result in severe dysplasia or even absence of the zygoma. The number 6 cleft produces the lower eyelid findings. The number 7 cleft results in absence of the zygomatic arch, hypoplasia of the temporalis muscles, microtia, and abnormal hairline. The number 8 cleft causes inferior displacement of the lateral canthi.

The other findings listed do not occur in Treacher Collins syndrome. Unlike in this child, Tessier cleft number 4 begins lateral to Cupid’s bow, passes onto the cheek, and curves onto the lower lid, disrupting the lower canaliculus. It can result in anophthalmia. Lack of formation of the mandibular ramus and condyle is typical in hemifacial microsomia. It produces asymmetric malformation, whereas Treacher Collins syndrome produces symmetric effects. Premature fusion of the coronal sutures results in acrocephaly or oxycephaly. It leads to extreme upward growth with reduced lateral and anteroposterior growth and occurs in Crouzon and Apert syndromes. Tessier cleft numbers 0 and 14 result in midline deformities, including hypertelorism and nasal deformities.

148
Q

The 2-year-old boy shown is scheduled to undergo cleft repair. Which of the following is the most appropriate Tessier classification of this cleft?
(A) Tessier No. 0
(B) Tessier No. 3
(C) Tessier Nos. 6, 7, and 8
(D) Tessier No. 7
(E) Tessier No. 14

A

The correct response is Option D.

The orbit, nose, and mouth are key landmarks through which craniofacial clefts cross. Tessier noted these landmarks and numbered craniofacial clefts from 0 to 14, with lower numbers (0 through 7) representing facial clefts and the higher numbers (8 through 14) representing their cranial extension. Tessier’s numeric system is purely a topographic map of the cleft fault line (see diagram below); it describes the axis of the cleft but does not specifically indicate which structures (soft tissue or bone) may be involved by the cleft. The combination of cranial and facial clefts usually totals 14. Cleft 0 is in the midline of the face, whereas cleft 14 is its cranial midline extension. Clefts 1, 2, and 3 begin at the Cupid’s bow, as does the common cleft lip. Treacher Collins syndrome is a combination of clefts 6, 7, and 8. The number 7 cleft is the most common of the craniofacial clefts and is more commonly referred to as hemifacial microsomia. The clinical expression of number 7 cleft is highly variable. In the photograph, this number 7 cleft is expressed only by macrostomia at the oral commissure.

149
Q

A 3-month-old infant has bifacial microsomia and mandibular hypoplasia (shown). Which of the following is the best rationale for performing tongue-lip adhesion in this patient?
(A) Improve maxillary dentition
(B) Optimize airway patency
(C) Promote mandibular growth
(D) Promote maxillary growth
(E) Protect mandibular dentition

A

The correct response is Option B.

Tongue-lip adhesion is performed in patients with retrognathia and glossoptosis to improve the airway. Pierre Robin sequence describes the clinical triad of microretrognathia, glossoptosis, and upper airway obstruction. Patients may present with airway obstruction, feeding difficulties, or both immediately after birth or during the following weeks of life. If left untreated, many of these infants experience failure to thrive, chronic hypoxemia, and cor pulmonale. Pierre Robin sequence may be nonsyndromic or associated with syndromes such as bifacial or hemifacial microsomia. Tongue-lip adhesion was first described as a treatment of upper airway obstruction in Pierre Robin sequence by Douglas in 1946. There has been a recent trend toward distraction osteogenesis of the infant mandible in patients who present with microretrognathia and airway obstruction. However, the long-term effects of distraction on the infant mandible have not been well studied. The efficacy of tongue-lip adhesion in improving airway obstruction with minimal sequelae has been documented and should be considered before distraction.

150
Q

The papilla of the parotid duct is most commonly located adjacent to which of the following maxillary teeth?
(A) First bicuspid
(B) Second bicuspid
(C) First molar
(D) Second molar
(E) Third molar

A

The correct response is Option D.

The papilla of the parotid is located most frequently adjacent to the maxillary second molar as it empties the secretions of the parotid gland via Stensen’s duct.

151
Q

A 12-week-old girl has a cleft of the soft palate, symptoms of respiratory obstruction when lying supine, and history of poor weight gain. Which of the following is the most likely diagnosis?
(A) Hemifacial microsomia
(B) Isolated cleft palate
(C) Pierre Robin sequence
(D) Van der Woude syndrome
(E) Velocardiofacial syndrome

A

The correct response is Option C.

The child described has Pierre Robin sequence, which is associated with the symptoms of micrognathia, glossoptosis, and respiratory distress. Children with this condition often have difficulty with feedings as well. Pierre Robin sequence is believed to result from the tongue coming between the palatal shelves in the developing fetus. These patients are managed conservatively with positioning and tube feedings if necessary. If, however, the airway cannot be protected with positioning, then surgical management includes lip-tongue adhesion, distraction osteogenesis, and if necessary, tracheostomy.

Hemifacial microsomia is manifest by underdevelopment of the middle and external ear, zygoma, maxilla, and facial muscles and often is associated with a Tessier No. 7 cleft (macrostomia).

Isolated cleft palate does not cause airway obstruction or micrognathia but can be associated with slow weight gain and difficulty feeding.

Van der Woude syndrome is associated with cleft lip and palate and lower lip pits from accessory salivary glands.

Velocardiofacial syndrome is the most common syndrome seen in association with cleft lip and palate. Other clinical manifestations associated with this syndrome include velopharyngeal insufficiency, facial and cardiac anomalies, and absence of the thymus and parathyroid glands.

152
Q

A 6-year-old boy is brought to the office by his parents for consultation regarding bilateral congenital facial palsy and syndactyly of the hands. He underwent surgical correction of strabismus five years ago. Which of the following conditions is the most likely cause of this patient=s symptoms?
(A) Apert syndrome
(B) Bell palsy
(C) Goldenhar syndrome
(D) Hemifacial microsomia
(E) Möbius syndrome

A

The correct response is Option E.

Möbius syndrome is a developmental disorder characterized by bilateral facial palsy and abducens nerve paralysis. Strabismus surgery is performed to correct paralysis of lateral gaze. Limb abnormalities, including clubfeet, syndactyly, and rudimentary fingers or toes, have been reported in 25% of cases. Additional cranial nerves (III, V, IX, XI, and XII) may be involved in Möbius syndrome, and some patients may present with congenital unilateral or partial facial paralysis. Hemifacial microsomia is a morphogenetic anomaly that can affect the skeletal, soft tissue, and neuromuscular structures derived from the first and second branchial arches. Typical cases have hypoplasia of the mandible that may be accompanied by hypoplasia of the zygoma and maxilla. Because the facial nerve is derived from the second branchial arch, patients with hemifacial microsomia can present with a congenital facial palsy. Goldenhar syndrome is hemifacial microsomia with epibulbar dermoids and vertebral anomalies. Apert syndrome is characterized by coronal craniosynostosis, syndactyly, and retardation. Bell palsy is a demyelinating inflammatory process of the facial nerve that classically presents as an acute unilateral facial paralysis and is believed to be caused by the herpes simplex virus.

153
Q

The harlequin deformity occurs most commonly in patients with which of the following craniosynostoses?
(A) Coronal
(B) Crouzon syndrome
(C) Lambdoidal
(D) Metopic
(E) Sagittal

A

The correct response is Option A.

The harlequin deformity is a radiographic appearance characterized by an oblique opacity extending from inferior and medial to superior and lateral through the orbital aperture. This is caused by superior displacement of the lesser wing of the sphenoid secondary to synostosis of the coronal suture on the side of the harlequin appearance.

154
Q

Which of the following craniofacial anomalies is associated with orbital hypotelorism?

(A) Crouzon syndrome
(B) Frontonasal dysplasia
(C) Frontonasal encephalocele
(D) Trigonocephaly

A

The correct response is Option D.

Hypotelorism, a decrease in intraorbital distance, occurs with trigonocephaly caused by metopic synostosis and holoprosencephaly. In patients with metopic synostosis, growth is inhibited perpendicular to the synostotic metopic suture, resulting in decreased interorbital distance as well as narrowing of the forehead. Holoprosencephaly results from a breakdown in the formation of prechordal mesoderm, leading to deficiencies of the structures located in the midline and narrowing of interorbital distance.

Hypertelorism, an increase in interorbital distance, is associated with numerous craniofacial disorders, including frontonasal encephalocele and median facial clefts occurring secondary to incomplete fusion of the medial orbital walls. Patients with more severe forms of craniosynostosis, such as Apert or Crouzon syndrome, may also have orbital hypertelorism. The hypertelorism seen in patients with frontonasal dysplasia occurs as a result of excess bone formation.

155
Q

In patients with Binder syndrome, the most likely physical finding is hypoplasia of which of the following structures?

(A) Anterior cranial base
(B) Anterior nasal floor
(C) Anterior wall of the maxilla
(D) Medial orbital wall
(E) Nasal septum

A

The correct response is Option B.

A patient with Binder syndrome, or maxillonasal dysplasia, typically has a shortened nose with flattening of the nasal bridge and perialar regions. The columella is shortened, the nasolabial angle is acute, and the upper lip is convex. The anterior nasal spine and frontonasal angle are absent. Occlusion is Angle class III. Binder syndrome is caused by hypoplasia of the anterior nasal floor (fossa praenasalis) and localized symmetric maxillary hypoplasia in the region of the alar rims. Nasal views show a retracted columellar-lip junction, a perpendicular alar-cheek junction, a convex upper nasal tip with a wide, shallow philtrum, crescent-shaped nostrils without a sill, a low-set and flat nasal tip, and a stretched and shallowed cupid’s bow. The triangular flair typically seen at the base of the nose is instead absent.

In patients with Binder syndrome, the primary goal of surgery is increasing the length of the nose and the projection of the nasal tip. This can be achieved by performing Le Fort I osteotomy, Le Fort II osteotomy, or a combination of both procedures, as well as compensatory orthodontic treatment. Autogenous bone and cartilage grafts may be required to reconstruct the nose.

156
Q

A 4-month-old infant has had a mass on the central nasal bridge (shown above) since birth. Physical examination shows a mass measuring 0.5 _ 1 cm that is soft and minimally mobile with a central pore. MRI of the head is shown above. Based on these clinical findings, which of the following is the most likely diagnosis?

(A) Dermoid cyst
(B) Encephalocele
(C) Glioma
(D) Lipoma
(E) Vascular malformation

A

The correct response is Option A.

This infant has a nasoglabellar dermoid cyst with an intracranial component, which requires transcranial and local excision.

In pediatric patients, orbitofacial dermoid cysts vary in presentation and location. They typically fall into three subgroups: brow region, orbital region, and nasoglabellar region. Most dermoid cysts are located in the lateral brow region. Typically, dermoid cysts in the brow and orbital regions do not have intracranial extension and can be excised locally. However, 10% to 45% of nasoglabellar dermoid cysts have sinus tracts and occasional intracranial extension. For dermoid cysts in this region, further studies with MRI or CT are recommended to exclude intracranial extension.

Recent reports have shown frontotemporal dermoid cysts with sinus tracts that have intracranial extension. If a sinus tract with extension is encountered during local excision, further radiographic evaluation is warranted. Frontotemporal dermoid cysts may represent a distinct entity from dermoid cysts in the brow region.

157
Q

A 1-month-old infant has right-sided microtia and hemifacial microsomia (shown). Weakness of which of the following branches of the facial nerve is demonstrated?
(A) Buccal
(B) Cervical
(C) Mandibular
(D) Temporal
(E) Zygomatic

A

The correct response is Option C.

The photograph of this patient demonstrates animation with symmetrical function of all branches of the facial nerve with the exception of the marginal mandibular branch on the right. The marginal mandibular branch is responsible for innervation of the lower lip depressor muscle, which is nonfunctional in this patient.

The second branchial cleft arch gives rise to the facial (VII) nerve. This nerve exits the skull at the stylomastoid foramen and contains motor and sensory fibers. It has six major branches: the temporal, zygomatic, buccal, mandibular, cervical, and auricular branches. The auricular branch separates before the facial nerve turns to the parotid gland. At the parotid gland, the facial nerve separates into two main divisions, the temporofacial and cervicofacial divisions, which further divide into the temporal, zygomatic, buccal, mandibular, and cervical branches.

Hemifacial microsomia produces anomalies in skeletal, soft tissue, and neuromuscular structures derived from the first and second branchial arches. It is the second most common congenital facial anomaly and is unilateral in 80% of affected patients. Usually, patients have some form of microtia and hypoplasia of the mandibular ramus. They may also have macrostomia, facial paralysis, and hypoplasia of the muscles of mastication.

158
Q

Mutation of the fibroblast growth factor receptor (FGFR) has been most commonly associated with which of the following single-suture synostoses?

(A) Lambdoid
(B) Metopic
(C) Sagittal
(D) Squamosal
(E) Unicoronal

A

The correct response is Option E.

Mutation of FGFR3, located at chromosome 4p16, has been found to cause unicoronal synostosis. This suggests a genetic basis for certain forms of synostotic frontal plagiocephaly. FGFRs regulate cell growth and bony proliferation. Mutations in FGFRs have been associated with syndromic craniosynostoses, such as in Pfeiffer, Crouzon, and Jackson-Weiss syndromes.

Evaluation of patients with unicoronal synostosis showed more severe cranial dysmorphology and a higher number of surgical revisions in those with FGFR3 mutation for facial dysmorphology compared with those without the mutation. This finding has led to genetic screening for all patients with unicoronal synostosis to better counsel patients and anticipate surgical outcomes.

159
Q

Which of the following findings is commonly caused by nonsyndromic unicoronal synostosis?

(A) Anterior displacement of the ipsilateral ear
(B) Deviation of the root of the nose to the contralateral side
(C) Flattening of the ipsilateral aspect of the occiput
(D) Occlusal cant up on the ipsilateral side
(E) Recession of the contralateral forehead

A

The correct response is Option A.

Nonsyndromic unicoronal synostosis commonly causes anterior displacement of the ipsilateral ear toward the affected suture. It also typically results in ipsilateral flattening and contralateral bossing of the forehead and deviation of the root of the nose to the ipsilateral side toward the affected suture. Unicoronal synostosis does not typically affect occlusion on either side and is not likely to cause significant change in occipital shape.

160
Q

A 2-month-old infant has a facial cleft extending from the upper lip through the nasal ala and into the medial canthal region. Which of the following is the most appropriate Tessier classification of this cleft?

(A) No. 1
(B) No. 2
(C) No. 3
(D) No. 4
(E) No. 5

A

The correct response is Option C.

As shown below, the most appropriate classification for this cleft is Tessier number 3 because this is the only classification in which the cleft involves the nasal ala and medial canthus. A cleft classified as number 0 involves the midline of the nose. A cleft classified as number 1 or 2 involves the nasal ala but is medial to the eye. A cleft classified as number 4 or 5 is lateral to the nose and typically involves the lower eyelid.

161
Q

In patients with Treacher Collins syndrome, which of the following is a characteristic skeletal finding?

(A) Brachycephaly
(B) Hypertelorism
(C) Macrogenia
(D) Malar hypoplasia
(E) Preaxial polysyndactyly

A

The correct response is Option D.

The characteristic skeletal finding in patients with Treacher Collins syndrome is hypoplasia of the malar bones, which often occurs in conjunction with clefting through the zygomatic arches. Patients also have hypoplasia of the maxilla and mandible and antegonial notching of the angle of the mandible. Occlusion is Angle class II; there is an anterior open bite and clockwise rotation of the occlusal plane. Effects on the temporomandibular joint are varied.

Brachycephaly, macrogenia, preaxial polysyndactyly, and hypertelorism do not occur in patients with Treacher Collins syndrome.

162
Q

According to Tessier’s classification, patients with which of the following types of clefts have displacement of the medial canthus of the eyelid?

(A) No. 1
(B) No. 3
(C) No. 5
(D) No. 7
(E) No. 9

A

The correct response is Option B.

The Tessier classification of craniofacial clefting, which was first proposed in 1973, integrates both tissue findings and underlying skeletal deformities. According to the Tessier system, displacement of the medial canthus of the eyelid is characteristic of the No. 3 cleft. This cleft begins in the alveolus, between the lateral incisor and canine, and extends through the maxilla and into the lacrimal bone. It is often referred to as a “naso-ocular” cleft because the inferomedial wall of the orbit is absent. Other associated soft-tissue deformities include shortening of the nose, colobomas of the nasal alae and the lower eyelids medial to the punctum, obstruction of the nasolacrimal duct, and malformations of the lower canaliculus. The lip deformity is similar to cleft lip.

The Tessier No. 1 cleft lies just lateral to the midline, beginning at the cupid’s bow and passing through the dome of the nostril lateral to the anterior nasal spine. Notching of the alar dome is a distinctive feature. The nasal bone may be absent, but the septum is unaffected. Hypertelorism and encephalocele may also be associated.

The No. 2 cleft is exceedingly rare and may be a transition between the No. 1 and No. 3 clefts.

The Tessier No. 4 cleft passes between the piriform aperture and the infraorbital foramen. It begins lateral to cupid’s bow and the philtrum and then passes lateral to the nasal ala and onto the cheek, terminating in the lower eyelid medial to the punctum. The medial canthal tendon is unaffected.

The rare Tessier No. 5 cleft begins behind the canine and extends through the maxillary sinus to the orbital floor. Colobomas of the lateral lower eyelids and clefting of the upper lip medial to the oral commissure are associated.
The No. 6 cleft is the incomplete form of Treacher Collins syndrome. This cleft passes inferior and lateral to the oral commissure, toward the angle of the mandible. Colobomas of the lateral lower eyelids occur.

The Tessier No. 7 cleft is the most common atypical cleft; it occurs in one of every 3000 births. It has sporadic transmission and highly variable expressivity. The zygomatic arch is typically absent.

The No. 8 cleft extends from the lateral commissure of the palpebral fissure to the temporal region and is also associated with colobomas of the lower eyelids.

The Tessier No. 9 cleft is merely a supraorbital extension of the No. 5 cleft.

163
Q

A 10-month-old boy with uncorrected left coronal synostosis is brought for evaluation by his parents who have noticed that the boy’s head is tilted to the right. On physical examination, the head can be straightened to a neutral position easily; there is no palpable mass or firmness within the sternocleidomastoid muscle.

Which of the following is the most likely cause?

(A) Muscular torticollis
(B) Paresis of the contralateral superior oblique muscle
(C) Paresis of the ipsilateral superior oblique muscle
(D) Paresis of the contralateral superior rectus muscle
(E) Paresis of the ipsilateral superior rectus muscle

A

The correct response is Option C.

The most likely cause of the findings in this patient is paresis of the ipsilateral superior oblique muscle. Unilateral coronal synostosis is characterized by foreshortening of the orbital roof on the affected side. As a result, some patients have strabismus occurring secondary to relative paresis of the superior oblique muscle. The child may tilt the head and thus elevate the eye to compensate for this problem; covering the affected eye is likely to resolve the head tilt.

Muscular torticollis is thought to be caused by abnormalities in the sternocleidomastoid muscle. When palpated, the muscle is firm or has a mass. Resistance to passive correction of the abnormal head position is associated. Although
torticollis is unlikely to occur simultaneously with unilateral coronal synostosis, any long-term head tilt, regardless of the cause, may lead to persistent foreshortening of the sternocleidomastoid muscle and ultimately result in secondary torticollis.

Abnormalities may be present within the other extraocular muscles but would not cause these findings.

164
Q

In mandibular distraction osteogenesis, the central region of the distraction gap is described as which of the following zones?

(A) Chondrocytic zone
(B) Fibrous zone
(C) Remodeling bone zone
(D) Transition zone of extending bone
(E) Zone of mature bone formation

A

The correct response is Option B.

In mandibular distraction osteogenesis, the central region of the distraction gap is referred to as the fibrous zone because it is characterized by fibrous tissue. The transitional zone, which lies adjacent to the fibrous zone, contains fibrous tissue undergoing ossification. This zone is surrounded by the zone of remodeling bone, which is itself surrounded by the zone of mature bone.

Chondrocytes are only present in mandibular distraction osteogenesis when there is excessive motion.

165
Q

In infants with hemifacial microsomia, which of the following deformities should be corrected initially?

(A) Abnormalities of the temporomandibular joint abnormalities
(B) Auricular deformities
(C) Deficiency of the mandibular ramus
(D) Deficiency of the mandibular body
(E) Macrostomia

A

The correct response is Option E.

Hemifacial microsomia is a condition associated with embryologic abnormalities involving the first and second branchial arches, including a Tessier No. 7 cleft. It is characterized by incomplete development of the external and middle ear, mandible, zygoma, maxilla, temporal bone, parotid gland, tongue, and the muscles of the palate and those that control mastication and facial expression. Affected patients have multiple clinical manifestations such as macrostomia, first branchial cleft sinus, and abnormalities of the cranial nerves.

Multiple procedures are necessary to reconstruct the face in patients with hemifacial microsomia. Macrostomia repair should be performed within the first few months after birth, similar to a typical cleft lip repair. Skeletal and soft-tissue repair is indicated at age 5 to 6 years. The mandibular deformities are typically corrected first, followed by the maxillary defects. Correction of soft-tissue deformities is generally delayed until skeletal reconstruction has been completed.

166
Q

Premature fusion of which of the following sutures is most often associated with abnormalities in the corpus callosum?

(A) Coronal
(B) Lambdoid
(C) Metopic
(D) Squamosal
(E) Sagittal

A

The correct response is Option C.

Premature fusion of the metopic suture, or trigonocephaly, is most often associated with abnormalities of the corpus callosum or other neurologic structures in the midline of the skull. Agenesis of the corpus callosum is a common abnormality. Children with trigonocephaly also have a higher than expected incidence of developmental delay.

Premature fusion of the coronal, lambdoid, or sagittal sutures is less likely to be associated with underlying neurologic abnormalities. The squamosal suture does not typically fuse prematurely.

167
Q

Mutations in the genetic loci for the fibroblast growth factor receptors (FGFR) have been shown to occur with greater frequency in patients with synostosis of which of the following sutures?

(A) Coronal
(B) Lambdoid
(C) Metopic
(D) Sagittal
(E) Squamosal

A

The correct response is Option A.

Mutations in the genetic locus for fibroblast growth factor receptor 3 (FGFR-3) have been found, in a greater than expected incidence, in both unilateral and bilateral coronal synostoses. Although these craniofacial malformations are thought to occur sporadically, this genetic link may explain the familial occurrence of coronal synostosis in some patients.

According to the results of recent studies, the craniofacial dysmorphology was more severe in patients who had the FGFR-3 mutation than in patients who did not carry the mutation, and outcomes of reconstructive surgery were poorer.

168
Q

Which of the following craniofacial anomalies does NOT demonstrate genetic transmission?

(A) Acrocephalosyndactyly
(B) Apert syndrome
(C) Craniofacial microsomia
(D) Mandibulofacial dysostosis
(E) Nager syndrome

A

The correct response is Option C.

Craniofacial microsomia is the most common major craniofacial anomaly and does not demonstrate genetic transmission. Instead, it is believed to be caused by an intrauterine event, such as occlusion of the stapedial artery or development of hematoma. Craniofacial microsomia is far more likely to be unilateral (hemifacial) than bilateral. It manifests as facial paralysis and malformations of the mandibular ramus. Growth of the mandibular condyle is impaired, resulting in inadequate development of the mandible and the craniofacial osseous complex on the affected side.

Both Apert and Crouzon syndromes, which are syndromes of acrocephalosyndactyly, are inherited as autosomal dominant conditions. Their genetic mutations have been linked to fibroblast growth factor receptor-2 (FGFR-2). These syndromes are characterized by craniosynostosis, exorbitism, and midface retrusion; however, each is differentiated by its associated extremity findings. Patients with Apert syndrome have severe syndactyly of the middle three digits of the hands and feet, often with a common nail. In Crouzon syndrome, the extremities are normal.

Mandibulofacial dysostosis, also known as Treacher Collins syndrome, is an autosomal dominant disorder; its gene has been mapped to chromosome 5q31.3-q33.3. Patients with this condition have facial clefting, an antimongoloid slant to the palpebral fissures, colobomas of the lower eyelids, absence of eyelashes on the medial portion of the lower eyelid, preauricular displacement of hair, malar and mandibular defects, and micrognathia.

Nager syndrome is an autosomal recessive disorder that has facial characteristics similar to Treacher Collins syndrome. Affected patients also have hypoplasia of the thumbs, metacarpals, and radius.

169
Q

In the patient who has the findings shown on the three-dimensional CT scans above, which of the following is the most likely diagnosis?

(A) Craniofacial microsomia
(B) Crouzon syndrome
(C) Goldenhar syndrome
(D) Romberg syndrome
(E) Treacher Collins syndrome

A

The correct response is Option E.

These findings are consistent with Treacher Collins syndrome, also known as mandibulofacial dysostosis or Franceschetti-Klein syndrome. The skeletal deformity seen in the CT scan represents bilateral Tessier No. 6, 7, and 8 clefts. Varying hypoplasia of the zygoma is a differentiating feature of this condition. Occlusion is Angle class II, with an anterior open bite.

There are numerous characteristic findings in patients with Treacher Collins syndrome. The orbit is oval or egg-shaped and has a superomedial base and inferolaterally oriented axis. A cleft in the inferolateral orbital floor allows for herniation of the orbital contents into the cheek. The inferior orbital rim, lateral orbital wall, and orbital floor are underdeveloped or absent. In contrast, the maxilla protrudes, and the palatal plane is rotated counterclockwise or upward and posteriorly. There is micrognathia and decreased length of the mandibular ramus and body, with broad curvature of the inferior aspect of the mandibular body. The condyle is hypoplastic or absent. The gonial angle is obtuse with antegonial notching. The chin is dysplastic, vertically long, and retrusive.

Craniofacial microsomia is typically hemifacial but can occur bilaterally. Hemifacial microsomia is associated with a Tessier No. 7 cleft. There are varying degrees of mandibular hypoplasia. The forehead, orbit, and zygoma may also be hypoplastic. Anophthalmia and microtia occur in the most severe cases. Clefting of the lateral orbit and antegonial notching are not characteristic features.

Crouzon syndrome is a syndrome of bilateral coronal synostosis or acrocephalosyndactyly. Affected patients have retrusion of the forehead and superior orbital rim, proptosis caused by shallow orbits, and midface hypoplasia. Occlusion is Angle class III.

Goldenhar syndrome is a subset of hemifacial microsomia and is characterized by epibulbar dermoids and anomalies of the scapula and/or spine.

Romberg syndrome, or progressive hemifacial atrophy, appears initially as cutaneous pigmentation in otherwise healthy patients but progresses to destruction of the facial soft tissues and skeleton. This is a hemifacial condition that varies in severity.

170
Q

A 7-year-old boy has hypernasality and velopharyngeal incompetence. He underwent repair of a ventricular septal defect at birth and repair of a cleft palate at age 9 months. His mother says that he has had difficulties with language learning. Physical examination shows upward slanting of the palpebral fissures, a broad nasal root, a small mouth, and a thin upper lip.

Which of the following studies is most likely to lead to a diagnosis in this patient?

(A) Measurement of serum creatine kinase level
(B) Chromosomal karyotyping
(C) Fluorescent in situ hybridization (FISH) analysis
(D) MR angiography

A

The correct response is Option C.

This 7-year-old boy has findings consistent with velocardiofacial syndrome, or Shprintzen syndrome, an autosomal dominant disorder with variable expressivity. Affected patients characteristically have velopharyngeal insufficiency, developmental delay, and facial abnormalities, including upward slanting of the palpebral fissures and a prominent nose with a broad nasal root and narrow alar base. Velocardiofacial syndrome has been diagnosed in as many as 8% of children with clefts of the secondary palate; however, cleft palate is not always seen in children with velocardiofacial syndrome. Cardiac anomalies, such as aberrant carotid arteries, are present in most patients, and the risk for arterial bleeding is increased during pharyngeal flap repair.

Because velocardiofacial syndrome is associated with a deletion on the long arm of chromosome 22q11.2, fluorescent in situ hybridization (FISH) analysis can be used to test any child with suspected velocardiofacial syndrome for chromosomal deletions. Although routine chromosomal karyotyping will not detect the small deletion associated with velocardiofacial syndrome, fluorescent-tagged DNA probes can be used in conjunction with routine cytogenetic examination. On examination with a fluorescent microscope, a child with a normal chromosomal pattern will have two signals (one on each chromosome), whereas only one signal will be present in the child with velocardiofacial syndrome.

Serum creatine kinase levels are increased in patients with muscular dystrophy. Although MR angiography will show aberrant blood vessels in patients with velocardiofacial syndrome, it is not diagnostic.

171
Q

A 3-month-old infant is being evaluated because his parents are concerned that his head is abnormally shaped. Examination shows scaphocephaly and occipital bossing. Which of the following is the most appropriate management for correction of the deformity?

(A) Modification of sleep positioning
(B) Molding helmet therapy
(C) Monobloc advancement
(D) Cranial vault remodeling with barrel staving
(E) Bilateral frontal craniotomy and bilateral fronto-orbital advancement

A

The correct response is Option D.

This 3-month-old infant with scaphocephaly should undergo cranial vault remodeling with barrel staving. Scaphocephaly involves premature closure of the sagittal suture. Affected patients have a long, narrow, keel-shaped skull. Frontal and occipital bossing may also be seen. Cranial vault remodeling with a barrel stave technique will help increase the width of the skull. Although sagittal strip craniectomy has been shown to produce improvement in infants with mild forms of sagittal synostosis, it cannot be used alone to widen the skull or correct frontal bossing.

Modification of the infant’s sleep position and/or use of a molding helmet are only effective in patients with open head sutures and will do nothing to change head shape in this infant with a prematurely closed sagittal suture. These techniques are used instead in infants with deformational (ie, nonsynostotic) plagiocephaly.

Monobloc advancement creates osteotomy lines similar to those produced by a Le Fort III procedure but does not osteotomize the nasofrontal junction and frontozygomatic suture. This technique allows for correction of supraorbital and midface deformities simultaneously but is associated with high rates of infection and cerebrospinal fluid leakage. These complications are thought to occur as a result of direct communication between the cranial and nasal cavities.

Bilateral frontal craniotomy and bilateral fronto-orbital advancement are effective for correction of unicoronal and bicoronal synostosis.

172
Q

A neonate who has a large, horseshoe-shaped cleft involving the hard and soft palates. He has difficulty breathing when placed in the supine position; his chest wall retracts and he grunts. Which of the following is the most likely diagnosis?

(A) Binder syndrome
(B) Klippel-Feil anomaly
(C) Pierre Robin sequence
(D) van der Woude syndrome
(E) Velocardiofacial syndrome

A

The correct response is Option C.

This neonate has the triad of symptoms associated with Pierre Robin sequence Ñ micrognathia, glossoptosis, and respiratory distress. Although cleft palate is often associated, it does not have to be present in order to make this diagnosis. Pierre Robin sequence is thought to result from forces that prevent the movement of the fetal tongue from between the vertically oriented palatal shelves, resulting in incomplete closure of the palatal shelves and ultimately cleft palate.

Appropriate management of neonates with cleft palate is attempted prone positioning to clear the airway and relieve the respiratory distress. The child should be evaluated for any anomalies of the lower airway that may be contributing to this problem. Secondary measures for airway control include tongue-lip adhesion and/or mandibular distraction osteogenesis. Tracheostomy is associated with high rates of morbidity and mortality in neonates and should only be performed if absolutely necessary.

Patients with Binder syndrome have hypoplasia of the nasomaxillary complex, with a low-set and flattened nasal tip, a short retracted columella, and absence of the anterior nasal spine. The columella and upper lip appear to be drawn into the floor of the nostrils. Angle class III malocclusion is characteristic.

Klippel-Feil anomaly manifests as a short neck, a low posterior hairline, deformities of the cervical spine, facial abnormalities, and hearing loss. Cleft palate is often associated.

Velocardiofacial syndrome, or Shprintzen syndrome, is associated with a deletion in chromosome 22q. It the most common syndrome seen in association with cleft lip and palate. Other findings include velopharyngeal dysfunction, facial abnormalities, cardiac anomalies, and absence of the thymus and parathyroid glands.

Patients with van der Woude syndrome have cleft lip and palate and lower lip pits resulting from the presence of accessory salivary glands. Anomalies of the extremities and genitalia have been reported.

173
Q

A 4-month-old child has an abnormal head shape. Physical examination shows anterior displacement of the right ear and zygoma and the right side of the forehead; there is abnormal flattening of the right side of the occipital skull. The skull shape appears similar to a parallelogram. There is no bulging of the mastoid or ridging of the sutures.

These findings are most consistent with which of the following?

(A) Bilateral posterior deformational plagiocephaly
(B) Left-sided posterior deformational plagiocephaly
(C) Left-sided posterior lambdoidal craniosynostosis
(D) Right-sided posterior deformational plagiocephaly
(E) Right-sided posterior lambdoidal craniosynostosis

A

The correct response is Option D.

Plagiocephaly, or abnormal head shape, can be characterized as synostotic (resulting from craniosynostosis) or nonsynostotic (resulting from deformation or molding of the skull). Unilateral coronal and lambdoidal craniosynostosis are the most common causes of synostotic plagiocephaly. In contrast, deformational plagiocephaly, or skull molding, typically results from placing the infant in a fixed supine position for sleep. Posterior deformational plagiocephaly now occurs in one out of 70 infants. The increase in the incidence of this condition is due to recent recommendations by the American Academy of Pediatrics that infants be placed in the supine position during sleep to decrease the incidence of sudden infant death syndrome (SIDS).

Most infants with posterior deformational plagiocephaly have unilateral findings, including flattening of the occiput associated with anterior displacement of the ipsilateral ear. In severe forms of deformational plagiocephaly, there may be anterior displacement of the ipsilateral forehead and zygoma and widening of the ipsilateral palpebral fissure, resulting in a parallelogram-shaped cranium. If the condition is diagnosed early, treatment involves repeatedly repositioning the child out of the “flat spot.” However, cranial molding helmets may be required for those infants who have severe deformational plagiocephaly, who are resistant to repositioning, or in whom the condition is diagnosed late.

Infants with bilateral posterior deformational plagiocephaly have flattening of the occiput, bulging of the mastoid bilaterally, and bossing of the biparietal eminence. The suture is not ridged, and the ears are essentially symmetric.

Lambdoidal craniosynostosis is characterized by ipsilateral flattening of the occiput in combination with ridging of the fused lambdoid suture. There is compensatory bulging of the contralateral parietal skull and bulging of the ipsilateral mastoid skull, resulting in an inferior, not anterior, displacement of the ipsilateral ear. This gives the cranium a trapezoid-like shape. Appropriate management is craniofacial surgery with cranial vault remodeling at approximately 6 to 9 months of age.

174
Q

For each patient or group of patients, select the most likely diagnosis (A-E).

(A) Apert syndrome
(B) Carpenter syndrome
(C) Crouzon syndrome
(D) Nager syndrome
(E) Pfeiffer syndrome

An infant has brachycephaly, hypertelorism, a bregmatic “bump,” and bony syndactyly of the fingers. A photograph is shown above.

Three members of a family have exorbitism, proptosis, retromaxillism, and recession of the frontal bone. There are no extremity abnormalities. A photograph is shown above.

A

The correct response for Item 49 is Option A and for Item 50 is Option C.

The infant has findings consistent with Apert syndrome, which is characterized by brachycephaly, hypertelorism with flattening of the face, strabismus, palsy of the ocular muscles, an antimongoloid slant of the palpebral fissures, and maxillary hypoplasia. Affected patients typically have a prominent bregmatic eminence or “bump.” There is bony syndactyly with complete fusion of the four fingers; the thumb is unaffected. Cutaneous syndactyly of the toes may be simple or complex. Although occurrence is typically sporadic, new mutations with autosomal dominant inheritance have been detected.

The affected family most likely has Crouzon syndrome, characterized by exorbitism, retromaxillism, inframaxillism, and paradoxical retrogenia. Inheritance is autosomal dominant, and occurrence is both sporadic and familial. Affected patients typically have recession of the frontal bone and supraorbital rim, midface retrusion, exorbitism with proptosis, and hypoplasia of the infraorbital rim. Hypertelorism, a bregmatic “bump,” and abnormalities of the hands are not characteristic.

Patients with Carpenter syndrome, or acrocephalopolysyndactyly, have craniosynostosis, shortened fingers, soft-tissue syndactyly, preaxial polydactyly, congenital cardiac disease, hypogenitalism, obesity, and umbilical herniation. The inheritance pattern of this syndrome is autosomal recessive.

Nager syndrome, or acrofacial dysostosis, is an autosomal recessive condition. Affected patients typically have short stature, cleft palate, and hypoplasia of the orbits, zygoma, maxilla, and mandible. Preaxial reduction defects occur in the upper, and sometimes lower, limbs. Hypoplasia or agenesis of the thumbs, radius, and one or more metacarpals is also characteristic.

Pfeiffer syndrome is an autosomal dominant disorder characterized by variable forms of craniosynostosis, acrocephalosyndactyly with broad thumbs and great toes, and severe midface hypoplasia.

175
Q

A 15-year-old girl with Apert syndrome has the deformity shown in the photograph and radiograph above. She underwent bifrontal craniotomy with bilateral supraorbital bar advancement at age 9 months. On examination, 25 mm of midface advancement is needed. Which of the following operative procedures is most appropriate?

(A) Monobloc advancement
(B) Le Fort I osteotomy with immediate advancement with bone grafting
(C) Le Fort I osteotomy with advancement by distraction osteogenesis
(D) Le Fort III osteotomy with immediate advancement with bone grafting
(E) Le Fort III osteotomy with advancement by distraction osteogenesis

A

The correct response is Option E.

In this 15-year-old girl with Apert syndrome who requires 25 mm of midface advancement, the most appropriate management is Le Fort III osteotomy with distraction osteogenesis. Although the forehead and superior orbital bar are positioned appropriately, there is deficiency and retrusion of the midface and malocclusion. Le Fort III osteotomy will correct the midface deficiency and the skeletal portion of the malocclusion. Distraction osteogenesis is preferred over bone grafting for advancement of more than 10 mm because it produces a more stable result that is less prone to relapse and allows for more gradual stretching of the soft-tissue envelope. Distraction osteogenesis is also associated with a lower rate of morbidity and less need for secondary midface procedures when compared with bone grafting.

Monobloc advancement will correct the midface retrusion and malocclusion but will also advance the forehead.

Le Fort I osteotomy and advancement with either bone grafting or distraction osteogenesis will not correct the midface deficiency.

176
Q

A 30-year-old woman has nasal obstruction on inspiration two years after undergoing rhinoplasty. On intranasal examination, the angle of the internal nasal valve is less than 10 degrees. Which of the following is the most appropriate surgical procedure for correction of this deformity?

(A) Auricular composite grafting
(B) Lateral crural strut grafting
(C) Use of spreader grafts
(D) Transposition of an alar base flap
(E) Submucous resection

A

The correct response is Option C.

If the dorsal hump is resected without adequately moving the medial upper lateral cartilage and mucosa away from the septum, a patient undergoing rhinoplasty is at risk for injury and destabilization of the internal nasal valve with the potential for subsequent collapse. The internal nasal valve angle typically measures 10 to 15 degrees. If the internal nasal valve angle is less than 10 degrees, spreader grafts should be inserted between the upper lateral cartilage and septum.

An alar base flap is appropriate for correction of vestibular stenosis accompanied by malpositioning of the alar base. This deformity typically results from excessive resection of the alar base and often occurs in patients undergoing secondary rhinoplasty procedures.

Patterned composite grafts of auricular cartilage are used in patients who have external nasal valve deformities that involve cartilage and vestibular skin.

Lateral crural strut grafts are indicated for correction of alar rim collapse and concavity or malpositioning of the lateral crura.

Because preservation of cartilage is an important consideration in any patient undergoing a secondary procedure, submucous resection should only be performed in patients who have airway obstruction caused by septal deviation.

177
Q

According to Tessier’s classification, which of the following is the most common craniofacial cleft?

(A) No. 0
(B) No. 3
(C) No. 4
(D) No. 6
(E) No. 7

A

The correct response is Option E.

The Tessier classification of craniofacial clefting was first proposed in 1973. This system integrates both tissue findings and underlying skeletal deformities; embryopathogenesis is not considered. Clefts No. 0 through 7 are located in the lower half of the face, while Nos. 9 through 14 occur in the upper hemisphere. According to the Tessier system, the No. 7 cleft is most common. This sporadic cleft, which has variable expressivity, is most likely to be seen in male neonates and occurs in one of every 3000 neonates. Macrostomia and absence of the zygomatic arch are typically associated.

Tessier No. 3 cleft involves the orbit. The cleft through the lip is located in the same position as a midline unilateral cleft lip. In the nasal area, the cleft changes its course and undermines the ala. The medial canthus is displaced inferiorly. Colobomas of the lower eyelid are medial to the punctum. The osseous component passes through the alveolus between the lateral incisor and canine. The cleft disrupts the lateral border of the piriform aperture.

Tessier No. 4 cleft passes lateral to the cupid’s bow and philtrum. In most affected patients, the cleft is located lateral to the nasolacrimal canal and sac. Like the Tessier No. 3 cleft, the osseous component is located between the lateral incisor and canine. However, unlike the No. 3 cleft, the No. 4 cleft spares the piriform aperture and courses medial to the intraorbital foramen.

Tessier No. 6 cleft involves an incomplete form of Treacher Collins syndrome. The cleft is directed inferior and lateral to the oral commissure toward the angle of the mandible. Colobomas of the lateral lower eyelids are seen.

178
Q

A 25-year-old woman has facial asymmetry. She says that she has had progressive loss of soft-tissue volume on the right side of the face since age 10 years that became stabilized four years ago. Examination shows significant subcutaneous atrophy of the right side of the face without bony asymmetry. She also has hypopigmentation of the iris on the affected side.

Which of the following is the most appropriate management?

(A) Bone graft augmentation of the midface
(B) Alloplastic augmentation
(C) Reconstruction with a microvascular serratus anterior free flap
(D) Reconstruction with a microvascular parascapular free flap
(E) Reconstruction with a superficial temporal fascia flap

A

The correct response is Option D.

This 25-year-old woman has Romberg’s hemifacial atrophy characterized by progressive unilateral loss of facial soft tissue. The underlying skeleton is also affected in patients with severe forms of the disease. Surgery should be delayed until the condition becomes stabilized, which is indicated by the cessation of facial atrophy. When this has occurred, a microvascular parascapular flap can be deepithelialized and customized to fit the dimensions of the defect, and then transferred and buried subcutaneously.

Skeletal augmentation with either bone graft or alloplast is not appropriate because the bones of the face are not affected. The serratus anterior flap would only atrophy over time, and the superficial temporal fascia flap would not provide the necessary volume.

179
Q

Which of the following substances has been shown to be associated with the mechanisms of cranial suture fusion in animal models?

(A) Epidermal growth factor (EGF)
(B) Interleukin-6 (IL-6)
(C) Prostaglandin-E2 (PGE2)
(D) Transforming growth factor-beta (TGF-B)
(E) Tumor necrosis factor-alpha (TNF-B)

A

The correct response is Option D.

Although the mechanisms of action resulting in premature fusion of cranial sutures, or craniosynostosis, are unknown, experimental animal studies have shown that transforming growth factor-beta (TGF-B) plays a role in the fusion of posterior frontal sutures. According to immunolocalization techniques, certain isoforms of TGF-B are expressed during fusion of posterior frontal sutures in rat and mouse models; increased immunoreactivity of isoforms of both TGF-B and insulin-like growth factor (IGF) has also been shown to occur during premature fusion of the sagittal, coronal, and lambdoid sutures in humans. Qualitative analysis of TGF-B protein levels in organ culture models has also demonstrated increased levels of TGF-B during the period of active suture fusion.

Although basic fibroblast growth factor (b-FGF) has also been implicated in the process of cranial suture fusion, epidermal growth factor (EGF) has not been shown to be associated with suture fusion. Interleukin-6 (IL-6), prostaglandin-E2 (PGE2), and tumor necrosis factor-alpha (TNF-B) have been shown to mediate inflammatory responses but not to affect cranial suture fusion.

180
Q

A 5-year-old boy has marked malar hypoplasia, a class II anterior open bite, and clockwise rotation of the occlusal plane. There is hypoplasia of the thumbs bilaterally. Which of the following is the most likely diagnosis?

(A) Bilateral craniofacial microsomia
(B) Goldenhar syndrome
(C) Klippel-Feil syndrome
(D) Nager syndrome
(E) Treacher Collins syndrome

A

The correct response is Option D.

This child has findings most consistent with Nager syndrome, or acrofacial dysostosis, an autosomal recessive disorder characterized by hypoplasia of the orbits, zygoma, maxilla, mandible, and soft palate. Although the findings are similar to Treacher Collins syndrome (an autosomal dominant disorder), patients with Nager syndrome can also exhibit preaxial limb anomalies and mental retardation. Auricular defects may be present.

Patients with bilateral craniofacial microsomia typically have hypoplasia of the mandibular ramus and varying degrees of auricular hypoplasia. Patients with Goldenhar syndrome have hemifacial microsomia and epibulbar dermoids.
Klippel-Feil syndrome is characterized by significant fusion of the cervical spine with varying involvement of the thoracic and lumbar spinal regions. Affected patients have limited cervical motion and a low posterior hairline.

181
Q

The anterior fontanelle typically closes completely at how many months of age?

(A) 3
(B) 9
(C) 12
(D) 24
(E) 36

A

The correct response is Option D.

The fontanelles are nonossified membranous intervals in the skull located at the angles of the parietal bones in infants. The anterior and posterior fontanelles are located in the midline, and the anterolateral (sphenoid) and posterolateral (mastoid) fontanelles are found laterally.

The anterior fontanelle is the largest, measuring approximately 4 cm anteroposteriorly and 2.5 cm transversely. It is located at the junction of the sagittal, coronal, and frontal sutures. The posterior fontanelle is triangular in shape and is found at the junction of the sagittal and lambdoid sutures. The sphenoid and mastoid fontanelles are irregular, small fontanelles that correspond to the sphenoid and mastoid angles of the parietal bones, respectively. Although the posterior fontanelle typically closes by age 2 months, the anterior fontanelle does not completely close until between the ages of 18 and 24 months.

182
Q

Which of the following is the most common craniofacial anomaly?

(A) Bilateral craniofacial microsomia
(B) Goldenhar syndrome
(C) Hemifacial microsomia
(D) Nager syndrome
(E) Treacher Collins syndrome

A

The correct response is Option C.

Hemifacial microsomia is the most common craniofacial anomaly; affected patients have unilateral malformations of the mandibular ramus and facial paralysis. Mandibular growth is impaired, leading to inadequate development of the mandibular and osseous complex on the involved side.

Bilateral craniofacial microsomia is less frequent than hemifacial microsomia.

Goldenhar syndrome, a variant of hemifacial microsomia, is characterized by vertebral or hemivertebral anomalies, lipodermoids, and epibulbar dermoids. The facial abnormalities resemble a Tessier No. 7 cleft. This condition may be bilateral.

Nager syndrome is an autosomal recessive disorder that manifests as hypoplasia of the orbits, zygoma, maxilla, mandible, and soft palate, as well as hypoplasia or agenesis of the radius, thumbs, and metacarpals. Auricular defects may also be associated.

Treacher Collins syndrome is an autosomal dominant disorder that involves hypoplasia of the zygoma, maxilla, and mandible. The palpebral fissures have an antimongoloid slant and the lashes of the medial lower eyelids are absent. Colobomas of the lower eyelids and malar defects are also associated. The preauricular hair may be displaced, and micrognathia may be present. Findings are similar to the Tessier No. 6, No. 7, and No. 8 clefts.

183
Q

Scaphocephaly is associated with which of the following suture synostoses?

(A) Bilateral coronal
(B) Lambdoid
(C) Metopic
(D) Sagittal
(E) Unilateral coronal

A

The correct response is Option D.

Scaphocephaly is just one form of craniosynostosis, or varying deformities of the cranial vault resulting from restrictions in development. Detectable craniosynostotic abnormalities occur in one of approximately 1800 neonates. The coronal, lambdoid, metopic, and sagittal sutures are primarily involved; minor sutures can include the frontonasal, frontosphenoidal, and temporosquamosal. Mental retardation is often seen in patients with multiple synostotic sutures; this occurs as a result of prolonged restriction of brain growth and cranial vault development secondary to fusion of the overlying sutures.

Scaphocephaly is associated with synostosis of the sagittal suture. Affected patients have an elongated, narrow cranial vault. In contrast, patients with bilateral coronal synostosis have brachycephaly, in which the frontal portion of the skull is wide and flat and the cranium is shortened anteroposteriorly. Lambdoid suture synostosis is also known as occipital or posterior plagiocephaly. This condition, characterized by an oblique posterior flattening, is rarely seen congenitally and more often results from childbirth or prolonged head positioning, such as during sleep. Infants with metopic suture synostosis, or trigonocephaly, have a triangularly shaped forehead with decreased bitemporal distance. Unilateral coronal synostosis, or frontal plagiocephaly, involves oblique frontal flattening of the skull.

184
Q

A 2-month-old male infant is brought to the office because of mid face hypoplasia, craniosynostosis, and bilateral hand and foot anomalies. A photograph of the left foot is shown. This patient most likely has which of the following syndromes?

A) Apert
B) Crouzon
C) Goldenhar
D) Nager
E) Treacher Collins

A

The correct response is Option A.

The patient described has Apert syndrome. This autosomal dominant syndrome is characterized by bicoronal craniosynostosis that leads to turribrachycephaly, mid face hypoplasia, and complex hand and feet syndactyly. Patients with Crouzon syndrome, an autosomal dominant disorder, typically have craniosynostosis involving the coronal, sagittal, and lambdoid sutures, as well as turribrachycephaly. Other findings include mid face hypoplasia, exorbitism, and proptosis. The extremities are normal.

Goldenhar syndrome, or oculoauriculovertebral dysplasia, involves asymmetry of the hard and soft tissues of the face. This condition is most commonly unilateral but may be seen bilaterally in some patients. Manifestations of this syndrome include hypoplasia involving the mandible and underlying soft tissues of the face, epibulbar dermoids, and varied degrees of microtia on the affected side. Most patients have associated vertebral abnormalities. Nager syndrome, or acrofacial dysostosis, is an autosomal recessive disorder characterized by craniofacial and upper extremity abnormalities. Patients with Nager syndrome have hypoplasia of the orbits, zygoma, maxilla, mandible, and soft palate. Auricular defects may also be present. Hypoplasia or agenesis occurs in the radius, thumbs, and metacarpals. Some patients may have radioulnar synostosis and elbow joint deformities. Patients with Treacher Collins syndrome, or mandibular dysostosis, have hypoplasia of the zygoma, maxilla, and mandible, downward slanting of the palpebral fissures, colobomas of the lower eyelids, absence of eyelashes, and auricular defects.

185
Q

An 11-year-old girl who is undergoing evaluation because of the appearance of her nose. Examination shows a short, flattened nasal bridge and midface hypoplasia. The anterior nasal spine is absent on radiographs. Which of the following is the most likely diagnosis?

(A) Binder’s syndrome
(B) Goldenhar’s syndrome
(C) Nager’s syndrome
(D) Treacher Collins syndrome
(E) Velocardiofacial syndrome

A

The correct response is Option A.

This 11-year-old girl has Binder’s syndrome, which is characterized by localized nasomaxillary hypoplasia resulting in a flat nasal bridge and a short, retracted columella. The columella and upper lip are depressed into the floor of the nose, and the anterior nasal spine is absent. Angle class III malocclusion is usually present.

Goldenhar’s syndrome, or oculoauriculovertebral dysplasia, involves asymmetry of the hard and soft tissues of the face. This condition is most often unilateral but may be seen bilaterally in some patients. Manifestations of this syndrome include hypoplasia involving the mandible and underlying soft tissues of the face, epibulbar dermoids, and varied degrees of microtia on the affected side. Most patients have associated vertebral abnormalities.

Nager’s syndrome, or acrofacial dysostosis, is an autosomal recessive disorder characterized by craniofacial and upper extremity abnormalities. Patients with Nager syndrome have hypoplasia of the orbits, zygoma, maxilla, mandible, and soft palate. Auricular defects may also be present. Hypoplasia or agenesis occurs in the radius, thumbs, and metacarpals. Some patients may have radioulnar synostosis and elbow joint deformities.

Patients with Treacher Collins syndrome, or mandibular dysostosis, have hypoplasia of the zygoma, maxilla, and mandible, downward slanting of the palpebral fissures, colobomas of the lower eyelids, absence of eyelashes, and auricular defects.

Velocardiofacial syndrome is characterized by overt or submucous clefting of the palate and cardiac abnormalities. Most patients have abnormal facial features, including narrow palpebral fissures and a prominent nose with a square nasal root and narrow alar base. The anterior nasal spine is present.

186
Q

Which of the following craniosynostotic disorders is NOT characterized by anomalies of the extremities?

(A) Apert syndrome
(B) Carpenter syndrome
(C) Crouzon syndrome
(D) Nager syndrome
(E) Pfeiffer syndrome

A

The correct response is Option C.

Patients with Apert syndrome have symmetric syndactyly of the hands and feet; other findings include synostosis of multiple sutures, exorbitism, and midface hypoplasia. In Carpenter syndrome, partial digital syndactyly and preaxial polysyndactyly of the feet are combined with suture synostosis. Nager syndrome is an autosomal recessive disorder in which the extremity anomalies range from hypoplasia to agenesis of the radius, thumbs, and metacarpals. Hypoplasia of the orbits, zygoma, maxilla, and mandible and auricular defects are also found. Patients with Pfeiffer syndrome have broad thumbs and halluces in addition to the suture synostosis. Partial syndactyly of the second and third digits has also been identified.

Crouzon syndrome is characterized by craniosynostosis, exorbitism, and midface retrusion; the extremities are unaffected.

187
Q

A 25-year-old woman comes for evaluation because she desires surgical correction of a gummy smile and a weak chin. On examination, she has clinical signs consistent with long face syndrome. Which of the following is the most appropriate management?

(A) Anterior segmental maxillary osteotomy with intrusion of the anterior segment
(B) Le Fort I osteotomy with inferior repositioning of the maxilla
(C) Le Fort I osteotomy with superior repositioning of the maxilla and genioplasty
(D) Sagittal split osteotomy with mandibular advancement and genioplasty
(E) Sliding genioplasty

A

The correct response is Option C.

In this patient who has long face syndrome resulting from vertical maxillary hyperplasia, the most appropriate management is Le Fort I osteotomy with superior repositioning of the maxilla combined with genioplasty. Long face syndrome is characterized by excessive length of the lower third of the face. Affected patients have lip incompetence with the lips in repose, a large interlabial gap, and excessive maxillary incisor show with the upper lip at rest; normal maxillary incisor show is quantified as 2 mm to 3 mm. In addition, the nasolabial angle is obtuse and the alar bases are narrow and constricted. Occlusion is often Angle class II with an anterior open bite. The chin is often vertically long and somewhat retruded.

Le Fort I osteotomy with superior repositioning of the maxilla (intrusion) should be performed for management of this patient’s vertical maxillary excess. If autorotation alone does not correct the chin positioning, genioplasty can be performed as well. Sagittal split osteotomy with mandibular setback may also be considered in order to effectively treat the malocclusion without significantly repositioning the maxilla.

Anterior segmental osteotomy with intrusion will treat the transverse maxillary arch deformities but not the vertical maxillary excess. Le Fort I osteotomy with inferior repositioning will only further increase lower facial height. As mentioned above, sagittal split osteotomy with mandibular advancement and genioplasty can be used in conjunction with Le Fort I osteotomy in this patient to correct the Angle class II malocclusion and/or any facial asymmetry, but will not address all of this patient’s facial concerns if performed alone. Sliding genioplasty will treat the weak chin only, and not the maxillary excess, nose and lip findings, or malocclusion.

188
Q

According to the Tessier classification, which of the following clefts is most closely associated with macrostomia?

(A) No. 1
(B) No. 3
(C) No. 5
(D) No. 7
(E) No. 9

A

The correct response is Option D.

The Tessier classification of craniofacial clefting was first proposed in 1973. This system integrates both tissue findings and underlying skeletal deformities; embryopathogenesis is not considered. According to the Tessier system, the No. 7 cleft manifests as macrostomia and absence of the zygomatic arch. This common sporadic cleft, which has variable expressivity, is more likely to be seen in male neonates and occurs in one of every 3000 births.

The Tessier No. 1 cleft lies just lateral to the midline, beginning at the cupid’s bow and passing through the dome of the nostril lateral to the anterior nasal spine. Notching of the alar dome is a distinctive feature. The nasal bone may be absent, but the septum is unaffected. Hypertelorism and encephalocele may also be associated.

The Tessier No. 3 cleft involves the orbit. It begins in the alveolus between the lateral incisor and canine and extends through the maxilla into the lacrimal bone. This cleft is commonly referred to as a naso-ocular cleft because the inferomedial wall of the orbit is absent. Associated soft-tissue deformities include shortening of the nose, colobomas of the nasal alae and the lower eyelids medial to the punctum, obstruction of the nasolacrimal duct, malformation of the lower canaliculus, and hypoplasia of the medial canthal tendon.

With the rare Tessier No. 5 cleft, the cleft moves laterally and becomes oblique. It begins beneath the canine and extends through the maxillary sinus to the orbital floor. Colobomas of the lateral lower eyelids and clefting of the upper lip medial to the oral commissure are associated.

The Tessier No. 9 cleft is merely a supraorbital extension of the Tessier No. 5 cleft.

189
Q
A